You are on page 1of 99

By

Dr. M. B. Lal
&
Ashok Gupta

UPKAR PRAKASHAN, AGRA2


www.upkar.in

Publishers

Publishers
UPKAR PRAKASHAN
(An ISO 9001 : 2000 Company)

2/11A, Swadeshi Bima Nagar, AGRA282 002


Phone : 4053333, 2530966, 2531101
Fax : (0562) 4053330, 4031570
E-mail : publisher@upkar.in
Website : www.upkar.in

Branch Offices
4845, Ansari Road, Daryaganj, 1-8-1/B, R.R. Complex (Near Sundaraiah Park,
New Delhi110 002 Adjacent to Manasa Enclave Gate), Bagh Lingampally,
Phone : 01123251844/66 Hyderabad500 044 (A.P.), Phone : 04066753330

The publishers have taken all possible precautions in publishing this book, yet if
any mistake has crept in, the publishers shall not be responsible for the same.
This book or any part thereof may not be reproduced in any form by
Photographic, Mechanical, or any other method, for any use, without written
permission from the Publishers.
Only the courts at Agra shall have the jurisdiction for any legal dispute.

ISBN : 978-93-5013-190-9

Price : 60/-
( Sixty Only)
Code No. 995

Printed at : UPKAR PRAKASHAN (Printing Unit) Bye-pass, AGRA


CONTENTS

1. Questions and Statements. 38


2. Verification of the Truth of the Statement 912
3. Statement and Conclusion 1318
4. Syllogism.. 1937
5. Statement and Argument.. 3843
6. Statement and Assumptions. 4448
7. Passage. 4952
8. Statement and Course of Action... 5358
9. Decision Making.. 5965
10. Cause and Effect... 6669
11. Data Sufficiency... 7074
12. Analytical Reasoning or Problem Solving 7595
1. Classification or Categorization 76
2. Blood Relations with their Relationship and Professions 78
3. Seating and Placing Arrangements... 82
4. Comparison Type Test.. 85
5. Sequential Order of Things... 86
6. Selection Based on Certain Given Pre-conditions 88
7. Jumbled Problems. 90
Logical Reasoning
&
Analytical Ability
1 Questions and Statements
In such type of test, a question is given (B) If only statement II is sufficient to
followed by two statements. You have to find out answer the question.
that which statements is/are sufficient to answer (C) If either I or II alone is sufficient to
the question ? answer the question.
ExampleQuestionWho is brother of B ? (D) If both the statements I and II are
StatementI. B is brother of A. sufficient to answer the question.
II. Bs father is husband of As (E) If both the statements I and II are not
mother. sufficient to answer the question.
Give Answer
(A) If only statement I is sufficient to answer 1. Lal is taller than Nand. Jim is taller than
the question. Herry. Who is the tallest of them ?
(B) If only statement II is sufficient to Statement I. Jim is taller than Nand.
answer the question. II. Lal is taller than Harry.
(C) If either I or II alone is sufficient to 2. In our school how many students of class V
answer the question. are there, who play chess ?
(D) If both statement I and II are not
sufficient to answer the question. Statement I. No girl likes to play chess.
(E) If both statement I and II are sufficient to II. There are 40 students in
answer the question. Class V.
Answer with Explanation (D)From the 3. Jay is 10th student in the class. How many
statement I, B is brother of A hence A may be students are in class.
brother of B and A may be sister of B. Therefore, Statement I. His friends rankes 58 in that
statement I is not sufficient to answer the question class which is the last rank.
from the statement II, Bs father is husband of As
mother. Hence, A and B are the children of single II. Jay ranks 49th from the
parents, but it is not certain that A is daughter or opposite end (another end).
son. Therefore, statement II is also not sufficient 4. What is Tomars age ?
to answer the question. Statement I. Tomar could not appear in
Beside this if we combine both the statements, the last examination. He is
still there is no answer for the question. Therefore, two months short than the
option D is our answer. required age which is 18
Exercise years.
Directions(Q. 132) Each question consists II. He will be able to cast his
of two statements I and II given below it. You vote this year and the
have to find out all possible combinations which minimum age for voting is
could be sufficient for answering a question. Read 18 years.
both the statements 5. How many brothers does Ramu have ?
Give Answer Statement I. Ramus father has three
(A) If only statement I is sufficient to answer children.
the question. II. Ramu has two sisters.
4 | Logical Reasoning & Analytical Ability

6. Who is the tallest in five friends ? II. Vikash is 3 shorter than


Statement I. D is taller than A and C. Pranab.
II. B is shorter than E but taller 15. In the game of playing cards there are four
than D. members, who is Cs partner ?
7. At what time did the train depart ? Statement I. D is sitting opposite to A.
Statement I. Generally the train departs II. B is sitting in the the right
at right time. of A and in the left of C.
II. The scheduled departure
time is 14 : 30. 16. On which day of January Suhas departed to
Germany ?
8. When will be Chandras birthday this year ?
Statement I. It occurs between 11 and 13 Statement I. Suhas has passed 10 years
February. in Germany till now.
(Thursday occurs on 11 II. Suhass friend Anil departed
February) to Germany on 15 Feb. and
II. It does not occur on he accompanied Suhas 20
Saturday. days after Suhass arrival.
9. How many students of science got more than 17. On which day Robin met an accident ?
60% marks ? Statement I. Robin left hospital on
Statement I. 35% from all the students Saturday 3 days after the
got 60% marks. accident.
II. The students who got 60% II. Robin was going on Scooter
marks are 55% from on Monday.
science.
18. In which year Rahul was born ?
10. How many matches will be played between A
and B in this competition ? Statement I. Rahul is 25 year younger
Statement I. A has already won 3 than his mother.
matches against B. II. Rahuls brother who was
Simultaneously it has also born in 1964 is 35 year
younger than his mother.
won the competition after
winning 3rd match. 19. What is monthly salary of Srikant ?
II. Fifth match will be played Statement I. Srikant gets 15% more than
next week. Subodh while Subodh get
11. What is relation between D and A ? 10% less than Prabhat.
Statement I. B is brother of A. II. Prabhats monthly salary is
II. B is son of D. Rs. 2500.
12. How many times is Utpal heavier than Vilas ? 20. How long did a man travel on a particular
day ?
Statement I. Vilass weight is 65
Statement I. His average was 160 km /
kilogram.
day in last 10 days.
II. Amars weight is 10 kg
more than utpals weight. II. He walked 8 hours with an
average speed of 20 kms.
13. In which year was Subhash born ?
21. A, B and C left a cinema hall and they forgot
Statement I. In 1989 Subhashs mother
the bag. Who forgot the bag there ?
was 44 years old.
Statement I. When they were entering
II. Subhash was born 3 years into the cinema hall B did
after his mothers marriage. not have any bag.
14. What is Ashoks height ?
II. C had his own bag when
Statement I. Pranabs height is 59" and they came out from the
he is 4" taller than Ashok. cinema hall.
Logical Reasoning & Analytical Ability | 5

22. Of three buildings, which is the lowest ? II. The third Saturday of that
Statement I. All buildings follow the month fell on 17th.
rules formed by the 29. Is Shridar capable to get entrance in a
corporation. company ?
II. Of these two buildings have Statement I. Company does not permit
similar height. the entrance of any un-
23. What is the weight of a person ? known person.
Statement I. If the weight of this person II. All the workers are capable
is not included the average to get entrance.
weight of the group 30. What is the reason that most of the musicans
decreases 2 kg while their are muslims ?
average weight was 30 kg.
Statement I. Besides Muslim religion no
II. The weight of this person one of the other religions
and his friend is 87 kg. encouraged the fine art.
24. How many cubes of ice can pervade in a II. Muslim did not study in the
vessel whose capacity is 80 C.C. ? western style.
Statement I. The length, width and 31. Why did Indian Scientists not progress in any
height of ice cube is 1/2 cm. field ?
II. 640 cubes of ice occupy
Statement I. Indian Scientist did not get
320 C.C.
the facilities for advanced
25. What was the total sale of the company ? scientific experiments.
Statement I. The company sold 8000 II. Indian Scientist thinks that
piece to A @ 25/- per piece. the knowledge of western
II. This company does not scientific progress is
manufacture any other sufficient for the progress
product. of any country.
26. What is Rajus age ? 32. Does coffee drinking cause the headache ?
Statement I. Raju, Vimala and Kishore Statement I. The stimulating of pan-
belong to same age group. crease causes headache.
II. The sum of the age of II. Coffee contains caffein
Vimala, Kishore and which stimulates pancrease
Abhishek is 32 and much.
Abhisheks age is equal to
the sum of Vimala and Directions(Q. 3337) In the followings
Kishores age. each question followed by two statement I and II
are given. You have to ascertain that the
27. What is the quantity of the exported rose information given in the statement is sufficient or
apple of Jhandoba ? not. Read carefully both the statements and
Statement I. 80,000 ton of Jamun has answer
been exported to America (A) If the information given in statement I is
from Jhandoba which is sufficient to answer the question whereas the
10% of the total export of information given in statement II is not sufficient
Jamun. to answer the question.
II. The export of Jamun is (B) If the information given in the statement
125% against the targeted II is sufficient to answer the question whereas the
export 19 ton from information given in the statement I is not
Jhandoba. sufficient to answer the question.
28. Which will be 14th day of a month ? (C) If either the information given in the
Statement I. The last day of that month statement I or in II is not sufficient to answer the
is Wednesday. question.
6 | Logical Reasoning & Analytical Ability

(D) If both the information given in the II. The position of hours hand
statements I and II are not sufficient to answer the is on P at 7 P.M.
question. Direction(Q. 3842) In each of the follow-
(E) If both the information given in the ing given questions there is a question followed by
statement I and II are sufficient to answer the two statements I and II are given. You have to
question. find out that the information given in the
33. In which direction is Shashidhar facing ? statements is sufficient to answer or not. Read
both the statements.
Statement I. Early morning, Shashidhar
was standing on the ground Give Answer
in front of an effigy. The (A) If only statement I is sufficient to answer
shadow of the effigy was the question.
ap-pearing towards the right (B) If only Statement II is sufficient to answer
side of Shashidhar. the question.
II. Early morning Shashidhar (C) If either I or II alone is sufficient to
was standing on the ground, answer the question.
when he turned towards the (D) If both the Statements I and II are
left. The shadow was sufficient to answer the question.
appearing behind him. (E) If both the Statements I and II are not
sufficient to answer the question.
34. Who teaches History from P, Q, R, S & T ?
38. What is the meaning of come in a coded
Statement I. From them, each teaches language ?
only one subject Q teaches
Statement I. The meaning of pit na ja
Maths whereas T Teaches
is come and go in that
Science. P or R does not
coded language.
teach Geography. S or P
does not teach English. II. The meaning of na dik sa
is you may go in that
II. T and R are the teachers of coded language.
Science and English. P is
39. What does the relation P have with M ?
the teacher of Math.
Statement I. P is brother of K and T.
35. Facing towards the South, who is just left to
II. T is daughter of Q and
Ramakant in a row of the boys ?
sister of Ms daughter.
Statement I. Suresh is just right of 40. Who is the tallest amongst M, T, R, K and
Chandrakant and he is Q?
fourth to the left of
Statement I. T is taller than R, M and Q
Ramakant.
but shorter than K.
II. Suresh is third to the left of II. R is shorter than T and M
Ramakant and Naresh is but taller than Q.
second to the left of Suresh.
41. In which direction is D from P ?
36. Among the six friends A, B, C, D, E and F Statement I. S is in the South from P
who got the maximum marks ? which is in the West of D.
Statement I. B got fewer marks than A II. P and R is in a straight line
and F but not fewer than C, and R is in the South from
D and E. D.
II. F did not get more marks 42. In which month of the year Mohan was born ?
than B but not as A. Statement I. Mohan was born in the
37. What will the position of hours hand in the winter.
clock at 7.30 P.M. ? II. Mohan was born exactly
Statement I. English alphabets are after 14 month of his sister,
placed in the dial of clock birth and she was born in
in place of number. the month of October.
Logical Reasoning & Analytical Ability | 7

Answers with Explanation Weight of new person = 620 560


1. (D) Lal > Nand (i) = 60 kg
Jim > Harry (ii) 24. (C) 25. (E)
We cannot determine the exact answer even 26. (E) In Statement II, Vimala + Kishore +
after considering both the statements. Abhishek = 32
2. (D) Both the statements can not derive any Abhisek = Vimala + Kishore
conclusion. Vimala + Kishore = 16
3. (B) According to II statement, the number of The age of Raju, Vimala and Kishore is the
all the students same. Hence, the age of Raju is 8 years.
= (10 + 49) 1 27. (C) 28. (B) 29. (D) 30. (A) 31. (A)
= 58 32. (E)
4. (A) Tomars age = 17 years 10 month. 33. (C) From Statement I, since, the shadow of
effigy is appearing towards the right side of
5. (E) Ramus father has three children in which Shashidhar and the shadow of any object
two are sisters. Clearly, Ramu is the only son appears towards the West direction in every
in the family. Thus, we can say, Ramu has no morning, hence the direction of right side of
brother. Shashidhar was West. Hence, Shashidhar was
facing towards South.
6. (E)
From Statement II, the shadow was appearing
7. (D) From the Statement I, train Generally in the West when Shashidhar turned towards
leaves at the right time but it is not clear that left, it means that his left is East. Hence,
the train always leaves at right time. It might Shashidhar was facing towards South.
be possible that the train would not have left
at right time. Hence, Statement II is not Thus, we can say that the information given
sufficient to answer. in Statement I or II is sufficient to answer.
34. (A) From Statement I
8. (A) From the Statement I, clearly, it is 12
February. PHistory
QMathematics
9. (D) 10. (E) REnglish
11. (D) From the Statements, it is not mentioned SGeography
that A and D are male or female. TScience
12. (D) There is no relation among Utpal and Hence, History is taught by P.
Vilass weight. From Statement II,
History is taught either by Q or R, hence, the
13. (D) In which year, Shuhas mother married, information given in Statement I is sufficient
it is not clear. to answer.
14. (A) Ashoks height is 5 9 4 = 5 5. 35. (B) From Statement I
15. (C) 16. (D) 17. (A) 18. (E) 19. (E) Right Left

20. (B) 21. (E) 22. (D) Ramakant Suresh Chandrakant


23. (A) The average weight of 21 persons (Facing towards South)
including the new person = 30 kg Hence, we cannot determine who is right of
Total weight = 21 30 Ramakant.
= 620 kg From Statement II,
Average weight of 20 persons excluding the Right Left
person = 28 kg
Total weight = 28 20 Ramkant Naresh Suresh

= 560 kg Hence, Naresh is sitting left of Ramakant.


8 | Logical Reasoning & Analytical Ability

36. (E) From Statement I, 38. (D) From Statement I and II,
A & F > B > C, D and E na go
From Statement II, From I, 9 Either pit or ja
A>F>B 39. (E) From the Statement II, T is Ms
From both the Statements I and II daughters sister means T is Ms daughter.
A > B > C, D and E From the Statement I,
Hence, it is clear with both the statements that P is Ts brother
A got maximum marks. P is Ms son.
37. (D) By placing letter in place of number in 40. (C) From Statement I, T is taller than R, M
the clock. and Q, but shorter than K.
1 A, 2 B, 3 C, 4 D, 5 E, 6 F, K is the tallest.
7 G, 8 , 9 , 10 J, 11 , From Statement II R, T and M are shorter
12 L than K but taller than Q. Hence, K is the
Hence, it is clear that the position of hand of tallest.
the clock between 7 and 8, or G and H, but
the G and H are imaginary letters, therefore, 41. (A) From the Statement I, S is in South of P
no inference can be drawn by the Statement I. and P is in West of D. Hence, D is East of P.
From Statement II. 42. (B) From Statement II, Mohan was born
7G exactly 14 month after his sisters birth and
But here, no letter is given for 8, hence we his sister was born in October. Hence, Mohan
cannot determine. was born in January.


Verification of the truth
2 of the Statement
In such type of test, a incident is given and it 2. A boy is sitting on the back-seat of the car.
is to be found that this incident occurs always or When the driver of the car starts it suddenly,
never. In some cases, any word or sentence is the boy bends towards the backside
given and then a question is asked. Which is the (A) Always (B) Generally
most important factor/element in that word or (C) Sometimes (D) Never
sentence ? The questions are based upon
Geographical, Scientific, Social, and Sports and 3. A lady in India can marry with the brother of
Games related information. deceased husband but no man can marry the
Example 1. A person is going towards North sister of deceased wife
in the night and he sees Dhruv Tara and sees it on (A) Always (B) Generally
his left side. Which of the following statements (C) Sometimes (D) Never
justify the fact ?
4. Yesterday I saw an ice-cube which had been
(A) Always (B) Sometimes melted in the heat of the hearth
(C) Generally (D) Never (A) Always (B) Sometimes
Answer with Explanation (D)Since, we (C) Generally (D) Never
know that Dhruv Tara always appears on North
and that person is also going towards North, 5. My 10 year niece is taller than my 12 year
clearly, he faces Dhruv Tara and it never appears son
on his right side. (A) Always (B) Generally
Example 2. Bewilderness always com- (C) Sometimes (D) Never
prises 6. A boy driving a bicycle put the brake
(A) Difficulty (B) Anxiety suddenly he bends forward
(C) Helpless state (D) Sharpness (A) Sometimes (B) Never
Answer with Explanation (B)Bewilderness (C) Generally (D) Always
always comprises mental tension and mental
7. Sun shines on the pole at midnight
tension causes anxiety.
(A) Never (B) Sometimes
Excercise1 (C) Always (D) Generally
DirectionIn each of the following questions,
many alternative answers are given for a statement. 8. High tides occurs on Maha-ashtami
Out of these alternatives only one answer verifies (A) Sometimes (B) Never
the statement. Find out the correct alternative. (C) Generally (D) Always
1. If we are going towards South in the morning, 9. The main work of Rajya is to formulate the
we will see the Sun rising on the left side law
(A) Always (A) Never
(B) Sometimes (B) Sometimes
(C) Generally (C) Always
(D) Never (D) Generally
10 | Logical Reasoning & Analytical Ability

10. Whenever the warm current and cold current 3. Justice is always related to
interact with each other, there is always a (A) Deceit (B) Generosity
fog (C) Just (D) Nobility
(A) Never (B) Sometimes
(C) Never (D) Always 4. A Tree always has
(A) Leaves (B) Flowers
11. An inferior egg always sinks in water
(C) Roots (D) Fruits
(A) Never (B) Sometimes
(C) Always (D) Generally 5. An animal always has
(A) Lungs (B) Skin
12. Ice-cube floats on water
(C) Brain (D) Heart
(A) Generally (B) Sometimes
(E) Life
(C) Never (D) Always
6. A car must have
Answers with Explanation
(A) Driver (B) Bonnet
1. (A) Sun rises in the East in the morning. If we
(C) Dicky (D) Bumper
go towards South direction in the morning,
we face towards South and our left hands (E) Wheels
position is on East direction. Hence, going 7. Run must have
towards Southwards Sun always appears (A) Umpire (B) Competitors
rising from our left side. (C) Spectators (D) Victory
2. (A) When car suddenly starts to move, the (E) Prize
person who is sitting on the back-seat always 8. A book always comprises
bends towards backward.
(A) Lessons (B) Pages
3. (C) There is no such type of custom, hence it (C) Figure (D) Content
may happen sometimes.
9. In a dispute there must be
4. (D) The piece of ice is placed near the (A) Hatred (B) Injustice
burning hearth it melts with the heat of the
hearth and it never happens that the ice had (C) Dissent (D) Excitement
melted and later we saw the piece of ice. 10. Bargaining always comprises
5. (C) Since, the age of a person does not (A) Generosity (B) Baseness
depend on his/her height, there are many (C) Give and take (D) Costliness
other factors which affect the height. 11. Bravery must have
6. (D) According to the rule of science, a cyclist (A) Courage (B) Intelligence
uses the brakes suddenly he always bend (C) Power (D) Experience
down towards the forward direction.
12. Astonishment always has
7. (C) 8. (B) 9. (A) 10. (C)
(A) Crowd (B) Surprise
11. (A) An inferior egg has low density therefore
(C) Wastage (D) Rustic
it sinks on the water.
12. (D) Ice is lighter than water hence the piece 13. Blameless always has
of ice always float on the water. (A) Punishment (B) Blame
Excercise2 (C) Prevention (D) Scold
1. Election must have 14. Mentality always has
(A) Procession (B) Speech (A) Cruelly (B) Perception vision
(C) Slogans (D) Candidates (C) Indifference (D) Impulse
2. Hills always comprise 15. A river must have
(A) Trees (B) Animals (A) Fish (B) Weeds
(C) Water (D) Height (C) Banks (D) Boats
Logical Reasoning & Analytical Ability | 11

16. A factory must have 28. In every singing, it should have


(A) Chimney (B) Labour (A) Chorus (B) Musician
(C) Electricity (D) Seller (C) Drum (D) Words
17. A paragraph always has 29. A lotus always comprises
(A) Name System (B) Group of words (A) Petals (B) Mud
(C) Expression (D) Structure (C) Roots (D) Water
18. Women is always 30. Milk always comprises
(A) Beautiful (B) Black
(A) Cream (B) Oilness
(C) Charming (D) Fair
(C) Whiteness (D) Water
(E) Lengthy
19. A husband is always than his wife. 31. A camera always comprises
(A) Taller (B) More handsome (A) Reel (B) Photograph
(C) Bigger (D) More intelligent (C) Flash (D) Lens
(E) None of these 32. Dispute always comprises
20. Bulky persons life is always (A) Hatred (B) Opposition
(A) Longer (B) Merry (C) Injustice (D) Anger
(C) Healthy (D) Short
33. Disease always comprises
(E) None of these
(A) Treatment (B) Medicine
21. A debate always comprises (C) Germs (D) Reason
(A) Room (B) Speaker
(C) Stage (D) Audience 34. A motor cycle always comprises
(E) Typist (A) Brake (B) Mirror
(C) Engine (D) Horn
22. A college always comprises
(A) Books (B) Building Answers with Explanation
(C) Black-board (D) Teacher 1. (D) An election cannot be organised without
(E) Register candidates.
23. Farming always comprises 2. (D) Hills are always higher than plains.
(A) Cultivator (B) Tractor 3. (C) Justice is guided by the laws.
(C) Spade (D) Land 4. (C) The existence of a tree is not possible
(E) Labour without its roots.
24. A country comprises 5. (E) Every animal has life.
(A) Aeroplane (B) King
6. (E) No car can run without wheels.
(C) Army (D) Railway
(E) Area 7. (B) In any run, there should be at least two
contenders hence there must be a competitor.
25. Danger always comprises
8. (B) No book can be prepared without the
(A) Enemy (B) Attack pages.
(C) Fear (D) Help
9. (C) The meaning of dispute is
26. In management, is always followed. Discussion on two different views or
(A) Counselling (B) Rules opinions. Different views or opinions always
(C) Encouragement (D) Undue Pressure comprise disagreement.
27. Disclosure always comprises 10. (C) Bargaining always comprises give and
(A) Mystery (B) Display take of any thing, commodity or article.
(C) Representative (D) Delivering 11. (A) Bravery must have courage.
12 | Logical Reasoning & Analytical Ability

12. (B) Astonishment and surprise are synonyms 23. (D) Farming is always done on the land.
to each other. 24. (E) A country should have area.
13. (B) The meaning of blameless is to free from 25. (C) Danger always comprises fear because of
blame. Hence, in blameless, there should insecurity.
always be blame.
26. (B) Management always gives improvement
14. (D) Impulse caues mentality. and for improvement, there should be some
15. (C) A river must have its bank. rules to follow.
16. (B) No factory can run without labour. 27. (A) Disclosure means to tell a mystery and
17. (B) Paragraph is a group of words. mystery means a hidden fact or point or topic.
Hence, for disclosure there should be a
18. (C) Every lady or woman is charming. mystery.
19. (E) It is not necessary that a husband must be
28. (D) For every singing , there should be some
taller and more handsome, bigger, more
words.
intelligent than his wife.
20. (D) The life of bulky person are always short 29. (A) A lotus flower always has petals.
because they are affected by various diseases. 30. (C ) Milk is always white.
21. (B) There must be a speaker in any debate. 31. (D) A camera must have lens.
22. (D) No college can run without a teacher. 32. (B) 33. (D) 34. (C)

3 Statement and Conclusion
In such type of test, a statement is given Conclusions
followed by its four conclusion. The candidate is (A) Recession causes unemployment
required to find out the correct conclusion, (B) The purchasing power of a person
whether it might be wrong. decreases on account of unemployment
ExampleStatementSome of the foot- (C) Unemployment causes recession
ball-players are not the teacher of the college. (D) The life of the people who live in the
Conclusion developed countries is better than that of
(A) No football player is a teacher of the less developed countries
college 3. StatementScientists are the biggest enemy
(B) Some teachers of the college may be of mankind.
football player
Conclusions
(C) Some players of the college, may be
(A) They have invented many harmful medi-
teacher
cines
(D) Shree Rajan who plays the chess, cannot
(B) They have invented many destructive
be teacher of the college
weapons
Answer with Explanation (B)If some (C) They were unable to prepare a healthy
teachers of the college are not football players the environment for all
remaining teachers of the college may be football-
(D) They are serving only for the interest rich
players. Hence, option (B) is correct answer.
nations
Exercise1 4. StatementThe family planning programme
Directions(Q. 15) A statement is given in in India has partly succeeded.
each of the following questions and it is followed Conclusions
by its four alternatives in which one alternative (A) The population growth rate in India is
either support or refute the statement. Find out the still very high
alternative from each question. (B) In rural areas, there is no arrangement for
1. StatementUnited States of America always proper health care
favour Israel. (C) The media of Telecommunication are not
Conclusions playing their roles to carry out the
(A) The headquarter of UNO is in United messages effectively
States of America (D) The criticism of limited family is
(B) Jewish Lobby in U.S.A. is very powerful restricted by our religious belief
(C) United States of America agreed to form 5. StatementIn Hindu Society there is a very
a committee to investigate the assassi- much prejudice against a female infant.
nation of 18 Palestinians by Israels Conclusions
soldiers (A) Males are physically stronger than
(D) Israel is capable to defend itself females
2. StatementIndustrial development promotes (B) Male infants are supposed to bring
the chances of employment. prosperity in the family
14 | Logical Reasoning & Analytical Ability

(C) Hindu Society is paternal in nature (C) Some of the students are those who have
(D) Various methods are being used to detest low standard
and abort a female infant in our society (D) All students are talented
6. StatementUsing too much time in T.V. 10. StatementMost of the pens of that shop are
viewing affects on eye-sight. Which of the costly.
following four alternatives is true ? Conclusions
Conclusions (A) There are no cheap pens in that shop
(A) T.V. viewing is a wastage of time and (B) Some pens are costly in that shop
energy as well and one can feel fatigue (C) Some pens are cheap in that shop
or tiredness if one watches for a longer
period (D) Camlin pens are costly in that shop
(B) If one watches T.V. in a bad posture, it 11. StatementIf a person is a rich, he is very
affects ones eye-sight influential.
(C) The persons who have problems with Conclusions
their eye-sight should not watch T.V. (A) John has much influence, therefore, he is
programmes rich
(D) 60% of the eyes problems are caused by (B) If a person is not rich, he has no
T.V. watching for a longer period influence
7. StatementTo succeed in examination one (C) Kasim is a rich so he is very influential
should work hard. Which of the given four person
conclusions is true ? (D) Poor persons have no influence
Conclusions 12. StatementSmokers are affected by cancer.
(A) Those who work hard always get success Conclusions
(B) Examination is closely related with hard (A) Many of the cancer patients are smokers
work (B) Smoking may cause cancer
(C) No one can get success without hard (C) Most of the smokers may be affected by
work cancer
(D) One who does hard work, always (D) Smoking always causes cancer
satisfies
Directions(Q. 810) In each of the follow- 13. StatementThe book which I read yester-
ing questions a statement is given and it is day, was the best.
followed by its four conclusions A, B, C and D. Conclusions
Find out the conclusion which is correctly derived (A) The other books which I read, were not
from the statement. excellent
8. StatementSoldiers serve their country. (B) The book which I read yesterday, was
Conclusions the best book among all the books that I
(A) Those who serve their country, are have ever read
soldiers (C) The book of yesterday was as good as
(B) Women do not serve their country other books
because they are not soldiers (D) The book of yesterday was excellent
(C) Generally, men serve their country
14. StatementSome leaders are corrupt as
(D) Some men, who are soldiers, serve their Nagesh.
country
Conclusions
9. StatementMost of the students are talented.
Conclusions (A) Nagesh is not corrupt
(A) Some students are talented (B) Leaders are not corrupt
(B) No students are there who are not (C) Nagesh never be a corrupt
talented (D) Some leaders are corrupt
Logical Reasoning & Analytical Ability | 15

15. StatementTelevision assures its viewers enemy of mankind because they have
that they may fall prey to some criminal invented deadly destructive weapons.
activity and simultaneously, it is urged T.V. 4. (A) The population growth in India is still
viewers that they should bear the happening very high. Hence, the statement that the
siliently. family planning programme in India has
The above-mentioned statement may furnish partly succeeded is true.
one of the following conclusions. 5. (B) It is prevalent in Hindu Society that male
Conclusions child extends the family and he enhances the
(A) People should not watch T.V. Pro- wealth and prosperity. Hence, second
gramme alternative confirms the statement.
(B) Television intesifies the insecurity feeling 6. (D) 7. (C) 8. (D) 9. (C)
among its viewers 10. (C) 11. (C) 12. (C) 13. (B)
(C) T.V. viewers are more prone to prey in 14. (D) 15. (C) 16. (B) 17. (A)
criminal activities in comparison to other Type II
people
In such type of questions, a statement is given
(D) T.V. viewing is responsible for enhancing followed by two conclusions. The candidate is
of criminal inclination required to deduce the conclusion based upon this
16. StatementIgnorance is much delighted. statement. If the conclusion (a) is correct, the
Conclusions answer will be (A) and the conclusion is (b) is
I. Knowledge is unnecessary. correct, the answer will be (B) If both the
II. Ignorant people are lucky. conclusions are correct, the answer will C and If
(a) is correct and (b) wrong and (b) is correct (a)
(A) Only I wrong, the answer will (D). If there is no
(B) Only II conclusion follows, the answer will be (E).
(C) Both I and II Example
(D) Neither I nor II StatementIf you are a refined artist, we
17. StatementFortune favours the brave. have a right job for you to prove your worth.
Conclusions Conclusions
I. The people who endanger themselves are (a) You are a refined artist.
more successful. (b) We a refined artist.
II. Coward can never be a lucky. (A) 1 (B) 2
(A) Only I (C) 3 (D) 4
(B) Only II (E) 5
(C) Both I & II Answer with Explanation (B)In the given
(D) Neither I nor II statement the word If is given which cannot
Answers with Explanation confirm that you are a refined artist. Hence, the
first conclusion can not be followed by the
1. (C) U.S.A. endorsed the formation of a
statement. But it is given in the statement, if you
committee for the investigation into the death
are a refined artist and we have a right job to
of 18 Palestines by the soldiers of Israel. It
prove your worth. It is clear that we have a
indicates that U.S.A. does not always favour
requirement of a refined artist. Thus, the second
Israel.
conclusion confirms the statement.
2. (D) The living standard of a person in a
country depends upon his employment. If a Exercise2
country is developing, the opportunities for Directions(Q. 115) In each of following
the employment will increase. Hence, the questions a statement is given followed by two
fourth alternative confirms the statement. conclusions.
3. (B) The second alternative confirms the Give answer
statement that the scientist are the biggest (A) If only conclusion I follows,
16 | Logical Reasoning & Analytical Ability

(B) If only conclusion II follows, 7. StatementsGood voice is a natural gift but


(C) If either I or II follows, one has to keep practicising to improve and
(D) If neither I nor II follows and excel in the field of music.
(E) If both I and II follows. Conclusions
1. StatementsCompany X has marketed the I. Natural gifts need nurturing and care.
product, go ahead to purchase if its price and II. Even though your voice is not good, one
quality are in your consideration. keep practicising.
Conclusions 8. StatementAll the organised persons find
time for rest. Sunita inspite of her very busy
I. The product must be good in quality.
schedule, finds time for rest.
II. The price of the product must be
Conclusions
reasonable.
I. Sunita is an organised person.
2. StatementsVegetables prices are soaring in
the market. II. Sunita is an industrious person.
Conclusions 9. StatementsJade plant has thick leaves it
requires little water
I. Vegetables are becoming a rare com-
modity. Conclusions
II. People cannot eat vegetables. I. All plants with thick leaves require little
water.
3. StatementsThis world is neither good nor
evil, each man manufactures a world for II. Jade plants may be grown in places
himself. where water is not in abundance.
Conclusions 10. StatementsUntil your country achieves
economic equality, political freedom and
I. Some people find this world quite good.
democracy would be meaningless.
II. Some people find this world quite bad.
Conclusions
4. StatementsOur security investments carry
I. Political freedom and democracy go
market risk. Consult your investment advisor
hand in hand.
or agent before investing.
II. Economic equality leads to real political
Conclusions freedom and democracy.
I. One should not invest in securities. 11. StatementsIn case of outstanding candi-
II. The investment advisor calculates the dates, the condition of previous experience of
market risk with certainity. social work may be waived by the Admission
5. StatementsToday out of the world popula- Committee for M.A. (Social work).
tion of several thousand millions the majority Conclusions
of men have to live under Governments I. Some of the students for M.A. (Social
which refuse them personal liberty and the Work) will have previous experience of
right of dissent. social work.
Conclusions II. Some of the students for M.A. (Social
I. People are indifferent to personal liberty Work) will not have previous experience
and right to dissent. of social work.
II. People desire personal liberty and right 12. StatementsWater supply in wards A and B
to dissent. of the city will be affected about 50% on
6. StatementsIn diabetes, there is an excess Friday because repairing work of main lines
of sugar in the body. Our body needs sugar is to be carried out.
for energy, the quantity of sugar in excess of Conclusions
bodys requirement is excreted through urine. I. The residents in these wards should
Conclusions economise on water on Friday.
I. The excessive consumption of sugar is II. These residents in these wards should
likely to lead to diabetes. store some water on the previous day.
II. Consumption of sugar should be 13. StatementsThe government of country X
avoided. has recently announced several concessions
Logical Reasoning & Analytical Ability | 17

and offered attractive package tours for 17. StatementsW @ V, V # X, Y V.


foreign visitors. Conclusions
Conclusions I. X % Y
I. Now, more number of foreign tourists II. X $ W
will visit the country. 18. StatementsM $ K, K F, F % H.
II. The government of country X seems to Conclusions
be serious in attracting tourists. I. M # F
14. StatementsFrom the next academic year, II. M $ F
students will have the option of dropping 19. StatementsT @ L, L % N, D N.
Mathematics and Science for them school Conclusions
leaving certificate examination.
I. N # T
Conclusions
II. L % D
I. Students who are weak in Science and
20. StatementsN % R, R @ K, K # F.
Mathematics will be admitted.
Conclusions
II. Earlier students did not have the choice
of continuing their education without I. R @ F
taking these subjects. II. R # F
15. StatementsIndustrial revolution which first 21. StatementsH D, D # R, R @ L.
of all started has brought about modern age. Conclusions
Conclusions I. L @ H
I. Disparity between rich and poor results II. H # R
in revolution. 22. StatementsE H, K $ H, K @ M.
II. Revolution overhauls society. Conclusions
Directions(Q. 1622) Symbols @, %, #, $, I. E # K
are used with different meanings as explained II. E $ M
below Answers with Explanation
P @ Q means P is not greater than Q. 1. (E) It has been mentioned in the statement
P % Q means P is neither greater than nor that the person who considers price as well as
equal to Q. quality before buying a product should buy
P # Q means P is neither smaller than nor the product of company X. Hence, both I and
equal to Q. II follow.
P $ Q means P is neither smaller than nor 2. (D) In the given I Statement, the availability
greater than Q. of vegetables is not mentioned. So, I does not
P Q means P is not smaller than Q. follow. For the conclusion II, it is not directly
In each question, three statements showing related to the statement and so it also does not
relationships have been given, which are followed follow.
by two conclusions I and II. Assuming that the 3. (E) In the given statement, it has been
given statements are true, find out which mentioned that the world for a man is as he
conclusions(s) is/are definitely true. make it himself. So, we can say that some
Now, mark your answer as people might find it good, and some quite
(A) If only conclusion I is true bad. Hence, both I & II follow.
(B) If only conclusion II is true 4. (B) The investment consultants are well
acquinted with the market risks. Hence, it has
(C) If either conclusion I or II is true
been suggested that one should consult them
(D) If neither I nor II is true before investing ones money in securities.
(E) If both conclusions I and II are true. 5. (B) It is obvious from the given statement that
16. StatementsJ # P, P @ B, B % H. people desire personal liberty and right to
Conclusions dissent.
I. J @ B 6. (A) It is not advisable for avoiding
II. J % H consumption of sugar is not the right
18 | Logical Reasoning & Analytical Ability

approach to avoid diabetes. So, conclusion II 15. (B) The cause of revolution cannot be
does not follow. However in the context of deduced from the statement. It, therefore I
the statement conclusion I follows as does not follow but the statement describes
excessive sugar is the most likely cause of that Industrial Revolution brought about
diabetes. modern age. It means that revolution
7. (A) It is clear that conclusion I follows overhauls society. Hence, conclusion II
directly from the statement. But, II is not follow.
related to the given statement and so it does 16. (D) J # P J > P, P @ B P < B,
not follow. B%HB<H
8. (E) Sunita has a very busy schedule. It J>P<B<H
denotes that she is an industrious but still she I. J @ B J < B (False)
spares time for rest. This shows that she is an II. J % H J < H (False)
organised person. Hence, both I and II follow. 17. (A) W @ V W < V, V # X
9. (B) The given statement talks of jade plants V > X, Y V Y > V
only and not all plants with thick leaves. It, W < V > X and V < Y
therefore I does not follow ? Also, since Jade
plants require little water, so they can be I. X % Y X < Y (True)
grown in the area where water is not in II. X $ W X = Y (False)
abundance. Hence, II follows. 18. (B) M $ K M = K,
10. (B) There is nothing mentioned about the K F K > F,
relation between political freedom and F%H F< H
democracy in the statement. Hence, I does not M=K>,F<H
follow. In case of conclusion II, it is directly I. M $ F M > F (False)
followed from the given statement. II. M # F M = K (True)
11. (E) With the given statement, previous 19. (E) T @ L T < L, L % N L < N, D N
experience is an essential condition for D > N, T < L < N < D.
candidates for outstanding candidates, this I. N # T N > T (True)
condition can be waived. It shows that some II. L % D L < D (True)
candidates will have previous experience 20. (D) N % R N < R, R @ K R < K, K # F
while some will not.
K>F
12. (E) Clearly, the information has been given N<R<K >F
before hand in order that the residents of that
I. R @ F R < F (False)
area can collect and store water on the
previous day and use less water on Friday. II. R # F R > F (False)
Thus, both I and II follow. 21. (B) H D H > D, D # R D
13. (E) It is clear from both the statements that > R, R @ L R < L
the government has taken the step to attract H>D>R<L
more tourists. Hence, both I and II follow. I. L @ H L < H (False)
14. (E) The new system provides the students the II. H # R H > R (True)
choice of dropping Science and Mathematics 22. (D) E H E > H,
so weak students of these subjects can also be K $ H K = H,
admitted. Hence, I follows. Also, it is K@MK<M
mentioned that the new system will come in
E>H=K<M
force from the forthcoming academic year. It
shows that this system was not prevalent I. E # K E > K (False)
earlier. Hence, II also follows. II. E $ M E = M (False)


4 Syllogism
The word Syllogism is originally a word the subject while maintaining the validity of the
given by the Greeks. The term Syllogism is used proposition A universal affirmative proposition is
to denote that form of reasoning where conclusion usually denoted by the letter A.
is drawn from two or more statements. Actually it ExampleAll boys are students.
is an inference or deduction of the given
statements. This is undoubtedly the most Students
important part of logical reasoning. Syllogism is an
indispen-sable feature of all competitive examina- Boys
tions and tests which determines the candidates
basic intelligence and aptitude. The questions on
logic are to be solved as per the information given
without any concern of the formal validity or truth From here, we conclude that in A type
of the statements i.e., conclusion should follow proposition, only subject is distributed and we
directly from the statement given. cannot say All students are boys.
In solving the questions of syllogism, some (2) Universal Negative PropositionThis
are of the opinion that Venn-diagram can be a type of proposition distributes both the subject and
great use and, no doubt, a few questions can be the predicate. It means an entire class of predicate
solved with the help of Venn-diagrams, but Venn- term is denied to the entire class of the subject
diagram alone does not help the students to solve term.
the various questions on syllogism. A universal negative proposition is usually
The problem of syllogism can be solved by denoted by the letter E.
using a little intelligence and common sense but ExampleNo man is intelligent.
we need to have, therefore, a definite and well-
defined method to tackle the problem.
PropositionAny statement is termed as the Man Intelligent
proposition. The proposition is a sentence that
makes a statement and gives a relation between
two terms. It comprises a subject, a predicate and We can conclude than Man and Intelligent
a copula. Subject is that which affirms or denies a have nothing in common and hence both subject
fact. The predicate is the part of the proposition and predicate are distributed.
denoting that which is affirmed or denied about (3) Particular Affirmative Proposition
the subject and copula is the part of the proposition This type of proposition distributes neither the
which establishes the relation between the subject subject nor the predicate. Particular affirmative
and the predicate. proposition is denoted by the letter I.
Propositions can be classified into four ExampleSome girls are students.
types
(1) Universal Affirmative Proposition
Girls Students
This type of proposition distributes only the
subject. The predicate is not interchangeable with
20 | Logical Reasoning & Analytical Ability

In this type of proposition, subject and a clear understanding of difference between


predicate have something in common. Particular Immediate and Mediate inference.
proposition either only partly include or only Immedicate inference : In immediate inference,
partly exclude the subject while making a conclusion is drawn from only one given
statement. proposition for example, let a given statement be
(4) Particular Negative PropositionThis All men are wise. Then based on this statement a
type of proposition distributes only in the conclusion could be drawn that some men are
predicate. It is denoted by the letter O. wise.
ExampleSome goats are not men. In case of two statements, connected by a
common term which is popularly known as
Goats Men Middle Term.
Statements I. Some girls are teacher.
Here, the subject term some goats is only II. No teacher is intelligent.
for a part of its class and hence it is undistributed In the above pair of statements, the term
while the predicate term men is denied in teacher is common both the statements and joins
entirely to the subject term and hence is the two statements. Now, if a conclusion is drawn
distributed. Syllogism is concerned with three from any one of the two statements without taking
terms. consideration the other statement is known as
(1) Major TermIt is the predicate of the Immediate inference.
conclusion and it is denoted by P.
ConclusionsSome Teacher are girls (From
(2) Minor TermIt is the subject of the Statement I)
conclusion and it is denoted by S.
No intelligent is teacher (From Statement II)
(3) Middle TermIt is the term common to
both the statements and it is denoted by M. Rules for immediate inferenceThere are
The Venn diagram. many aspects or methods of immediate inference
There is a pictorial way of representing the and can be drawn with the help of some rules.
proposition. Suppose that the proposition is trying These rules are divided into four heads.
to relate the subject (S) with predicate (P). There (A) Conversion
are four ways in which the relation could be (B) Obversion
according to the four propositions. (C) Contraposition
TypeA TypeE (D) Sub-alteration
P (A) ConversionIn conversion, the subject
S S P and the predicate of a given statement are inter-
changed i.e., the subject becomes the predicate
All S are P No S is P and the predicate becomes the subject but the
TypeI quality of the proposition remains intact i.e., the
affirmative statement remains affirmative and the
P
S P
negative remains negative.
S
Or Thus, we can say that A-type proposition can
be changed into I-type. E-type can be converted
Some S are P
into E-type. I-type can be converted into I-type
TypeO but O-type proposition can not be converted.
S P Examples(i) StatementAll dogs are
cats. (A-Type)
Or
ConclusionSome cats are dogs. (E-type)
P S S P (ii) StatementNo girl is beautiful. (E-type)
Or
Some S are not P ConclusionNo beautiful is girl. (E-type)
Syllogism is actually a problem of mediate (iii) StatementSome bats are crows.
inference but before we proceed to the rules for (E-type)
arriving at valid inferences, it is necessary to have ConclusionSome crows are bats. (E-type)
Logical Reasoning & Analytical Ability | 21

(B) ObversionIn obversion, the conclusion The inference can also be drawn from logic
is drawn from interchanging the quality of using Venn-diagram. But the important point is to
proposition i.e., affirmative to negative and be noted while drawing such inferences from the
negative to affirmative but its original meaning Venn diagrams that all possibilities of Venn-
remains the same. The subject of the given diagrams should be drawn.
statement remains the subject of the conclusion. (1) Universal Affirmative (A-type)
Example(i) StatementNo cow is pet. All S are P
(E-type) In A type of proposition, all S are contained
ConclusionAll cow are non-pet (A-type) in B. The circle representating S will be either
(C) ContrapositionIn contraposition, there inside or equal to circle representating B. Thus, in
will be double conversion. First to obverse and both the cases conclusions (some P are S) and
then to converse. (some S are P) are true. We can understand by
taking two sets in all possible ways.
Example(i) StatementNo women a r e
wise. (E-type) P
S SP
ConclusionAll women are non-wise. 1
1 4
(A-type, obversion) 2 2
5 3
ConclusionSome non-wise are women. 3
(I-type, conversion)
(D) Sub-alterationA proposition can be (i) S = (1, 2, 3) P = (1, 2, 3, 4, 5)
changed into sub-alternation by retaining the same (ii) S = (1, 2, 3) P = (1, 2, 3)
subject, same predicate and same quality but with The above two cases show all the possibilities
changed quantity of proposition. Thus, we can say of two different sets S and P which show the
that A-type will be changed into I-type and E-type relationship between each other as represented by
will be changed to O-type. the propositionAll S are P.
Examples(i) StatementAll goats are In both the cases we observe that set (1, 2, 3)
elephants. (A-type) is the part of Set A and Set B. Now, we can say
ConclusionSome goats are elephants. that inference (some S are P) is true from this
(I-type) relationship. In the same way set (1, 2, 3) is the
(ii) StatementNo ball is black. (E-type) part of set B and also of set A. Therefore, it is also
clear that inference (some P are S) is also true.
ConclusionSome ball are non-black.
(O-type) (2) Universal Negative (E-type)
We display in the below-mentioned table all No S is P
the possible valid immediate inference drawn In E-type of proposition, there is only one
from each type of proposition i.e., A. E. I. O. A possibility of Venn-diagram. This relationship can
study of the table given below can help the be shown by two sets S = (1, 2, 3) & P (4, 5, 6).
students to remember all the valid immediate With the help of these two sets we see that 1, 2,
inferences easily without going into details of and 3 is the part of set S but not of set P. Similarly
above rules. 4, 5, 6 is the part of set P but not set of S.

Immediate 1, 2, 3 4, 5, 6
Type Proposition
Inference
A All S are P Some P are S S P
Some S are P Therefore, on the basis of E-type of
No P is A proposition we can draw following immediate
E No S is P Some S are not P inferences
Some P are not A (i) No P is S
I Some S are P Some P are S (ii) Some S are not P
O Some S are not P No inference (iii) Some P are not S
22 | Logical Reasoning & Analytical Ability

(3) Particular Affirmative (I-type) Set (2, 3) is the part of Set S but not set P,
Some S are P hence shows the relation representated by the
proposition Some S are not P.
In this proposition, some part of the circle
denoting S as indicated by the shaded area of S S
(representing same S lies within the circle 1 P
denoting P). Slightly more attention-seeking is the 4 2
representation for the O proposition some S are 5
not P. This proposition gives rise to many 3
possible representations of Venn-diagrams and (iii)
hence most of the inferences drawn therefrom are
invalid and doubtful. The verbal interpretation of (iii) S = (1, 2, 3, 4)
this figure would be there are some S that are P = (4, 5)
definitely not P while there may some S that Set (1, 2, 3) is the part of set S but not set P,
might be P or might not be P hence denotes proposition some S are not P.
S P
We can say on behalf of all possible combi-
nations showing relationship between S and P no
valid inference can be drawn. Some S are P is true
Some S are not P
from case (i) and (iii) but not true from case (ii)
[Some S are P] hence it is an invalid inference. Similarly, some P
S P are not S is true from case (i) and (ii) but not true
from case (iii) and hence it is also an invalid
Some S are not P inference.
[No S are P] The Hidden Proposition
(4) Particular Negative (OType) There are some other sentences which are not
Some S are Not P on the standard patterns. Therefore, we have to
find out the hidden proposition in such sentences.
This proposition give no clue whether the
remaining S are there in P or not and from this 1. Some A type propositions which do not begin
proposition no immediate inference can be drawn. with All.
We can deduce this proposition in the light of (A) All positive sentences which begin with
Venn-diagram representation. There may be three every, each any are A-type
possibilities. propositions.
1 3 5 Examples
2 4 6 (a) Every man loves his country.
(All men love their country)
S P
(i) (b) Each of them was present there.
(i) S = 1, 2, 3, 4 (All were present there)
B = 3, 4, 5, 6 (c) Any one can do this work.
Set (1, 2) is the part of Set S but not Set B, (All person can do this work)
hence this shows the relationship representated by (B) A positive sentence with a particular
the proposition Some S are not P. person as its subject is always an A-type
S P
proposition.
1 4 Examples
2 5 (a) He should be awarded a handsome prize.
3 6
(He who should be awarded a handsome
(ii)
prize.)
(ii) S = (1, 2, 3) (b) General Suharto is a controversial
P = (4, 5, 6) personality.
Logical Reasoning & Analytical Ability | 23

(C) A positive sentence with a very definite Examples


exception is also a A-type. (a) Men are usually masculine.
Examples (Some men are masculine)
(a) All girls except Reena have failed. (b) Students are generally naughty.
(b) All except Reena (are the students) who (Some students are naughty)
have failed. (B) Negative proposition which begin with
words such as often, seldom, hardly,
2. Some E-type propositions not beginning with
scarcely rarely, little etc, are to be
No.
deduced to the I-type.
(A) All negative sentences beginning with Examples
none no one not a single etc are
(a) Few men are not honest.
called E-type propositions.
(Some men are honest)
Examples (b) Rarely is a rich man content.
(a) None can escape from death. (Some rich are content)
(No person is one who can escape from 4. Some O type preposition not beginning with
death). somenot.
(b) Not a single person was present. (A) All negative proposition beginning with
(No person was present) words such as all, every, any, each etc.
(B) A sentence with a particular person as its with not are to be reduced to O type
subject but a negative sense is an E-type propositions.
proposition. Examples
Examples (a) All women are not rich.
(a) He does not deserve for a prize. (Some women are not rich)
He (is not a man) who deserves for a (b) Everyone is not there.
prize. (Some are not there)
(b) General Suharto is not a controversial (c) All that glitters is not gold.
personality. (Some glittering objects are not gold)
(C) A negative sentence with a very definite (B) Negative propositions with words as
exception is also of E-type. most a few mostly generally almost
Examples frequently with not are to be reduced to
the O type.
No man except David has failed.
Examples
(D) When an interrogative sentence is used
to make an assertion, this could be (a) Girls are usually not feminine.
reduced to an E-type proposition. (Some girls are not feminine)
(b) Students are not frequently ill-tempered.
Examples
(Some students are not ill-tempered)
(a) Is there any honesty left in this world ?
(c) Act all the furniture have not been sold.
(No honesty is left in this world.)
(Some furniture have not been sold)
(b) Is there any person who can deceive
himself ? (C) Positive proposition with beginning
(None can deceive himself). negative sense words such as few,
seldom, hardly, scarcely, rarely,
3. Some I-type propositions which do not begin little etc are to reduced to the O type.
with some.
Examples
(A) Positive propositions which begin with
words such as most, a few, mostly, (a) Few men are innocent.
generally, almost, frequently often (Some men are not innocent)
are to be reduced to the I-type (b) Seldom are people jealous.
proposition. (Some people are not jealous)
24 | Logical Reasoning & Analytical Ability

(D) A negative sentence with an exception, Conclusion I is an A-type proposition which


which is not are definite are to be distribute the subject cows only.
reduced to O type. Since, the term cows is distributed in
Examples conclusion I without being distributed in the
premises so conclusion I cannot follow.
(a) No except one have passed.
Rule 3The middle term must be distributed
(Some students have not passed)
atleast once in the premises, otherwise conclusion
(b) No girls except a few are absent. is uncertain or doubtful.
(Some girls are not absent)
Example
Rules for Mediate Inference I. All stool are chairs.
There are certain rules to derive the valid II. Some chairs are benches.
conclusion. These rules should be applied step by Conclusions
step to solve the questions on syllogism. To I. All stools are benches.
understand well these rules we can take help of
Venn-diagram representation. II. Some benches are stools.
III. No stool is a table.
Rule 1The conclusion does not contain the
middle term. SolutionThe term chair is common to both
Example the statements and hence it is the middle term.
Statement is (I) is of A-type proposition and in
Statements
A-type proposition only subject is distributed,
I. All boys are men. hence chair being the predicate in the statement (I)
II. Some men are students. is not distributed. In the same way, chair is not
Conclusions distributed in the second statement because
I. All men are boys. statement (II) is I-type in wich neither subject nor
II. Some students are men. predicate is distributed. We can analyse that
middle term chair is not distributed in anyone of
Since, both the conclusions 1 and 2 contain the statements, therefore as per rule 3, no mediate
the middle term men so neither of them can conclusion can be drawn. Thus, none of the
follow. conclusions of following statements above is a
Men Men Men valid conclusion.
Boys Boys We can also explain this rule with the help of
Boys Venn-diagram.
Chairs Chairs
Students
Students
Stools Benches Stools Benches

Rule 2No term can be distributed in the


conclusion unless it is distributed in the premises.
Example (A) (B)
Statements Chairs Chairs
I. Some men are women.
II. All women are cows. Stools Benches Stools Benches
Conclusions
I. All cows are women.
II. Some men are cows.
Statement I is an I-type proposition which (C) (D)
distributes neither the subject nor the predicate. All the four figures (A), (B), (C) and (D)
Statement II is an A-type proposition which represent the relationship among stool, chairs and
distributes the subject women only. benches as given in the statements. We can find
Logical Reasoning & Analytical Ability | 25

from these four figures that we can not establish


any definite relationship between stools and Books Pencils
benches.
Though it can be concluded from above Books Pencils Erasers
Erasers
analysis that no valid conclusion can be drawn
between stool and bench yet we can conclude that
(A) (B)
either some stools are benches or no stool is a
table because both the conclusion pair a With the help of both the figures, we cannot
complementary pair. deduce any inference.
Rule 4If one premises is particular, Rule 6If both the premises are negative, no
conclusion is particular. conclusion follows.
Example Example
Statements Statements
I. Some men are intelligent. I. No flower is apple.
II. All intelligent are wise. II. No apple is banana.
Conclusions Conclusions
I. All men are wise. I. No flower is banana.
II. No man is wise. II. Some bananas are apples.
III. Some men are wise. SolutionSince, both the statements are
negative, we cannot draw any valid mediate
SolutionAccording to this rule, statement I
inference from such pair of given statements.
is particular type, therefore, Conclusion (III) is the
Therefore, no conclusion follows.
valid inference. Simultaneously, it does not violate
rules (2) and (3). Venn diagram representation.
We can also define this rule with the help of
Venn-diagram. Flower Apple
Flower Apple
Wise
Wise
Banana Banana
Men Intelligent
Men Intelligent
(A) (B)
The study of both the figures (A) and (B)
reveals the fact that in case both the premises are
(A) (B) negative, no definite relationship between the
terms other that the middle term can be
In both the figures (A) and (B) it is clear that
established. Hence, conclusion I is not valid
some men are wise.
because it is not from figure (A). In the same way
Rule 5If both the premises are particular, conclusion II is also not valid because it is not true
no conclusion follows. from figure (B). Therefore, none of the
Example conclusions is definitely true.
Statements Rule 7If one of the premises is negative,
I. Some books are pencils. conclusion will always be negative.
II. Some pencils are erasers. Example
Conclusions Statements
I. All books are erasers.
I. Some cups are bottles.
II. Some erasers are books.
II. No bottle is glass.
Since, both the premises are particular, no
conclusion follows. Conclusions
By Venn-diagram, we can check the I. Some cups are glasses.
authencity of the rule. II. Some cups are not glasses.
26 | Logical Reasoning & Analytical Ability

III. All glasses are cups. Mothers


IV. No cup is glass.
Married
As mentioned in above rule (7) if one of the
premises is negative, the conclusion will always Women
be negative. From the given conclusions, we can
observe that conclusions (II) and (IV) are
negative. Therefore, these may be valid
inferences. But for the validity of this rule they
must not violate any of the other rules. In above-mentioned diagram it is clear that all
women are mothers.
Conclusion (IV) Violate rule because cup is
Case of complimentary pair of conclu-
distributed in conclusion but it is not distributed in
sionsIn drawing mediate inferences from
the statement I. This conclusion also violate rules
given statements, it is expected to be more
which stipulates if one of the premises is
attentive to select a complimentary pair of
particular, conclusion will be particular but
conclusions in which neither of the conclusions is
conclusion (IV) is universal. Conclusion II
individually true but a combination of both makes
complies with the rule as term glass which is
a complementary pair. In a complimentary pair,
distributed in the conclusion is also distributed in
atleast one of the two statements is always true.
the statement II and also conclusion (II) being of
As we have already discussed in a rule that in the
particular quantity.
statements where middle term is not distributed no
Venn-diagram. valid inference but there still exists a possibility
that a complimentary pair of conclusions follows
Cups Bottle from the given statement.
Cups Bottle Example
Statements
Glass Glass
I. Some cups are books.
(A) (B) II. Some books are white.
Conclusions
From the above-mentioned graphical Venn-
diagram representation, we can observe that no I. Some cups are white.
valid representation between cups and glass can II. No cup is white.
be established. But we can definitely conclude that From the above example, it is clear that
part of the cups which is bottle can never be glass neither of the conclusions is definitely true as per
because no cup is glass. Therefore, from Venn- rule because middle term is not distributed in the
diagram also, we draw valid inference that some statements. This situation can become more clear
cups are not glass. with the help of Venn-diagram.
Rule 8If both the premises are affirmative,
the conclusions would be affirmative.
Cups Books
Example
Statements Cups Books White White
I. All women are married.
II. All married are mothers. (A) (B)
Conclusions From the these two figures, it is clear that
I. All women are mothers. conclusion (I) Some cups are white is wrong
because it not true from figure (A). In the same
II. Some women are not mothers. way conclusion (II) No cup is white is also wrong
Here, both the premises are affirmative, the because it is not true from figure (B). However, it
conclusion will be affirmative. Venn-diagram is to be noted from the figures that either
representation : conclusion (I) or (II) follows from the statement.
Logical Reasoning & Analytical Ability | 27

Exercise 15. Statements


Type I Some books are dictionaries.
Some dictionaries are files.
DirectionsIn each question below are Some files are papers.
given two statements followed by two Conclusions
Conclusions
numbered I and II. You have to take the given
two statements to be true even if they seem to be I. Some papers are files.
variance from commonly known facts. Read these II. Some files are books.
two conclusions and then decide which of the 16. Statements
given conclusions logically and analytically Some stones are rocks.
follows from the given two statements desregar- All rocks are clouds.
ding commonly known facts. All clouds are rains.
Give the answer Conclusions
(A) if only conclusion I follows. I. Some rains are stones.
(B) if only conclusion II follows. II. Some clouds are rocks.
(C) if either conclusion I or II follows.
17. Statements
(D) if neither conclusion I nor II follows.
Some keys are holes.
(E) if both conclusion I and II follow.
All holes are folders.
11. Statements Some folders are stands.
Some pins are forks. Conclusions
All forks are keys.
I. Some stands are keys.
No key is lock.
II. Some stands are holes.
Conclusions
18. Statements
I. Some locks are pins.
II. No lock is a pin. Some desks are tents.
Some tents are rivers.
12. Statements
All rivers are ponds.
Some shirts are trousers.
Conclusions
Some trousers are jackets.
I. Some ponds are tents.
All jackets are shawls.
II. Some ponds are desks.
Conclusions
I. Some shawls are shirts. 19. Statements
II. Some jackets are shirts. All chairs are pens.
Some pens are knives.
13. Statements
All knives are rats.
Some leaves are plants.
Conclusions
Some plants are trees.
I. Some rats are chairs.
Some trees are fruits.
II. Some rats are pens.
Conclusions
I. Some fruits are trees. 10. Statements
II. Some trees are plants. Some forests are huts.
14. Statements Some huts are walls.
Some rats are dogs. Some walls are nets.
Some dogs are horses. Conclusions
Some horses are camels. I. Some nets are forests.
Conclusions II. Some nets are huts.
I. Some horses are rats. 11. Statements
II. Some camels are horses. All tables are windows.
28 | Logical Reasoning & Analytical Ability

All windows are rooms. Conclusions


All rooms are buses. I. Some pencils are books.
Conclusions II. Some pictures are books.
I. Some buses are tables. 18. Statements
II. Some rooms are tables. All tables are pens.
12. Statements All pens are papers.
Some trees are boxes. All papers are tablets.
All boxes are bricks. Conclusions
I. All tables are papers.
All bricks are dogs.
II. Some tablets are pens.
Conclusions
I. Some dogs are trees. 19. Statements
II. Some bricks are trees. All buildings are houses.
Some houses are towers.
13. Statements
All towers are huts.
All goats are flowers.
Conclusions
No flower is branch.
I. Some huts are houses.
Some branches are roots.
II. Some huts are buildings.
Conclusions
I. Some roots are goats. 20. Statements
II. No root is goat. Some leaves are branches.
All branches are flowers.
14. Statements
Some flowers are fruits.
All pots are rings.
Conclusions
All bangles are rings.
I. Some fruits are leaves.
All rings are paints.
II. Some fruits are branches.
Conclusions
I. Some paints are pots. 21. Statements
II. Some bangles are paints. Some novels are desks.
15. Statements No desk is ladder.
All cars are wheels. Some ladders are chimneys.
No wheel is chair. Conclusions
Some chairs are spokes. I. Some chimneys are novels.
II. No novel is chimney.
Conclusions
I. Some spokes are cars. 22. Statements
II. Some spokes are wheels. All beads are rings.
All bangles are rings.
16. Statements
All rings are diamonds.
Some tapes are trunks.
Conclusions
Some trunks are halls.
I. All beads are diamonds.
All halls are desks.
II. All bangles are diamond.
Conclusions
23. Statements
I. Some desks are tapes. Some cards are plastics.
II. Some desks are trunks. Some plastics are metals.
17. Statements All metals are pots.
Some books are chairs. Conclusions
Some chairs are pictures. I. Some pots are cards.
Some pictures are pencils. II. No pot is a card.
Logical Reasoning & Analytical Ability | 29

24. Statements All clouds are rains.


All chairs are tables. All rains are stones.
All tables are trains. Conclusions
All trains are buses. I. All rains are stars.
Conclusions II. All clouds are stones.
I. All tables are buses.
II. All trains are tables. 31. Statements
All windows are doors.
25. Statements
Some doors are building.
Some machines are computers.
Some computers are calculators. All buildings are cages.
Some calculators are phones. Conclusions
Conclusions I. Some cages are doors.
I. Some phones are computers. II. Some buildings are windows.
II. Some computers are machines. 32. Statements
26. Statements Some chairs are rooms.
All spoons are bowls. All rooms are trees.
Some bowls are glasses. All trees are poles.
Some glasses are plates. Conclusions
Conclusions I. Some poles are chairs.
I. Some glasses are spoons. II. Some trees are chairs.
II. Some plates are bowls.
Answers with Explanation
27. Statements
1. (C) Keys
Some envelopes are packets.
Some packets are boxes. Forks
All boxes are parcels. Pins Lock
Conclusions
I. Some parcels are packets. Or
II. Some parcels are boxes.
Keys
28. Statements
Some toys are desks. Lock Pins Forks
Some desks are pens.
All pens are rods.
Conclusions 2. (D)
Shawls
I. Some rods are toys.
II. Some pens are toys. Shirts Trousers Jackets
29. Statements
Some tables are huts.
No hut is ring. Or
All rings are bangles.
Conclusions Shirts Trousers
I. Some bangles are tables.
II. No bangle is table.
Jackets
30. Statements
Shawls
All stars are clouds.
30 | Logical Reasoning & Analytical Ability

3. (E) Or
Leaves Plants Trees Fruits Folders

Holes
Or Keys

Leaves Plants
Stands
Or
Fruits Trees Folders

Keys Holes Stands


4. (B)
Rats Dogs Horses Camels
8. (A) Ponds
Or
Desks Tents Rivers
Rats Dogs

Or
es

Ca
me
rs

Desks Tents
Ho

ls

5. (A)
Rivers
Books Dictionaries Files Papers Ponds

9. (B) Pens Rats


Or Chairs Knives

Dictionaries
Books
Or
Pens Rats

Papers Files Chairs Knives

6. (E) Rains
Clouds 10. (D)
Rocks Forests Huts Walls Windows

St
on
Or
es
Windows
7. (D) Folders
Holes Forests Huts

Keys Stands
Walls
Logical Reasoning & Analytical Ability | 31

11. (E) Buses Or


Rooms Paints
Windows
Rings
Tables
Pots Bangles

12. (E) Dogs 15. (D) Wheels


Bricks Spokes
Cars
Trees Boxes Chairs

Or
Wheels
13. (C) Flowers
Cars Chairs
Goats Branches Roots
Spokes

Or None follows.

Branches 16. (B) Desks


Flowers
Tapes Trunks Halls
Goats
Roots

Or
14. (A) Paints Desks
Rings Tapes Trunks
Pots
Halls
Bangles
Only II follows.
17. (D)
Books Chairs Pictures Pencils

Or
Or
Paints
Books Chairs
Rings

Pots Bangles Pe
res

nc
ctu

ils
Pi

None follows.
32 | Logical Reasoning & Analytical Ability

18. (E) Teblets Or


Papers Novels
Pens
Ladders Desks
Tables
Chimneys

Either I or II follows.
22. (E) Diamonds
Both follows.
Rings
19. (A) Houses Huts Beads Bangles

Buildings Towers

Or
Or Diamonds
Rings
Houses Huts
Beads Bangles
Buildings Towers

Only I follows. Both follow.


23. (C) Pots
20. (D) Flowers
Cards Plastics Metals
Leaves Branches Fruits

Or
Pl
Cards as
Or tic
s

Fruits Metals
Fl

Pots
ow
er
s

Leaves Branches 24. (A) Buses


Trains
Tables
None follows. Chairs
21. (C)
Novels Desks Ladders Chimneys
Logical Reasoning & Analytical Ability | 33

25. (B) Or

Machines Computers Calculators Phones Toys Desks

Pens
Rods
Or
29. (C) Bangles
s Computers
ine Tables Huts Rings
ch
Ma

Ph rs
o ne ul ato Or
s Calc
Tables
26. (D) Bowls Huts
Rings
Spoons Glasses Plates
Bangles

30. (B) Stones


Or Rains
Bowls Clouds

Spoons Stars

Glasses
Plates
31. (A) Doors Cages
27. (E) Parcels
Windows Buildings
Envelopes Packets Boxes

Or
Or
Doors Cages
es

Buildings

Packets
lop

Windows
ve
En

Boxes
32. (E) Poles
Parcels
Trees
28. (D)
Chairs Rooms
Toys Desks Pens Rods
34 | Logical Reasoning & Analytical Ability

Type II (A) None follows


DirectionsIn each of the questions below (B) Only I & II follow
are given four statements followed by three (C) Only II & III follow
conclusions numbered I, II and III and some- (D) Only I & III follow
where IV. You have to take the given statements (E) None of these
to be true even if they seem to be at variance from
commonly known facts. Read all the conclusions 14. Statements
and then decide which of the given conclusions Some fans are coolers.
logically follows from the given statements Some coolers are machines.
disregarding commonly known facts. Some machines are computers.
11. Statements All computers are televisions.
Some leaves are flowers.
Conclusions
No flower is fruit.
I. Some televisions are machines.
Some fruits are branches.
II. Some machines are fans.
Some branches are stems.
III. No machine is fan.
Conclusions
I. Some leaves are stems. (A) None follows
II. All leaves are either stems or fruits. (B) Only I follows
III. All stems are either branches or fruits. (C) Only either II or III follows
(A) Only I follows (D) Only I and either II or III follow
(B) Only II & III follow (E) All follow
(C) Only III follows 15. Statements
(D) All follow All keys are staplers.
(E) None follows All staplers are blades.
12. Statements Some blades are erasers.
All lions are tigers. Some erasers are sharpners.
All tigers are leopards. Conclusions
Some leopards are wolves. I. Some sharpners are keys.
Conclusions II. All keys are blades.
I. No elephant is lion. III. Some erasers are keys.
II. Some wolves are lions. (A) Only I & II follow
III. Some leopards are lions. (B) Only I follows
(A) Only I follows (C) Only II follows
(B) Only II follows (D) All follow
(C) Only III follows (E) None of these
(D) Only I & III follow 16. Statements
(E) All follow All flowers are fruits.
13. Statements No fruit is juice.
Some caps are umbrellas. Some juices are proteins.
Some umbrellas are raincoats. All proteins are vitamins.
All raincoats are trousers. Conclusions
All trousers are jackets. I. Some vitamins are fruits.
Conclusions II. Some juices are vitamins.
I. Some raincoats are caps. III. No flower is juice.
II. Some trousers are umbrellas. (A) Only I and II follow
III. All raincoats are jackets. (B) Only I and III follow
Logical Reasoning & Analytical Ability | 35

(C) All I, II and III follow 10. Statements


(D) Only II and III follow Some towers are pillars.
(E) None of these Some pillars are buildings.
All buildings are flats.
17. Statements
No flat is house.
Some bags are suits.
Conclusions
All suits are trousers.
Some trousers are shirts. I. No building is house.
All shirts are coats. II. Some towers are houses.
Conclusions III. Some towers are flats.
I. Some trousers are coats. (A) None follows
II. Some bags are trousers. (B) Only I follows
III. Some suits are coats. (C) Only II and III follow
(A) Only I and II follow (D) All follow
(B) Only I follows (E) None of these
(C) Only II and III follow
Answers with Explanation
(D) All follow
(E) None of these 1. (E)

18. Statements
All books are novels. Flowers
Leaves
Some novels are poems.
Some poems are stories.
No story is a movie. Fruits
Stems
Conclusions
I. Some books are poems. Branches
II. Some movies are novels.
III. No movie is a novel. Or
(A) Only I follows
(B) Only I and II follow Leaves
Stems Flowers
(C) Only either II or III follows
(D) None follows
(E) None of these
Br

Fruits
an

19. Statements
ch
es

All cups are bowls.


All bowls are trays. None follows.
Some trays are plates. 2. (C)
No plate is spoon. Leopards
Conclusions
Tigers
I. Some bowls are plates.
II. Some cups are spoons. Lions
III. No cup is spoon.
(A) Only I follows
(B) Only II follows
(C) Only II and III follow Wolves
(D) All follow Elephant
(E) None of these
36 | Logical Reasoning & Analytical Ability

Or Blades

Leopards Staplers

Tigers Keys
Erasers
Lions

W nts Sharpners
a
ol
ve eph
s El Only II follows.
6. (E)
Only III follows.
Flowers
3. (C) Leaves

Jackets
Fruits
Stems
Trousers
Branches
Raincoats
Caps Or

Um Stems Leaves
Flowers
br
ell
as
Br

Fruits
an

Only II and III follow.


ch
es

4. (D) None follows.


Televisions 7. (A)
Coats
Bags

Suits Shirts
Computers
Fans Coolers Machines user
Tro
Only I and II follows.
Only I and either II or III follow. 8. (C)

5. (C) Novels
M
ov
ie
Blades Books
Novels

Staplers Books
Erasers s
Or em
Keys Po
Poems
Stories
Sharpners Stories Movie

Or Only Either II and III follows.


Logical Reasoning & Analytical Ability | 37

9. (C) 10. (E) Flats

Bldgs.

Towers
Cups Pillars

Bowls
Trays House

Plates Spoons
Or
Or
ns

Flats
oo
Sp

Bldgs.
House

Towers
Plates

Cups Pillars

Bowls
Trays

Only II and III follow.


5 Statement and Argument
In such type of test, a statement is given which Example 3.
is followed by two arguments Candidates are StatementShould prohibition on alcohol
required to distinguish between these two be banned ?
arguments and find out which one is strong ? ArgumentYes, Prohibition on alcohol
A argument may be strong in the following should be banned.
conditions ExplanationThis argument is merely
(1) A strong argument should give the repetition of the statement. Hence, this is not a
realistic diagnosis of the situation strong argument.
described in the statement. Example 4.
(2) A strong argument should relate with the StatementShould luxary hotels be banned
statement and be supported up by the in India ?
facts or established notions. ArgumentYes, these are the places from
(3) A strong argument should be guazed by where international criminals operate.
the previous experiences and it can be ExplanationThe luxary hotels are symbol
predicted that the result will follow. of countrys standard and places for staying the
(4) A strong argument should not be mere affluent foreign tourists and it is not confirmed
reiteration of the situation given in the that the criminals always stay there. Hence, this is
statement. not a strong argument.
No argument should be based on emotion, Example 5.
feeling etc., because a weak argument is StatementShould State Government ban
very simple, superflous, ambiguous and on Lottery ?
long drawn one. ArgumentYes, to spend money aimlessly
is not a proper thing.
Example 1.
ExplanationIt is not clear that the money
StatementWill Suresh get second class in is being spent on lottery or on other things. Hence,
the examination ? it is not a strong argument.
ArgumentYes, he always stands second. Example 6.
ExplanationHe always stands second, it StatementShould we encourage compu-
does not mean that he will come second this year terization ?
also. Hence, we can say that this argument is not ArgumentYes, America is also doing this.
strong.
ExplanationAny argument cannot be taken
Example 2. with the context of a person, incident of a country
StatementAre human cultures developing or some examples. We cannot compare certain
into savage and unsound way ? qualities with the other country. Hence, it is not a
ArgumentYes, man is an uncivilized strong argument.
animal. Example 7.
ExplanationWe know that man is a social StatementWill the political activities go
animal while in this argument, it has been stated on this year ?
against the social belief. Hence, this is a weak ArgumentYes, many of the newspapers
argument. endorse the statement.
Logical Reasoning & Analytical Ability | 39

ExplanationIf the newspaper write this, it Arguments


does not mean that the political activities will go I. No, computerization demands a lot of
on. Hence, it is not a strong argument. money. We should not waste money on
Exercise it.
Each of the following questions consists of a II. Yes, when advanced countries are
statement followed by two arguments I and II. introducing computers in various areas,
Give Answer how can we afford to lag behind.
(A) if only argument I is strong , 7. StatementShould personal tax be abolished
(B) if only argument II is strong, in India ?
(C) if either I or II is strong, Arguments
(D) if neither I nor II is strong and I. Yes, it will motivate people to earn more.
(E) if both I & II are strong. II. No, individual must learn to share their
1. StatementShould there be reservation of wealth with other people.
seats and posts on communal basis ? 8. StatementShould high chimneys be in-
stalled in industries ?
Arguments
Arguments
I. Yes, it will check most of the inter
I. Yes, it reduces pollution at ground level.
communal biases.
II. No, it increases pollution in upper
II. No, ours is a secular state.
atmosphere.
2. StatementGovernment stop spending huge 9. StatementShould judiciary be independent
amounts of money on international sports. of the executive ?
Arguments Arguments
I. Yes, this money can be utilised for I. Yes, this would help curb the unlawful
upliftment of the poor. activities of the executive.
II. No, sport persons will be frustrated and II. No, the executive would not be able to
will not get international exposure. take hold measures.
3. StatementShould there be a ban on product 10. StatementShould an organization like
advertising ? UNO be dissolved ?
Arguments Arguments
I. No, it is an age of advertising. Unless I. Yes, with cold war coming to an end,
your advertisement is better than your such organizations have no role to play.
other competitors, the product will not be II. No, in the absence of such organizations
sold. there may be a world war.
II. Yes, the money spent on advertising is 11. StatementShould there be a complete ban
very huge and it inflates the cost of the on strike by government employees in India ?
product. Arguments
4. StatementShould we scrap the Public I. Yes, this is the only way to teach
Distribution System in India ? discipline to the employees.
Arguments II. No, this deprives the citizens of their
I. Yes, protectivism is over everyone must democratic rights.
get the bread on his/her own. 12. StatementShould luxary hotels be banned
II. Yes, the poor do not get any benefit in India ?
because of corruption. Arguments
5. StatementShould non-vegetarian food be I. Yes, they are places from where inter-
totally banned in country ? national criminals operate.
Arguments II. No, Affluent foreign tourists will have no
I. Yes, it is expensive and therefore, beyond place to stay.
the means of most people in our country. 13. StatementShould internal assessment in
II. No, nothing should be banned in a college be abolished ?
democratic country. Arguments
6. StatementShould India go for computeriza- I. Yes, this will help in reducing the
tion in Industry ? possibility of favouritism.
40 | Logical Reasoning & Analytical Ability

II. No, teaching faculty will lose control II. Yes, it is a genuine demand of lakh of
over students. employees.
14. StatementShould we impart sex education 21. StatementShould there be a restriction on
in schools. number of one-day international cricket
Arguments match to be played by any country within a
I. Yes, all the progressive nations do so. calendar year ?
II. No, no we cannot impart it in co- Arguments
educational schools. I. No, each country should have the
15. StatementShould fashionable dresses be freedom to play as many matches as they
banned ? want.
Arguments II. Yes, otherwise the top ranking players
I. Yes, fashions keeps changing and hence will burn out in quick time.
consumption of cloth increases. 22. StatementShould there be only one type of
II. No, fashionable clothes are a persons schools upto matriculation in the entire
self-expression and therefore, his/her country ?
fundamental right. Arguments
16. StatementShould family planning be made I. Yes, this exists in some of the western
compulsory in India ? countries.
Arguments II. No, schools in rural and urban areas need
I. Yes, looking to the miserable conditions to be different.
in India, there is no other go. 23. StatementShould there be a complete ban
II. No, in India there are people of various on use of pesticides for maintaining fruit-
religions and family planning is against growing plants and trees ?
the tenets of some of the religions. Arguments
17. StatementShould there be a world govern- I. No, all these plants and trees will get
ment ? destroyed by the attacks of the pests
Arguments causing severe financial loss to the
I. Yes, it will help in eliminating tensions farmers.
among the nations. II. Yes, the hazardous chemicals used in the
II. No, then, only the developed countries pesticides find their way into the fruits
will dominate in the government. causing serious health hazard to all those
18. StatementShould we scrap the system of who consume these fruits.
formal education beyond graduation ? 24. StatementShould gambling be made
Arguments legally allowed in India ?
I. Yes, it will mean taking employment at Arguments
an early date. I. Yes, Government can earn huge amount
II. No, it will mean lack of depth of of money by imposing tax on the amount
knowledge. involved in gambling as people otherwise
19. StatementShould Central Government gamble illegally.
well equipped hospital for every sub-division II. No, people belonging to poorer sections
of every district ? of the society will spend their entire
Arguments earnings on gambling if it is made legal
I. Yes, health and well being of every which will lead them and their family to
citizen is the primary responsibility of starve.
the government.
II. No, it is not possible society must come 25. StatementShould all the small shops
forward to help the government. selling household commodities be closed
20. StatementShould the age of retirement in down in big cities in India ?
government services be increased in view of Arguments
longer life span in India ? I. No, all those people engaged in running
Arguments these small shops will be jobless and will
I. Yes, other countries have already taken be left with no earning to manage their
such decision. livelihood.
Logical Reasoning & Analytical Ability | 41

II. Yes, people prefer to carry out all their II. No, the authorities do not have right to
purchases under one roof in big shopping impose such restrictions in a democratic
malls in the big cities and hence these country.
small shops may not get customers to 31. StatementShould all the villages in India
survive. be compulsory given training on use of
26. StatementShould there be a complete ban modern techniques of cultivation ?
on use of chemical pesticides in the Arguments
agricultural fields ? I. No, the training should be given on
Arguments selective basis depending upon the land
I. No, the crops will get damaged by the holding and crop pattern.
pests as almost each crop is attacked by II. Yes, this is the only way to improve the
pests during its life cycle. economic situation in the rural areas.
II. Yes, this pollutes environment and also 32. StatementShould day to day management
contaminates ground water, instead of all the government hospitals in the country
biological pesticides should be used. be handed over to the private companies on
27. StatementShould there be only one operate and maintain basis ?
entrance test for all the medical colleges-both Arguments
Govt. and private in India ? I. Yes, this will considerably improve the
Arguments quality of services of these hospitals
I. No, each college has unique requirement benefiting both the patient and the
and decisions on admissions should be private companies.
left to the concerned colleges. II. No, the poor people may not be able to
II. Yes, this will bring in uniformity in the afford high charges fixed by the private
standard of students at the time of companies and they will be deprived of
admissions and will reduce multiplicity. medical facilities.
28. StatementShould the maintenance of all 33. StatementShould the tuition fees in all
roads in the big cities be entrusted to private schoolsboth Govt. aided and unaided upto
companies ? class X be completely waived for all
Arguments children ?
I. Yes, this is the only way to ensure good Arguments
conditions of the roads throughout the I. Yes, this will improve the literacy rate in
year. our country.
II. No, the private companies are not II. No, particularly the unaided schools
capable of maintaining all the roads. should be allowed to charge tuition fees
29. StatementShould India stop exploring to meet the regular expenses.
presence of hydrocarbons in both onshore and 34. StatementShould there be no entertain-
offshore locations ? ment tax levied by the Govt. on the movie
Arguments shows in India ?
I. Yes, this leads to wastage of precious Arguments
resources as the hydrocarbon reserves are I. No, the Govt. must levy entertainment
scanty. tax on movie shows.
II. No, we must try to become less II. Yes, the common people should not be
dependent on foreign countries for our taxed for movie shows.
energy needs by exploring all possi- 35. StatementShould there be a complete ban
bilities. on registration of new vehicles in the big
30. StatementShould there be a complete ban cities in India ?
on sale of soft drinks within the premises of Arguments
all the schools in India ? I. Yes, this is the only way to reduce air
Arguments pollution and traffic congestion signifi-
I. Yes, this will considerably decrease the cantly in the big cities.
consumption of such drinks by the II. No, Govt. cannot put such restrictions as
school children as these contain harmful people can buy new vehicles as and
chemicals. when they need and can afford.
42 | Logical Reasoning & Analytical Ability

Answers with Explanation strong because ban on hotel is not a proper


1. (B) Clearly, reservations on communal basis way to end the success of international
will widen inter communal biases. So, criminals.
argument I is not clear. Also, it will be against 13. (A) Abolishing the internal assessment would
our secular policy, According to which no surely reduce favouritism on personal grounds
communal group is given preference over the because the teachers of that college would not
others. So, only argument II is strong. be involved in examination system so that
they cannot extend personal benefit to anyone
2. (B) Clearly, the spending of money on sports
whom they like. So, argument I is strong. But
cannot be avoided merely because it can be
it will not affect the control of teaching
spent upon our various other problems.
faculty on students because still the teachers
Therefore, argument I is not valid. In case the
would be teaching them. So, argument II is
expenses on sports are curtailed, the sport
not clear.
persons would face lack of facilities and
14. (B) Clearly, the pursunance of any policy in
training. Our country will lag behind in
India cannot be based on the ground that it is
international sports competitions.
followed in other countries. Basically, every
3. (E) Clearly, it is the advertisement which country has its own environments, culture and
makes the customer aware of the qualities of values. So, argument I is not clear. Also,
the product and customer leads to buy it. So, imparting sex education in co-educational
argument I is valid, But on the contrary schools where boy and girls study together,
advertising of any product on various media would spoil the atmosphere and hinder the
has become a costly matter and the expenses studies. So, argument II is strong.
on it add to the price of product. So, argument 15. (B) Clearly, imposing ban on fashionable
II is also strong. dresses will a restriction on the personal
4. (D) The PDS is indeed necessary to provide choice and it violates the right to freedom of
basic amenties to the poorer sections of an individual. Therefore, only argument II is
society. So, argument is not clear. Also, if the strong.
objectives and purposes of any system are not 16. (C) Family planning is an essential and
fulfilled because of corruption then geting rid important step to curb population growth. So,
of the system is not a proper solution, efforts argument I is strong. Also family planning
should be made to end coruption and we being against the beliefs of some of the Indian
should extend its benefit to the needful religions, it is not necessary to make it
people. So, argument II is also not valid. compulsory or mandatory. It would be better
5. (B) 6. (D) 7. (B) 8. (A) 9. (C) if it is enforced by creating public awareness
10. (B) UNO is meant to maintain peace and of the benefits of family planning. Therefore,
harmony all over the world and it will always argument II is also valid.
serve to prevent conflicts and wars among 17. (B) It is clear that a world government cannot
member countries. So, its role never ends. So, remove tensions and liability among nations
argument I is not strong. Lack of such type of because it will also have the ruling group and
organisation may in future lead to increased the opposition groups. Simultaneously, the
mutual conflicts and international wars. So, more powerful and diplomatic shall rule the
argument II is strong. world to their interests. So, only argument II
11. (B) Clearly, strike is not a means of indisci- is strong.
pline but only a practice and tool in which the 18. (B) Clearly, argument I is not clear because at
citizens of any particular country exercise present too many fields are open to all after
their fundamental rights. So, argument I is not graduation. However, eliminating the post
clear and argument II is valid. graduation courses would abolish higher and
12. (B) Clearly, the luxary hotels of any country, specialised studies which provide to
are a mark of countrys standard and place for understanding thing better and deeply. So,
staying the affluent people of that country as argument II is valid.
well as foreign tourists. So, argument II is 19. (A) Clearly, health care services must be
strong. But, on the contrary, argument I is not cared by the government and opening well-
Logical Reasoning & Analytical Ability | 43

equipped hospitals in each and every area 28. (D) In argument I, it is not necessary that
would surely provide better health services to private companies are capable to do better
every citizen. So, argument I is valid and service throughout the year. In argument II,
strong. Also, it is not an impratical task and we cannot blame that these companies are not
can be achieved by the government. So, capable of maintaining all the roads.
argument II is vague. 29. (C)
20. (B) Clearly, the pursuance of a policy in India 30. (A) Argument I is strong because soft drinks
cannot be based on the pretext that it is contain some harmful chemicals which are
followed in other countries because each and harmful to school children. But II is not
every country has its own environment and strong because it is not the meaning of
situations. So, argument I is not clear. But democratic country that we cannot ban on
increasing the age of retirement is indeed a wrong things.
genuine demand of most of the employees to 31. (B) Argument I is not valid, because it
be self dependent throughout. So, argument II emphasizes on selection on the basis of land
is valid and strong. holding and crop pattern, which is not fair and
21. (B) Being an unlimited number of one-day justified. Argument II asserts the importance
international matches will be a cause of extra of training particularly in rural area.
labour, therefore, if any work is done beyond 32. (B) Argument I is not valid, because it is not
the limit be the cause of harmfulness. necessary that private companies will care for
Therefore, a limited number of matches will the service benefitting for the patients. On the
be significance of proper controlling, other hand, they will think over only for their
mentality and healthfulness. personal and monetary benefit. In argument
22. (D) Neither I nor II argument is strong. II, it is clearly given that the poor people may
23. (C) One of these two arguments will be strong not be able to afford high charges fixed by the
necessarily, because it is impossible to private companies and the poor people will
happen both together because these two not get proper medical facilities.
arguments are opposite to each other. 33. (C) The improvement in the literacy rate is
24. (E) Both the arguments are strong. Argument the first necessity of our country. Therefore,
I is therefore, strong because when the argument I is strong. The argument II is also
gambling will be legalized and tax will be strong because unaided schools are in need of
imposed and the Government revenue will income source to look after and manage the
increase, but II is therefore, strong, because school. Hence, either argument I or argument
the people of poorer section will gamble II is strong because these are opposite to each
beyond their earning limit. other.
25. (A) Only the argument I is strong, because if 34. (A) Argument I is strong and valid, if the
the shops are closed the people of poorer government pass the order to waive the
sections will get suffer and the effect of tuition fees on all the aided and un-aided
money centralization will be strong. school upto class, it will improve literacy rate
26. (E) I and II both arguments are strong, if there in India.
is no ban on the use of pesticides, the whole 35. (B) Argument I is not valid because gov-
crop will be damaged by pests and if there is ernment cannot ban on registration of new
ban on the use of pesticides then it will vehicles. One has right to purchase whatever
pollute the environment. one wants if one has able to purchase.
27. (B) Only argument II is strong because by Argument II is valid because it supports the
only one entrance test it will bring uniformity idea of the right of any person who lives in a
in the standard of students which is necessary. country.

6 Statement and Assumptions
An assumption is the hidden part of an Assumptions
argument and it is something taken for granted. It I. We will always have necessary funds in
means a fact that can be supposed as considering our account to settle the bills.
the contents of the given statement. In these II. We are capable of verifying and clearing
questions, a statement is followed by two or more the bills in less than three working days.
assumptions. The candidate is required to assess
SolutionSince, the statement says for
the given statement and decide which of the given
making all payment within three days, it is clear
assumptions is implicit in the statement ?
that the company has the necessary funds and the
DirectionIn each question below a state- bills can be verified and cleared within the
ment (or a passage) followed by assumptions stipulated period. So, both the assumptions I and
numbered I and II. An assumption is something II are implicit.
supposed or taken granted. Consider the statement
Example 3.
and decide which of the given assumptions is
implicit ? StatementThe patient condition would
improve after this operation.
Give Answer
Assumptions
(A) if only assumption I is implicit.
I. The patient can be operated upon this
(B) if only assumption II is implicit. condition.
(C) if either assumption I or II is implicit. II. The patient cannot be operated upon this
(D) if neither assumption I or II is implicit. condition.
(E) if both the assumptions are implicit. SolutionIt is very much clear in the
Example 1. statement that the patient is in a position to be
StatementThe Chairman and the Secretary operated upon. Hence, assumption I is implicit
of the Housing Society have requested society second assumption is contrast to the I assumption.
members to use water economically to help It will not implicit.
society to save water on tax. Example 4.
Assumptions StatementIt is desirable to put the child in
I. Majority of members of society are school at the age of 5 or so.
likely to follow the request. Assumptions
II. It is desirable to reduce expenditure I. At the age the child reaches appropriate
wherever possible. level of development and is ready to
SolutionIn assumption I, nothing about the learn.
society members to the societys request can be II. The schools do not admit children after
deduced from the statement. So, assumption I is six years of age.
not implicit. But from assumption II, if it is SolutionThe statement I talks of putting
possible the expenditure can be reduced. So, the child in school at the age of 5, it means that
assumption II is implicit. the child is mentally prepared for the same at this
Example 2. age. So, assumption I is implicit. But nothing
StatementWe must settle all the payment about admission after six years of age is said in
due to our suppliers within three working days. the statement. So, assumption II is not implicit.
Logical Reasoning & Analytical Ability | 45

Example 5. 14. StatementLike a mad man, I decided to


StatementAs there is a great demand, every follow him.
person seeking tickets of the programme will be Assumptions
given only five tickets. I. I am not a mad man.
Assumptions II. I am a mad man.
I. The organisers are not keen on selling 15. StatementIf it is easy to become an
the tickets. engineer, I dont want to be an engineer.
II. No one is interested in getting more than Assumptions
five tickets. I. An individual aspires to be professional.
SolutionClearly, the organisers are II. One desires to achieve a thing which is
adopting the policy not to reduce the sale but to hard earned.
cope with great demand in order that everyone get
16. StatementAll the employees are notified
the ticket. So, I is not implicit. Also, due to great
that the organisation will provide transport
demand, the maximum number of tickets one
facilities at half the cost from the nearby
person has been reduced to five. So, II is also not
railway station to the office except those who
implicit.
have been provided with travelling allowance.
Exercise Assumptions
DirectionsIn each question below is given I. Most of the employees will travel by the
a statement followed by two assumptions office transport.
numbered I and II. An assumption is something II. Those who are provided with travelling
supposed or taken for granted. You have to find allowance will not read such notice.
out from the following assumptions which one is 17. StatementAn advertisement of a Bank
implicit in the satatement. Want to open a bank account ! Just dial our
Give Answer room service and we will come at your
(A) if only assumption I is implicit. doorsteps.
(B) if only assumption II is implicit. Assumptions
(C) if either I or II is implicit. I. There is a section of people who require
(D) if neither I nor II is implicit. such service at their home.
(E) if both I and II are implicit. II. Now a days banking has become very
11. StatementOpening a library in Rambali competitive.
will be a wastage. 18. StatementI can take you quickly from
Kanpur to Lucknow by my cab but then you
Assumptions
must pay me double the normal charges.
I. Inhabitants of Rambali are illiterate. Assumptions
II. Inhabitants of Rambali are not interested I. Normally, it will take more time to reach
in reading. Lucknow from Kanpur.
12. StatementA car is required on rentan II. People want to reach quickly but they
advertisement. will not pay extra money for it.
Assumptions 19. StatementPatients condition would
I. All types of vehicles are available on improve after this operation.
rent. Assumptions
II. People will respond to the advertisement. I. The patient can be operated upon in this
13. StatementAs advice to BIf you want condition.
to study computer, join Institute X. II. The patient cannot be operated upon in
Assumptions this condition.
I. Institute X provides good computer 10. StatementEven with the increase in the
education. number of sugar factories in India, we still
II. B listen to As advice. continue to import sugar.
46 | Logical Reasoning & Analytical Ability

Assumptions II. There are other books available on this


I. The consumption of sugar per captia has issue.
increased in India. 17. StatementWhy dont you invite Anthony
II. Many of the factories are not in a for the Christmas party this year ?
position to produce sugar to their fullest Assumptions
capacity. I. Anthony is not from the same city.
11. StatementUnemployed allowance should II. Unless invited, Anthony will not attend
be given to all unemployed youths above 18 the party.
years of age.
18. StatementIf you offer peanuts, you get
Assumptions only monkeys.
I. There are unemployed youths in India
Assumptions
who need monetary support.
II. The government has sufficient funds to I. Money is not a good motivation.
provide allowance to all unemployed II. Money cannot buy everthing.
youth. 19. StatementBest way to solve this problem
12. StatementThe leader of main opposition of workers dissatisfaction is to offer them
asserted that the call for chakka jam turned cash reward. If this type of incentive can
out to be a great success in the entire state. solve the problem in CIDCO company than
Assumptions why not here. A personnel manager tells the
I. The people in future will support the Chairman of a company.
main opposition party. Assumptions
II. People probably are convinced about the I. The reason for workers dissatisfaction in
reason behind the chakka jam strike call. both company was similar.
13. StatementA sentence in the letter to the II. Monetary incentives have universal
candidates called for written examsyou appeal.
have to bear your expenses on travel etc. 20. StatementWe need to appoint more
Assumptions teachersPrincipal informs the school staff.
I. If not clarified, all the candidates may Assumptions
claim reimbursement of expenses.
II. Many organisations reimburse expenses I. Teachers are available.
on travel to candidates called to written II. Present teachers are not good.
examination. 21. StatementWe should use detergents to
14. StatementIn case of any difficulty about clean objects.
this case you may contact our companys Assumptions
lawyer. I. Detergents help to dislodge grease and
Assumptions dirt.
I. Each company has lawyer of his own. II. Detergents form more lather.
II. The companys lawyer is thoroughly 22. StatementEvery year doctors, scientists and
briefed about this case. engineers migrate from India to greener
15. StatementA good executive has to be task pastures.
oriented as well as people-oriented.
Assumptions
Assumptions
I. Brain drain has affected adversely.
I. Some executives are people oriented.
II. Some people are not people-oriented. II. Better scales and better standards of
living at as a bait to lure them.
16. StatementRead this book to get detailed
and most comprehensive information on this 23. StatementThe government is making
issue. efforts boost tourism in J & K.
Assumptions Assumptions
I. The person who wants this information I. Tourism in J&K dropped during last
can read. couple of years.
Logical Reasoning & Analytical Ability | 47

II. Special discount in the railway fare has It may be possible that opening a library in
been announced. Rambali is a wastage because there is already
24. StatementGovernment should deploy army a library there.
to rehabilitate the people displaced due to 12. (B) This statement says that a car is required
earthquake. for rent and it does not say that any vehicle is
Assumptions required. So, it cannot be assumed that any
I. Army can be used purposed other than vehicle is available for rent. But assumption II
war also. clearly implicit. Whenever any advertisement
II. Only army can rehabilitate the displaced is published, it is assumed that people will
victims of earth quake. respond to the advertisement.
25. StatementThe railway authorities have 13. (B) Since, A advises B to join institute X
decided to increase the freight charges by 10 to study computer, hence it is assumed that
per cent in view of the possibility of incurring institute X provides good coaching for
losses in the current financial year. computer. Secondly, it is also assumed that B
listens A advice.
Assumptions
I. The volume of freight during the 14. (A) In the statement, it is said like a mad man.
remaining period may remain same. It means not a mad man. Hence, assumption
I is implicit.
II. The amount so obtained may set off a
part or total of the estimated deficit. 15. (B) It is not given in the statement, an
individual always aspires to be a professional
26. StatementThe government has decided to but we can deduce that the a person desires
reduce the custom duty on computer those things which are hard earned.
peripherals.
16. (D) From the statement, both the assumptions
Assumptions are not implicit.
I. The domestic price of computer
17. (E) Both the assumptions are implicit. There
peripherals may go in near future.
is a section of people who can be benefitted
II. The domestic manufactures may oppose by such type of service. This type of
the decision. advertisement has been given by the bank
27. StatementHelping the poor is the real because now-a-days banking has become
service to the humanity. competitive.
Assumptions 18. (A) The word quickly shows in the statement
I. Poor people are in need of help from that normally, it takes more time to reach
others. Lucknow to Kanpur and the cab-drivers
II. If we do not help poor, we will not be promises to save the time. Assumption II does
called human being. not implicit in the statement.
28. StatementThe police in India have to cope 19. (A) It is very much implied in the statement
with tremendous stress and strain, while that the patient is in a position to be operated
maintaining security and order. soon. Hence, assumption I is implicit.
Assumptions 10. (D) Neither of the assumptions is implicit in
I. In other countries the police do not have the statement as there may various reasons for
to undergo stress and strain while doing importing sugar.
their duties. 11. (A) Assumption I says that Indian
II. The police are expected to do their duties unemployed youths need monetary support is
without stress and strain. solid reason for providing allowance to all
unemployed youths. However, assumption II
Answers with Explanation that government has sufficient funds does not
11. (D) None is implicit in the statements. The valid deduction. Hence, only assumption I is
statement only says that opening a library implicit.
would be wastage but it does not mention 12. (B) Since, the Chakka jam call was accepted
why ? Assumptions I and II are only guessess. by and large by the entire state. It means that
48 | Logical Reasoning & Analytical Ability

people are convinced about the reasons are similar, that is why the same solution has
behind the Chakka jam strike call. Hence, been recommended for their problems.
assumption I is implicit. However, it is not Secondly, it assumed in the context of the
necessary that people will support the main statement that monetary incentives have
opposition party in future without knowing universal appeal.
any strong and valid reason. 20. (A) The school requires more teachers. It is
13. (E) In the letter, specific instruction has been assumed in the context of statement that
given in writing to prevent chance of claim of teachers are available but it is not clear that
reimbursement of travelling expenses present staff of teachers of that school is not
Moreover, it can also be assumed that good. Therefore, only assumption I is
instruction has been given because expenses implicit.
are paid by other organisation. Therefore, 21. (A) It is assumed that detergents generally
both the assumptions are implicit. help to clear grease and dirts, that is why it
14. (B) It is not necessary that every company has has been recommended in the statement.
a lawyer. Hence, assumption I is not implicit. Hence, assumption I is implicit.
As it is advised in the statement that for any 22. (B) The statement is based on the assumption
difficulty about the case, consult companys that the lucrative offers of any organisation
lawyer. It is, therefore assumed that tempt the scientists to migrate abroad. Hence,
companys lawyer has been thoroughly assumption II is implicit.
briefed about the case. Therefore, assumption
23. (A) It has been assumed that the government
II is implicit.
is making efforts to boost the tourism in J&K
15. (D) The statement explains that an executive during last couple of years due to some
should be task oriented as well as people reasons. Therefore, assumption I is implicit.
oriented. It is based upon the assumptions
that some executives are task oriented while 24. (A) It is given in the statement that army
some are people-oriented. Therefore, none of should be deployed to rehabilitate people. It
the assumptions follows. means that army can be used for purposes
other than war. Hence, assumption I is
16. (A) It can be assumed that the person implict.
instructed is capable of reading the book.
However, we cannot say anything about the 25. (B) Nothing is said about volume of freight
other sources of information. Hence, the during the remaining period. So, assumption I
information I is implicit. is not implicit in the statement. Secondly, it is
given in the statement that fare has been
17. (B) We cannot say that Anthony is not from increased in view of the possibility of
the same city. However, it is assumed that incurring losses. Hence, assumption II is
unless invited, Anthony will not attend the implicit.
party as invitation has been emphasised in the
statement. Therefore, only assumption II is 26. (D) None of the assumption is implicit in the
implicit. statement.
18. (D) None of the assumptions is implicit 27. (A) The statement tells that the poor are in
because they are not related with the need of help from others which is a quality of
statement. being of a good human being. Assumption I is
implicit.
19. (E) It is clear that the reasons given for
workers dissatisfaction in both the companies 28. (D) None of the assumptions is implicit.


7 Passage
In such type of questions, a passage is given and in the third if step, we see if some key words
which is usually a part of report or an economic justify the given inference.
essay or any other similar thought-provoking topic Example of PassageThe immediate
or subject some inferences are given followed by challenge is on the food front. Shortfalls in
the passage. Generally, there are five inferences production have been allowed to affect supplies
the candidate is asked to decide whether the a and hence prices. The Government is planning to
given inference follows or not in the context of the focus on investment in irrigation and even revival
passage. of agricultural extension system what is probably
The pattern of the passage is given below : needed is a fresh dose of Green Revolution
strategy. It appears that the Green Revolution
DirectionsGiven below a passage followed
instruments to encourage farmers to invest are no
by five inferences based on it. The candidate has
longer effective. The Green Revolution strategy
to examine the passage carefully and then decide
was based on the state taking out the risk of
the validity of the each inference.
collapse in prices. Farmers were offered
Mark Answer remunerative prices and a guaranteed procurement
(A) if the inference is definitely true i.e., of their produce in case the open market could not
it properly follows from the statement of absorb it. Farmers could then borrow from banks,
facts given. acquire the Green Revolution Technology and
(B) if the inference is probably true though produce as much as they could. The pressure on
not definitely true in the light of the the food subsidy was manageable as long as there
facts given. was a food shortage. Prices in the open market
(C) if the data are inadequate i.e., from the then tended to be above the procurement prices.
facts given you cannot say whether the But with the food surpluses the situation has
inference is likely to be true or false. changed. The situation was unsustainable not
(D) if the inference is probably false though merely because of the magnitude of this subsidy.
not definitely false in the light of the It was also inefficient. It meant farmers were
facts given. being led to produce crops based just on the prices
(E) if the inference is definitely false i.e., it Government fixed and not in relation to any real
cannot possibly be drawn from the facts demand. In these circumstances, the Government
given or it contradicts the given facts. was reluctant to keep increasing procurement
prices at the pace that used to be the norm in
We can solve the passage in three steps
earlier years.
(i) Directly follows from the given passage. 1. The Government is planning to make crucial
(ii) Can be inferred from the given passage. changes in the Green Revolution strategies.
(iii) Can be inferred with the help of some 2. The Government is no longer in a position to
key words in the given passage. provide subsidy to farmers.
In the first step try the easy questions where 3. As the open market prices are lower, all the
an inference is more or less a restatement of some burden of procurement of crops in on the
thing which is already stated in the passage. In the Government.
second step we check if the inference can be 4. Demand is much higher than the quantity of
derived from the facts mentioned in the passage crops produced by the farmers.
50 | Logical Reasoning & Analytical Ability

5. The farmers tend to produce the crops as per and biotech manufacturers and their shareholders.
their convenience and not consonant with the However, companies and their investors may risk
demand. value destruction if they acquire rights to a drug
that suddenly poses unanticipated safety risks for
Answers with Explanation patients. Similarly, safety related compliance
1. (B) The statement is probably true though not violations committed by an acquired company
definitely true in the light of the facts given as could significantly impair the ultimate value of the
according to the passage. The Government is transaction and the reputation of the acquirer. The
planning to focus on investment in irrigation pace of deal making between pharma and biotech
and revival of agriculture extension which companies continued to accelerate in 2006,
is probably needed in green revolution increasing 17% to about $ 18 billion. Pharma
strategies. companies were typically the buyers, and the
2. (C) Nothing is said about it in the passage so premiums they paid increased substantially as
the data are inadequate. competition intensified, to secure access to novel
3. (A) The given statement is definitely true as it drugs and biologics. The stakes increase everyday
is given in the passage that the farmers were as competition pushes up prices and drives deal-
offered remunerative prices and a guaranteed making to earlier development stages with greater
procurement of their produce in case the open uncertainty and less time to complete thorough
market could not absorb it. due diligence
1. Acquisitions of biotech companies was
4. (E) The given statement is definitely true
preferred among pharmaceutical companies in
because it is mentioned in the passage that the
the recent past.
farmer were being led to produce crops based
just on the prices government fixed and not in 2. Biotech companies are not capable to acquire
relation to any real demand. pharmaceutical companies.
3. Pharmaceutical companies at times may incur
5. (A)
loss after acquisition of biotech companies.
Exercise 4. Safety related issues are prime concerns for
Directions(Q. 15) Below given a passage the pharmaceutical companies while negotiat-
followed by several possible inferences which can ing acquisitions of biotech companies.
be drawn from the facts stated in the passage. You 5. Stiff competitions among the prospective
have to examine each inference separately in the buyers have resulted into erosion of value of
context of the passage and decide upon its degree the biotech companies.
of truth or falsity.
Directions(Q. 610) Below is given a
Mark Answer passage followed by several possible inferences
(A) if the inference is definitely true, i.e., it which can be drawn from the facts stated in the
properly follows from the statement of passage. You have to examine each inference
facts given. separately in the context of the passage and decide
(B) if the inference is probably true though upon its degree of truth or falsity.
not definitely true in the light of the Mark Answer
facts given. (A) if the inference is definitely true, i.e., it
(C) if the data are inadequate, i.e., from the properly follows from the statement of
facts given you cannot say whether the facts given.
inference is likely to be true or false. (B) if the inference is probably true though
(D) if the inference is probably false, not definitely true in the light of the
though not definitely false in the light facts given.
of the facts given. (C) if the data are inadequate, i.e., from the
(E) if the inference is definitely false, i.e., it facts given you cannot say whether the
cannot possibly be drawn from the facts inference is likely to be true or false.
given or it contradicts the given facts. (D) if the inference is probably false,
Growth through acquisitions and alliances has though not definitely false in the light
become a critical part of creating value for pharma of the facts given.
Logical Reasoning & Analytical Ability | 51

(E) If the inference is definitely false i.e., it (D) if the inference is probably false,
cannot possibly be drawn from the facts though not definitely false in the light
given or it contradicts the given facts. of the facts given.
The latest data to show that the overall power (E) if the inference is definitely false, i.e.,
situation has gotten worse, with the ratio for peak- it cannot possibly be drawn from the
load shortages now the highest in a decade. In facts given or it contradicts the given
absolute terms, the power deficit has hit record facts.
levels and seems almost certain to further The main benefit to the economy from an
deteriorate without real reforms on the ground. active stock market is the ready availability of risk
Even as aggregate technical and commercial capital for investment in equities through the
losses in the power system remain much high at primary market. For that risk capital be readily
over a third of total generation, pan-India capacity available. Investors need to have an easy exit
addition is now well below target. A shortage of route. A liquid secondary market provides an easy
equipment and skills is blamed for the marked exit route through the active involvement of
slow down in augmenting power capacity. But the buyers and sellers. It does not matter whether
dearth of resources can only be relative. In fact, these buyers and sellers have short or long term
the real bane of the sector is continuing revenue investment horizons. Liquidity in the market is
leakage in the state power utilities and enhanced by leveraged players who either borrow
unacceptably high aggregate technical and to play the market or achieve a similar result
commercial losses, much of it plain theft of through futures contracts whose economic value
electricity. Given the preponderance of state includes financing costs. Short sellers confer a
utilities in power supply, the fact that they remain similar benefit by borrowing stock or achieving
very much in red does affect investor comfort and the same result through futures contracts.
return funds flow.
11. Nature of activity of the longterm players in
6. Indian power generation is largely controlled
the secondary market has significant effect on
by private sector.
the health of stock market.
7. Reforms in power sector in India has not yet
attained its desired level. 12. Players in the primary market generally
borrow funds and earn money by quick
8. Indian power sector is yet to attain status
disposal in the secondary market.
comparable to developed countries.
9. Power theft is one of the major components of 13. Active secondary market provides liquidity to
revenue losses in power sector. the transactions in primary market.
10. Aggregate technical and commercial loss is 14. Passive stock market enhances availability of
much less than thirty per cent of the total capital.
power generation. 15. Only short term buyers or sellers provide
Directions(Q. 1115) Below is given a effective exit route to the primary market.
passage followed by several possible inferences Answers with Explanation
which can be drawn from the facts stated in the
passage. You have to examine each inference 1. (A) The inference is definitely true because
separately in the context of the passage and decide according to the given passage The pace of
upon its degree of truth of falsity. deal making between pharma and biotech in
Now, mark your answer as 2006 is increasing accelerately.
(A) if the inference is definitely true i.e., it 2. (C) According to the passage Pharma com-
properly follows from the statement of panies were typically the buyers out it is not
facts given. necessary that Biotech companies are not
(B) if the inference is probably true capable to acquire pharmaceutical companies.
though not definitely true in the light of Hence, data is inadequate.
the facts given. 3. (A) The inference is definitely true as it is
(C) if the data are inadequate, i.e., from clearly mentioned in the passage. However,
the facts given you cannot say whether companies and their investor may risk value
the inference is likely to be true or false. destruction if they acquire rights to a drug
52 | Logical Reasoning & Analytical Ability

thatsuddenly poses unanticipated safety risks 10. (E) Given in the passage aggregate technical
for patients. and commercial losses in the power system
4. (B) remain much high at over a third of total
5. (E) The inference is definitely false as it is generation and the inference contradicts the
given in the passage that competition pushes given facts. Hence, definitely false.
up prices. 11. (B) Because the role of long term players has
6. (C) In this passage, nothing is mentioned significant effect on the health of stock
regarding Indian power generation. Hence, market and this inference can be probably
data are inadequate. true. We can only infer in the light of the facts
7. (B) As the whole passage narrates for the given.
power situation, it can be inferred that 12. (C) No data are given in the passage.
reforms in power sector in India has not yet 13. (D)
attained its desired level. Hence, the inference 14. (A)
is probably true.
15. (E) It contradicts the statement in the passage.
8. (C) The statement says that a liquid secondary
9. (A) Given in the passage much of it plain market provides an easy exit route through
theft of electricity. Hence, definitely true. the active involvement of buyers and sellers.

8 Statement and Course of Action
In many of the competitive examinations this II. The banks should advise their customers
type of questions has become a regular feature. In to be more careful in dealing with the
such type of questions, a situation is presented and plastic money.
some courses of action are suggested in the SolutionThe course of action I follows
context of the situation. because according to the statement, an increased
To solve this type of questions, students are number of credit/debit and forgery have been
advised to keep in mind various points. recorded recently. Hence, the banks should
(A) Correct course of action should either make stricter verification to ensure the access
lessen the problem or improve the to credit/debit by the authorised person.
situation created by the problem. Simultaneously the banks should advise their
(B) Course of action should be feasible and customers to be more careful in dealing with the
should relate with the practical aspect of plastic money. Hence, both I and II courses of
life. action follow. Therefore, our answer is (E).
(C) Decide quickly whether the problem Example 2.
involves a problem solution relationship StatementDrinking and rave parties have
or a fact improvement relationship. become fashion among the present college
(D) Simple problem must have simple course youth which divert from their studies.
of action and not a complex one which Courses of Action
may create more problem than to solve I. The State Government should make it a
or reduce it. rule to disallow the students from entering
DirectionIn each question below is given a Pubs and Bars.
statement followed by two courses of action II. The parents as well as the colleges should
numbered I and II. The candidates is required to impose strict discipline among the college
grasp the statement analyse the problem and then going youth.
decide which of courses of action logically follow.
SolutionIn order to stop the present college
Give Answer
youth from drinking and attending the rave parties
(A) if only I follows. the State Government, the parents as well as the
(B) if only II follows. college should impose strict discipline among
(C) if either I or II follows. them. Hence, I and II both follow. Therefore, our
(D) if neither I nor II follows. answer is (E).
(E) if both I and II follow. Example 3.
Example 1. StatementIncreasing competition on
StatementAn increased number of credit/ various fronts is putting children and youth
debit card forgery cases have been recorded under tremendous pressure leading them to
recently. take extremely drastic steps.
Courses of Action Courses of Action
I. The banks should make stricter verifica- I. Children and youth alongwith their
tion to ensure the access to credit/debit parents should be counselled for facing
card by the authorized person. the competition in right spirit.
54 | Logical Reasoning & Analytical Ability

II. Parents and teachers should be advised 3. StatementPeople in the locality were
not to bring pressure on children. agitated as more than thirty people died in a
building collapse.
SolutionIncreasing competition on various
fronts is putting children and youth under Courses of Action
tremendous pressure leading them to take I. Government should immediately
extremely drastic steps. In order to avoid, children announce compensations for the affected
alongwith their parents should be counselled for families.
facing the competition in right spirit. Hence, I II. Authorities should take a stringent action
action follows II is not proper. Therefore, our against builders tending to compromise
answer is (A). over quality of material used.
4. StatementWith the onset of monsoon all
Exercise the hospitals are getting increased number of
Directions(Q. 15) In each question below patients due to various epidemics.
is given a statement followed by two courses of Courses of Action
action numbered I and II. A course of action is a I. Civic authorities should educate the
step or administrative decision to be taken for public the need for observing minimum
improvement, follow-up or further action in required hygiene.
regard to the problem, policy, etc. On the basis of II. Civic authorities should make arrange-
the information given in the statement, you have ments to equip the hospitals with
to assume everything in the statement to be true, required medicines and other facilities.
then decide which of the suggested courses of 5. StatementMore number of students
action logically follow(s) for pursuing. passing SSC examination has resulted into
Give Answer frustration among children for not getting
(A) if only I follows. admissions into colleges of their choice.
(B) if only II follows. Courses of Action
(C) if either I or II follows. I. Government should permit the colleges
to increase the number of seats.
(D) if neither I nor II follows.
II. Children and their parents should be
(E) if both I and II follow. counselled for being flexible on the
1. StatementMisuse of subsidies offered to choice of college.
the farmers was observed and brought to the Directions(Q. 610) In each questions
notice of the concerned authorities. below is given a statement followed by two
Courses of Action courses of action numbered I and II. A course of
I. Government should issue orders to the action is a step or administrative decision to be
concerned officials to be stricter and taken for improvement, follow-up or further
more careful while verifying the required action in regard to the problem, policy, etc. On the
documents while granting subsidy. basis of the information given in the statement,
you have to assume everything in the statement to
II. Government should take stringent action be true, then decide which of the suggested
against those making false claim of courses of action logically follow(s) for pursuing.
subsidy. Give Answer
2. StatementExpensive clothes and accesso- (A) if only I follows.
ries are becoming a growing need among (B) if only II follows.
college going teenage children of middle-
(C) if either I or II follows.
income group.
(D) if neither I nor II follows.
Courses of Action (E) if both I and II follow.
I. Colleges should introduce a dress code. 6. StatementThe local college principal has
II. Children should be counseled empha- ordered that all the students must strictly
sizing the importance of many other adhere to the dress code stipulated by the
things. college authority in the admission brochure.
Logical Reasoning & Analytical Ability | 55

Courses of Action who may not adhere to the time


I. Those students who are found to violate schedule.
the dress code should be terminated from II. All such employees who are found to be
the college. failing to maintain time schedule should
II. Those students who are found to violate be summarily suspended.
the dress code for the first time should be Directions(Q. 1115) In each question
reprimanded and be warned against below is given a statement followed by two
further violation. courses of action numbered I and II. A course of
7. StatementThe railways have decided to action is a step or administrative decision to be
repair the main tracks within the city on the taken for improvement, follow-up or further
following Sunday and has decided to suspend action in regard to the problem, policy, etc. On the
operations for the whole day. basis of the information given in the statement,
you have to assume everything in the statement to
Courses of Action
be true, then decide which of the suggested
I. The railway authority should issue public courses of action logically follow(s) for pursuing.
notification well in advance to ease Give Answer
inconvenience to the passengers.
(A) if only I follows.
II. All the long distance trains at the time of (B) if only II follows.
repair hours should be terminated outside
(C) if either I or II follows.
the city limit.
(D) if neither I nor II follows.
8. StatementMany motorists driving on the
(E) if both I and II follows.
highway within the city are found to be
driving much beyond the permissible speed 11. StatementA speeding truck has seriously
limit. injured many persons sleeping on the roadside
early in the morning.
Courses of Action
Courses of Action
I. The traffic police officials should
personally monitor the movement of I. The local administration should
vehicles on the highway within the city. immediately put a complete ban on
people sleeping on the roadsides.
II. The Govt. should immediately put in
place a mechanism to identify and punish II. The driver of the speeding truck should
erring drivers. be nabbed and tried for the crime he
9. StatementMajority of the city employees committed.
in the renowned BPO company has left their 12. StatementThere has been an unprece-
jobs to protest against inhuman treatment dented increase in the number of students
meted out to them by the company. applying for admission to first Std. in a local
Courses of Action school making it difficult for the school
I. The Govt. should immediately order the authority to convince the parents of rejected
BPO company to close down its applicants.
operation. Courses of Action
II. The BPO company should shift its I. There has been an unprecedented
operations to some other place to increase in the number of students
continue its operations. applying for admission to first Std. in a
10. StatementThe management of the local school making it difficult for the
organization has issued a circular to all its school authority to convince the parents
employees stating that each employee must of rejected applicants.
report for duty at 10.00 a.m. sharp and should II. The school authority should open another
remain in his/her work place till 5.30 p.m. school in the area to accommodate the
everyday. remaining students.
Courses of Action 13. StatementThe apex court has directed that
I. The management should evolve a there is a need to bring in mechanism in the
mechanism to identify such employees Govt. functions to make it transparent.
56 | Logical Reasoning & Analytical Ability

Courses of Action them the circumstances under which the


I. The Govt. should immediately appoint a fees had to be hiked.
task force to work out the modalities. II. The school authority should immediately
II. The Government should appeal to the scout for new students to fill the vacancy
apex court to reconsider its directive. if large number of students are
14. StatementMany people in the coastal town withdrawn from the school.
are suffering from respiratory diseases during 17. StatementThe prices of mangoes in the
past few months. local market have dropped substantially due
Courses of Action to supply-demand mismatch.
I. The Government should immediately Courses of Action
send a team of health professional to I. The Govt. should encourage the
provide medical care to the affected wholesale fruit suppliers to sell the
people. mangoes to the markets of other states.
II. The people suffering from such diseases II. The Govt. should encourage the farmers
should be kept in a special ward and put to extract pulp and sell the pulp to the
through proper medication. wholesalers.
15. StatementThere has been a substantial 18. StatementA large mob gathered outside
drop in number of people opting for new the local police station to protest the recent
telephone connection from the public sector harassment of local people by some police
telephone company in the recent months. personnel.
Courses of Action Courses of Action
I. The public sector telephone company
should immediately set up a committee I. The officer in charge of the police station
to identify the reasons for the drop. should be instructed to persuade the mob
to disburse immediately.
II. The public sector telephone company
should offer new schemes with value II. The police authority should immediately
added services to woo the new clients. initiate an enquiry into the allegations
and inform the public accordingly.
Directions(Q. 1620) In each question
below is given a statement followed by two 19. StatementThe prices of crude oil in the
courses of action numbered I and II. A course of international market have risen by about 40
action is a step or administrative decision to be per cent in a months time and show no
taken for improvement, follow-up or further downward trend.
action in regard to the problem, policy, etc. On the Courses of Action
basis of the information given in the statement, I. The petrol prices in the domestic retail
you have to assume everything in the statement to market should immediately be increased
be true, then decide which of the suggested by about 30 per cent to absorb the cost
courses of action logically follow(s) for pursuing. escalation.
Give Answer II. The Govt. should provide subsidy to the
(A) if only I follows. oil marketing companies to absorb the
(B) if only II follows. cost escalation.
(C) if either I or II follows. 20. StatementMany students were caught red
(D) if neither I nor II follows. handed while using unfair means during the
(E) if both I and II follow. recent university examinations.
16. StatementThe parents of majority of the Courses of Action
students of the local public school have I. All these students should be debarred
decided to withdraw their wards from the from taking university examination in
schools to protest against the sharp fee hike. future.
Courses of Action II. The students apprehended while using
I. The school authority should start a unfair means should be handed over to
dialogue with the parents and explain to the police.
Logical Reasoning & Analytical Ability | 57

Directions(Q. 2125) In each question Courses of Action


below is given a statement followed by two I. Parents and teachers should not put
courses of action numbered I and II. A course of pressure on children for high achieve-
action is a step or administrative decision to be ment, rather they should help children
taken for improvement, follow-up or further face the situation as it comes.
action in regard to the problem, policy, etc. On the
II. School/college authorities should appoint
basis of the information given in the statement,
counsellors for regular conselling of
you have to assume everything in the statement to
students.
be true, then decide which of the suggested
courses of action logically follow(s) for pursuing. 25. StatementQuite a few cases of post
calamity false claims have been noticed by
Give Answer
the concerned Govt. authorities.
(A) if only I follows.
Courses of Action
(B) if only II follows.
I. Government should ensure stricter
(C) if either I or II follows.
verification of identity by the concerned
(D) if neither I nor II follows. officials.
(E) if both I and II follow. II. Government should deploy more number
21. StatementWith the arrival of monsoon, the of officials to handle the situation.
condition of roads in the city has begun to
deteriorate. Answers with Explanation
Courses of Action 1. (E) 2. (B) 3. (A) 4. (B) 5. (E)
I. The Road-Development Authorities 6. (A) 7. (C) 8. (D) 9. (B) 10. (B)
should be directed by the State Govt. to
11. (E) Both action I and II follows.
take up regular up-keeping of the roads.
12. (D) Neither I nor II action follows, because
II. People should be encouraged to use the
after rejecting the application, there is no
pooled/public transport by improving the
importance of objective criteria. And it is not
system to take care of their needs and
possible to open another school recently.
expectations.
13. (A) Only action I follows.
22. StatementDespite consistent economic
growth, our country has not been able to 14. (B) Only action II follows because it contains
eradicate poverty. cause to check the diseases with medical care.
Courses of Action 15. (E) To put both the actions successively is
I. The Government should focus their necessary.
attention towards the below poverty line 16. (A) Only the course of action I follows
strata of society. because parents can change their decisions
II. People in the high income group just knowing the circumstances. But the
should volunteer and contribute to course of action II cannot be done quickly. By
the Governments plan of poverty the way, if the course of action II is imposed
eradication. quickly the mentality of forth-coming parents
will be same also.
23. StatementIncidence of terror attacks are on
a constant rise since last few years. 17. (E) Both the actions follow, because
Government has rights to take the decision of
Courses of Action proper consumption of mangoes, which can
I. Strict control should be imposed on check the destruction of natural material.
immigrants across the states. Being the higher supply it is possible that the
II. Commuters should not be allowed to excess mangoes out of consumption may be
carry baggage in shared public transport. rotten. Therefore, both the courses of actions
24. StatementEvery year there is an increasing are legal.
number of students committing suicide due to 18. (B) The course of action II follows, because
fear of failure. giving faith of immediately initiate an
58 | Logical Reasoning & Analytical Ability

enquiry, it is possible to give the peace to the 22. (E) Both I and II follow. If the Government
mob. If the policeforce is used to disturb the make some employment plans for the people
mob, in such a position the mob may be more below the poverty line then their poverty can
infuriated of which result will not be good. be removed. If the people in the high income
19. (C) Either course of action I or II follows. If group, help with money volunteerly to plans
the cost are to be increased in the domestic made by Government, it be helpful in the
retail market, then it is not good to provide development of the poor.
subsidy to the oil marketing companies and if 23. (E) Both the actions follow. By strict control
the subsidy is to be provided to the marketing on immigrants, we should focus and observe
companies then the price should be remain each and every immigrant across the State.
constant in the retail market. Commuters should not be allowed to carry
baggage in shared public transport, because it
20. (A) The course of action I follows, because is be possible that they carry some bombastic
when the students were caught red handed things.
while using unfair means in the examinations 24. (E) Both I and II follow. The I.Q. of all
i.e., the students are in blame. Keeping the students are different. Hence, parents and
education level in view they should be teachers should not pressure on children for
punished by an education means, so that all high achievement otherwise the students
other students will change their mind not to commit suicide due to fear of failure. Besides
do so. The second course of action is not so school authorities should appoint counsellers
proper. for regular counselling of students.
21. (A) Only (I) follows because Road 25. (A) Only I follows. If the verification of
Development authorities are made by the identity of the concerning man is done
State Government to take care regular up- correctly then false claims can be ruled out. II
keeping of the roads. But II does not follow does not follow because there is no use to
because it is not possible to use public deploy more number of officials to handle the
transport to go everywhere. situation.


9 Decision Making
This chapter deals with questions in which (a) At (II) above but has secured at least 50
you have to decide upon the course of action to be per cent marks in post graduation and at
taken upon a candidate who has applied for a least 65 per cent marks in the selection
vacancy or allotment or membership to an process, the case is to be referred to
institution. It is an indispensable process in ones Executive Director.
personal and official life. A decision is the final (b) At (III) above but has at least twelve
outcome of the process of evaluating and years post qualification work experience
analysing the given data. Systematic analysis of out of which at least eight years as
facts and figures reduces the chances of Senior Manager-HR in an organization,
inaccuracy and ensures speedy decision-making. the case is to be referred to Chairman.
Students are required to match personal data of a In each question below details of one
candidate with the conditions given to select candidate are given. You have to take one of the
him/her to the job or admission. The decision courses of action based on the information
about each candidate has to be made from provided and the conditions and sub-conditions
amongst the five choices named (a), (b), (c), (d) given above and mark the number of that course
(e) which state the cources of action to be taken as of action as your answer. You are not to assume
per the candidates potentials. anything other than the information provided in
Example each question. All these cases are given to you as
Directions(Q. 110) Study the following on 1.5.2010.
information carefully and answer the questions Mark Answer
given below (A) If the candidate is to be selected
Following are the conditions for selecting (B) If the candidate is not to be selected
Assistant General Manager-HR in an organization. (C) If the data provided are not adequate to
The candidate must take a decision
I. Be at least 40 years and not more than 50 (D) If the case is to be referred to Executive
years old as on 1.5.2010. Director
II. Be post graduate in Personnel Manage- (E) If the case is to be referred to Chairman.
ment/HR with at least 60 per cent marks.
1. Pranab Ghosh was born on 8th March 1968.
III. Have post qualification work experience He has been working for the past eighteen
of at least fifteen years out of which at years in an organization out of which last
least five years as Senior Manager-HR in seven years as Senior Manager-HR after
an organization. completing his post graduation in HR with 68
IV. Have successfully implemented HR per cent marks. He had successfully
system in his/her organization some time implemented HR System last year in his
during the past three years. organization. He has secured 50 per cent
V. Have secured at least 45 per cent marks marks in the selection process.
in the selection process. 2. Sheetal Jha has been working in an organi-
In the case of a candidate who satisfies all the zation for the past twenty years out of which
conditions except at ten years as Senior Manager-HR after
60 | Logical Reasoning & Analytical Ability

completing her post-graduation in Personnel per cent marks. He has secured 60 per cent
Management with 70 per cent marks. She was marks in the selection process. He was born
born on 2nd December 1965. She has secured on August 16, 1965. He has successfully
45 per cent marks in the selection process. implemented HR-System in his organization
3. Prabir Sengupta was born on 8th May 1963. in past three years.
He has secured 65 per cent marks in the 9. Suneeta Govil was born on April 5, 1964. She
selection process. He has been working for has been working for the past seventeen years
the past fifteen years in an organization out of out of which nine years as Senior Manager-
which twelve years as Senior Manager-HR HR in an organization . She has secured 48
after completing his post graduation in HR per cent marks in the selection process. She
with 55 per cent marks. He has successfully has also secured 69 per cent marks in post
implemented HR system in his organization graduation in Personnel Management. She
during the last two years. has successfully implemented HR-System in
4. Shailesh Kumar has been working in an her organization last year.
organization for the past thirteen years out of 10. Mohit Saxena was born on July 27, 1963. He
which nine years as Senior Manager-HR after has been working for the past thirteen years
completing his Post graduation in HR with 68 out of which nine years as Senior Manager-
per cent marks. He was born on September HR after completing his post graduation in
15, 1968. He has secured 48 per cent marks in HR with 67 per cent marks. He has secured
the selection process. He has successfully 49 per cent marks in the Selection process. He
implemented HR-System in his organization has successfully implemented HR-System in
two years back. his organization during the past three years.
5. Navin Chopra was born on 12 June, 1967. He
has been working for the past sixteen years Answers with Explanation
out of which seven years as Senior Manager- 1. (A) Pranab Ghosh fulfils all the conditions so
HR after completing his post graduation in he is to be selected.
Personnel Management with 75 per cent 2. (C) Nothing is said for Sheetal Jha about the
marks. He has secured 44 per cent marks in condition IV. So the data provided are not
the selection process. He has successfully adequate to take a decision.
implemented HR-System in his organization 3. (D) Prabir Sengupta does not satisfy the
last year. condition II but he fulfils the condition (a).
6. Meera Kulkarni has been working for the past So his case is t o be referred to Executive
seventeen years out of which eight years as Director.
Senior Manager-HR after completing her post 4. (E) Shailesh Kumar does not satisfy the
graduation in Personnel Management with 66 condition III but he fulfils the condition (b).
per cent marks. She has successfully So his case is to be referred to Chairman.
implemented HR-System in her organization 5. (B) Navin Chopra does not satisfy the
during the last two years. She has secured 49 condition V. Hence he is not to be selected.
per cent marks in the selection process. She
was born on December 14, 1971. 6. (B) Meera Kulkarni is younger than 40 years.
Hence she is not to be selected.
7. Akash Shastri was born on April 12, 1967. He
has been working for the past sixteen years 7. (D) Akash Shastri does not satisfy the
out of which six years as Senior Manager-HR condition II but he fulfils the condition (a). So
in an organization after completing his post his case is to be referred to Executive
graduation in HR with 58 per cent marks. He Director.
has successfully implemented HR-System in 8. (A) Shekhar Jena fulfils all the conditions so
his organization last year. He has secured 65 he is to be selected.
per cent marks in the selection process. 9. (A) Suneeta Govil fulfills all the conditions so
8. Shekhar Jena has been working for the past she is to be selected.
fifteen years out of which last seven years as 10. (E) Mohit Saxena does not satisfy the condi-
Senior Manager-HR in an organization after tion III but he fulfils the condition (b). So his
completing his post graduation in HR with 68 case is to be referred to Chairman.
Logical Reasoning & Analytical Ability | 61

Exercise engineering firm. He doesnt want to sign the


bond for Rs. 50,000. He has cleared the
Directions(Q. 110) Study the following
selection test with 72% marks.
information carefully and answer the questions
given below. 4. Nishant is an Electronics Engineer passed out
in June 2002 at the age of 22 years. Since
An organization wants to recruit System
then he is working as a Programmer in a
Analysts. The following conditions apply.
software company. He has passed the
The candidate must selection test with 66% marks and is willing
(i) be an engineering graduate in Computer/ to sign the bond.
IT with atleast 60% marks. 5. Kalyani is an engineer with 72% marks in
(ii) have working experience in the field of Telecommunication. She has just completed
Computer atleast for 2 years after 27 years of age. She has cleared the selection
acquiring the requisite qualification. test with 59% marks. She is willing to sign
(iii) have completed minimum 25 years and the bond.
maximum 30 years of age as on 6. Sangita is an IT Engineer and is working as
1.12.2005. an EDP Officer in a bank for past 5 years.
In each question below, detailed information She has completed 28 years of age and is
of candidate is given. You have to carefully study willing to sign the bond of Rs. 50,000. She
the information provided in each case and take has obtained 65% marks in the selection test.
one of the following courses of actions based on 7. Abhijit is a Computer Engineer with 65%
the information and the conditions given above. marks passed out in 2001 at the age of 22
You are not to assume anything other than the years. Since then he is working as a Software
information provided in each question. All these Engineer in a private firm. He is willing to
cases are given to you as on 01.12.2005. You have sign the bond for the required amount. He
to indicate your decision by marking answers to has secured 63% marks in the selection test.
each question as follows. 8. Giridhar is working as a Software Engineer
Mark Answer in a reputed company for past 4 years after
(A) if the case is to be referred to VP. completing Computer Engineering with 68%
marks. He has cleared the selection test with
(B) if the case is to be referred to GM.
75% marks and is willing to sign the bond.
(C) if the data provided are not sufficient to His date of birth is December 17, 1978.
take a decision.
9. Nikita has just completed 26 years of age.
(D) if the candidate is to be selected. She has passed IT Engineering examination
(E) if the candidate is not to be selected. in 2002 with 66% marks and is working as a
1. Ms. Suneeta is an IT Engineer with 60% Sr. Programmer since then. She has no
marks at graduation as well as in selection problem in signing the bond of Rs. 50,000.
test. She is working as a Software Engineer She has secured 53% marks in the selection
for last 3 years after completing engineering test.
degree and has completed 27 years of age. 10. Mr. Rajan is working as a Programmer for
She is willing to sign the bond of Rs. 50,000. the last 6 years in an engineering firm after
2. Rakesh Rao is a Computer Engineer Graduate passing engineering examination. He has
and thereafter is working as a Software passed Electronics Engineering with 76%
Manager for past 6 years. He has secured marks. His date of birth is 16.05.78. He has
72% marks at graduation and 67% marks in cleared the selection test with 62% marks and
selection test. His date of birth is 5th is willing to sign the bond for Rs. 50,000.
December 1976. He is not willing to sign the Directions(Q. 1118) Study the following
bond for Rs. 50,000. information carefully and answer the questions
3. Ram Kumar is an Engineering Graduate in given below :
Computers with 78% marks passed out in Following are the conditions for admitting
1999 at the age of 23 years. Since then he is students for Graduate course in Architecture in a
working as a Software Manager in an college :
62 | Logical Reasoning & Analytical Ability

The student must : Physics, Chemistry and Mathematics in XII


(i) be atleast 17 years old as on 1.7.2006. Std. final examination.
(ii) have secured at last 50 per cent overall 13. Malini Sawant was born on 19th April, 1988.
marks in XII Std. final examination. She has secured 60 per cent marks in the
(iii) have secured at last 60 per cent aggregate entrance test. She can pay Rs. 60,000 at the
marks in Physics, Chemistry and Mathe- time of admission. She has secured 55 per
matics in XII Std. final examination. cent overall marks and 62 per cent average
(iv) have secured atleast 55 per cent marks in marks in Physics, Chemistry and
the entrance examination. Mathematics in XII Std. final examination.
(v) Pay Rs. 60,000 annual fee at the time of 14. Shikha Gupta has secured 60 per cent
admission. average marks in Physics, Chemistry and
If a student satisfies alone above conditions Mathematics and 58 per cent overall marks in
except XII Std. final examination. She was born on
(a) at (iii) above but as secured at least 70 14th February, 1988. She has secured 60 per
per cent overall marks in XII Std. final cent marks in the entrance test. She can pay
examination, his /her case is to be Rs. 38,000 at the time of admission and the
referred to Principal of the college. remaining amount within two weeks of
(b) at (v) above but can pay atleast 60 per
cent of the annual fee at the time of admission.
admission and the remaining amount 15. Samir Malhotra was born on 5th March,
within six months, his/her case is to be 1986. He can pay Rs. 60,000 at the time of
referred to Vice-Prinicpal of the college. admission. He has secured 65 per cent
In each of the questions given below, detailed average marks in Physics, Chemistry and
information about one student is given. You have Mathematics in XII Std. final examination.
to take one of the following courses of actions He has also secured 65 per cent marks in the
based on the information provided and the above entrance test.
conditions and sub-conditions. You are not to 16. Joseph DSilva has secured 62 per cent
assume anything other than the information average marks in Physics, Chemistry and
provided in each questions. Mathematics and 55 per cent overall marks in
All these cases are given to you as on XII Std. final examination. He was born on
1.7.2006. Now, mark your answer as 5th July, 1986. He can pay Rs. 60,000 at the
time of admission. He has secured 65 per
(A) if the data provided are not adequate to cent marks in the entrance test.
take a decision.
17. Sudha Ghosal was born on 8th August, 1989.
(B) if the student is to be omitted.
She can pay Rs. 60,000 at the time of
(C) if the case is to be referred to the
Principal. admission. She has secured 65 per cent
overall marks in XII Std. final examination
(D) if the student is not to be omitted.
and 55 per cent marks in Physics, Chemistry
(E) if the case is to be referred to Vice-
Principal. and Mathematics. She has also secured 75
per cent marks in the entrance test.
11. Anirban Mandal has passed his XII Std. final
examination with 75 per cent overall marks. 18. Mohan Awasthi was born on 25 October,
1980. He can pay Rs. 40,000 at the the time
He was born on 20th October 1988. He has of admission and the remaining amount
secured 56 per cent marks in the entrance test within two months. He has secured 65 per
and 62 per cent overall marks in Physics, cent marks in the entrance test. He has also
Chemistry and Mathematics in XII Std. final secured 60 per cent average marks in
examination. He can pay Rs. 40,000 at the Physics, Chemistry and Mathematics and 56
time of admission. per cent overall marks in XII Std. final
12. Arvind Gogoi has secured 75 per cent overall examination.
marks in XII Std. final examination. He has Directions(Q. 1926) For recruiting Agri-
secured 60 per cent marks in the entrance culture Officers in an organization, the following
test. He was born on 8th January 1987. He conditions apply. The candidate must
can pay Rs. 60,000 at the time of admission. (i) be an Agriculture Graduate with atleast
He has secured 58 per cent average marks in 65% marks.
Logical Reasoning & Analytical Ability | 63

(ii) clear the selection test with atleast 50% marks in the selection test and 52% marks in
marks. interview. He is willing to work in rural area.
(iii) secure atleast 45% marks in the 22. Nikhil, a young boy of 22 years has just
interview. passed B.Sc. Agriculture with 78% marks. He
(iv) be born on or after 1.11.1975 but not is a resident of a village in rural area of
after 1.11.1984.
Maharashtra state and would like to live in
(v) be willing to work in rural area.
rural area in future as well. He cleared the
If the candidate satisfies all of the above
selection test with 66% marks and the
mentioned conditions except
interview with 58% marks.
(a) at (i) above but has passed post 23. Raj Barua is working as an Agriculture
graduation in Agriculture, the case may Officer for past three years after completing
be referred to the Vice-President. B.Sc. Agriculture with 62% marks at the age
(b) at (iii) above but has secured 60% or of 23 years. He has cleared the selection test
more marks in selection test and has with 48% marks and interview with 47%
secured at least 40% marks in interview, marks. He is willing to work in rural area.
the case may be referred to the Chief 24. Avinash is 24 years of age and has passed
General Manager. graduation and post graduation in Agriculture
In each question below, detailed information with 67% and 69% marks respectively. He
of one candidate is provided. You have to read the has secured 70% marks in the selection test
information provided carefully and decide which and 42% marks in interview.
of the following courses of actions you will adopt 25. Ramesh is an Agri Graduate with 60% marks
as regarding status of the candidate based on the and has just completed post-graduation in
conditions stated above. You are not to assume Agriculture with 56% marks. He has secured
anything other than the information provided in with 62% and 58% marks in the selection test
each question. All these case are given as on and interview. His date of birth is 26.4.1982.
1.11.2005. Study the information of candidate in He has no problem in working in urban or
each question and decide the course of action. rural area.
Give Answer 26. Rajesh is an Agriculture graduate with 75%
(A) if the candidate is to be selected. marks passed out in 1996 at the age of 22
(B) if the case is to be referred to the Vice- years. Thereafter, he is working for last 7
years as Agriculture Assistant. He has secured
President. more than 55% marks in selection test as well
(C) if the case is to be referred to the Chief as interview. He prefers to work in rural area.
General Manager. Directions(Q. 2736) Following are the
(D) if the candidate is not to be selected. criteria laid down for the selection of Jr.
(E) if the information in the question is not Executives in an Organisation.
adequate to take the deicsion. The candidate must
19. Sahil has passed B.Sc. Agriculture in 2003 at (i) have scored atleast 70% marks in SSC
the age of 21 years with 68% marks. He is examination.
doing post-graduation in Agriculture and will (ii) have obtained graduation degree in any
complete in May 2006. He has secured 58% discipline with atleast 65% marks and
marks in selection test and has secured 50% first class.
marks in the interview. He is willing to work (iii) have cleared the selection test with at
in rural area. least 55% marks.
(iv) have cleared the interview with atleast
20. Milind is an Agriculture Graduate with 70% 50% marks.
marks. His date of birth is 11.7.1984. He (v) be not less than 21 years of age and not
scored 68% marks in the selection test and more than 28 years of age as on
42%, marks in the interview. He is willing to 1.9.2008.
work in rural area. (vi) have completed atleast a certificate /
21. Pravin is a post-graduate in Agriculture with diploma course in Computers.
58% marks. He is working for last 2 years However, if a candidate statisfies all the
after completing post-graduation at the age of above mentioned criteria except
25 years. He had scored 56% marks at (a) at (ii) above, but has completed post-
graduation in Agriculture. He secured 60% graduation in any discipline; the case
64 | Logical Reasoning & Analytical Ability

may be referred to the Vice-President of interview with 61% marks. He has compe-
the company. leted 23 years of age on 21.2.2008.
(b) at (iii) above, but has secured atleast 31. Ankita is a Commerce Graduate passed in
50% marks in the selection test and first class with 68% marks. She has completed
atleast 60% marks in the interview, the a two years diploma course in Computers after
case may be referred to the General graduation. Her SSC score was 85%. She has
Manager (Recruitment) of the company. cleared the selection test with 64% marks and
In each of the questions below, information of interview with 58% marks. Her date of birth
one candidate is given. You have to take one of is 27.8.1985.
the following five decisions based on the informa- 32. Manisha has scored 73% marks in SSC and
tion provided and the criteria and conditions given 62% marks in graduation. After graduation
above. You are not to assume anything other than she is working in a private firm. She has
the information provided in each question. All cleared the selection test with 58% marks and
these cases are given to you as on 1.09.2008. You interview with 62% marks. Her date of birth
have to indicate your decision by marking answers is 11.5.1983. She has completed a certificate
to each question as follows. course in Computers.
Mark Answer 33. Nikita is a graduate in Engineering passed in
(A) if the candidate is to be selected. first class wtih 72% marks. She has cleared
(B) if the candidate is not to be selected. the selection test and interview with 63% and
(C) if the data are inadequate to take a 65% marks respectively. She has done one
decision. year diploma course in Computers. Her date
(D) if the case is to be referred to the Vice of birth is 16.7.1984.
President of the company. 34. Jyotika Basu is B.Sc. passed out in 2004 in
(E) if the case is to be referred to the General first class with 68% marks. She scored 67%
Manager (Recruitment) of the company. marks in SSC. She is pursuing M.Sc. at
27. Shamika is a Science Graduate passed in first present. She has cleared the selection test and
class with 69% marks. She had passed SSC interview with 56% and 53% marks
exam securing 83% marks. She has cleared respectively. She has completed a certificate
the selection test with 53% marks and course in Computers. Her date of birth is
interview with 63% marks. Her date of birth 15.5.1985.
is 1.8.1983. She has completed a six months 35. Nandan Savla is a B.Com. passed in first
certificate course in computers. class with 66% marks. He had secured 75%
28. Vivek is presently pursuing post-graduation marks in HSC. He has completed 25 years of
in Economics. He has passed graduation in age of July 25, 2008. He has cleared the
first class with 72% marks. He had scored selection test and interview each with 56%
78% marks in SSC. His date of birth is marks. He has completed one year certificate
21.8.1985. He has secured 58% marks in the course in Computers in grade A.
selection test and 62% marks in interview. He 36. Rajesh Chavan is an Arts Graduate passed in
has passed a 6 months diploma course in first class with 61% marks. He has also
Computers. completed M.A. in English with 62% marks.
29. Abhijit is a graduate with 60% marks and a His SSC score was 78%. He has cleared the
post-graduate with 65% marks. He has selection test with 56% and interview with
completed one year diploma course in 54% marks. His date of birth is 25.3.1984. He
Computers. He had scored 85% marks in has completed a diploma course in Computers.
SSC. He has cleared the selection test and Answers with Explanation
interview with 58% and 56% marks 1. (D) Ms. Suneeta fulfils all the conditions.
respectively. His date of birth is 17.5.1986. Hence, she is to be selected.
30. Sujay Chaudhary has passed SSC with 2. (A) Rakesh Rao fulfils all the conditions
distinction and 81% marks and B. Com. in except (iv) but he has been working as a
first class with 68% marks. He has completed software manager for the last 6 years so
a certificate course in Computers. He has according to (b) his case is to be referred to
cleared the selection test with 51% marks and V.P.
Logical Reasoning & Analytical Ability | 65

3. (A) Ram Kumar fulfils all the conditions within two months. Therefore, his case is to
except (iv) but he has been working as a be referred to the Vice-Principal.
software manager for the last 6 years so 19. (A) Sahil satisfies all the conditions, hence he
according to (b) his case is to be referred to is to be selected.
V.P. 20. (C) Milind does not satisfy the condition (iii).
4. (C) Since the marks obtained by Nishant in But according to (b) his case is to referred to
graduation are not given. So, the data are not the Chief-General Manager.
sufficient to take a decision. 21. (B) Pravin does not satisfy the condition (i)
5. (E) Kalyani is a Telecommunication Engineer. but according to (a) his case is to be referred
Hence, she is not to be selected. to the Vice-President.
6. (C) The marks of graduation are not given. 22. (A) Nikhil satisfies all the conditions so he is
So, data are inadequate to take the decision. to be selected.
7. (D) Abhijit fulfils all the conditions. Hence, 23. (D) Raj Barua does not satisfy the conditions
he is to be selected. (i) and (ii). So, he is not to be selected.
8. (D) Girdhar fulfils all the conditions. Hence, 24. (E) Information in the question is not
he is to be selected. adequate to take decision.
9. (E) Nikita does not fulfil the condition V. 25. (B) Ramesh does not satisfy the condition (i)
Hence, she is not to be selected. but according to (a) his case is to be referred
10. (D) Mr. Rajan does not fulfil the condition (i) to the Vice-President.
But he passed the Electronics Engineering 26. (D) Rajesh does not satisfy the condition (iv)
more than 65%. Hence, according to (a) his so he is not to be selected.
case is to be referred to G.M. 27. (E) Shamika does not fulfil the condition (iii).
11. (A) Anirban Mandal satisfies all the Hence, according to (b) her case is to referred
conditions except (V). But he can pay 60% of to the General Manager (Recruitment officer
the annual fee, but he will he pay the rest is of the company).
unknown. Therefore, the given data are not 28. (A) Vivek fulfils all the conditions. So he is
sufficient to take a decision. to be selected.
12. (C) Arvind Gogoi does not satisfy the 29. (D) Abhijit does not fulfil the condition (ii)
condition (iii), but he has secured 75 per cent but he has completed post-graduation. Hence,
marks in XII Std. final examination. his case will be referred to the Vice President.
Therefore, his case is to be referred to the 30. (E) Sujay Chaudhary does not fulfil the
principal. condition (iii). But he has secured more than
13. (B) Malini Sawant satisfies all of the given 50% marks in selection and more than 60% in
conditions. Therefore, she is to be admitted. interview. So, his case is to be referred to the
14. (E) Shikha Gupta does not fulfil the condition General Manager.
(v), but she can pay 60 per cent of the annual 31. (A) Ankita fulfils all the conditions. So, she is
fee and she can also pay the rest fee within six to be selected.
months. Therefore, her case should be 32. (B) Manisha does not fulfil the condition (ii)
referred to the Vice-Principal. and she has not completed Post-graduation.
15. (A) Samir Malhotra has not shown the marks Hence, she is not to be selected.
of XII Std. final examination. Therefore, the 33. (C) Nothing is said about Nikitas S.S.C.
provided data are not adequate to take a Hence, the data is inadequate to take a
decision. decision.
16. (B) Joseph DSilva satisfies all the given 34. (B) Jyotika Basu does not fulfil the condition
conditions. Therefore, he is to be admitted. (i). She is not to be selected.
17. (D) The age of Sudha Ghosal is less than 35. (C) Nothing is said about S.S.C. of Nandan
required age. Therefore, she is not to be Savla. Hence, the data is adequate to take a
admitted. decision.
18. (E) Mohan Awasthi satisfies all the conditions 36. (D) Rajesh Chavan does not fulfil the
except (v) and he can pay more than 60 per condition (ii). But he has completed post-
cent of annual fee at the time of admission graduation. So, his case is to be referred to the
and he can pay the rest amount of annual fee Vice President.

10 Cause and Effect
This types of question are frequently asked (E) If both the statements I and II are effects
these days usually. In such type of questions the of some common cause.
candidates are asked to determine whether a given 1. I. This year, the cut off percentage for
event is the cause or the effect of some other admission to junior colleges have
event. These questions are to ascertain the increased over the last year.
analytical and logical reasoning ability of the II. This year performance of students in Xth
aspirants. final exam was considerably higher than
Relation between cause and effectEvery that of the previous year.
event hapens because it has a cause behind it. This 2. I. The conditions of most of the national
cause is the condition under which any event highways are very bad.
happens. II. Govt. has now sanctioned a huge amount
In our day to day life we may come across of money to maintain the national
many situations which are more subjective and highways.
require an initiative skill. It is not necessary all the
time can a cause be so scientifically accurate and 3. I. Many students of the local school have
precise. Thus, we can say that a necessary failed in English Language paper in the
condition for the occurrence of a specific event is annual examination.
a circumstance in whose absence for event can not II. Many students of the local school have
occur or a necessary candition for occurrence of a failed in Mathematics paper in the annual
specific event is a circumstance in whose presence examination.
the event must occur. 4. I. Rain and thunder showers bashed the
city during the past three days.
Example
II. Many people stayed indoor during the
Directions(Q. 15) Below in each question past three days.
is given two statements I and II. These statements 5. I. There has been a considerable increase in
may be either independent causes or may be the sale of fat free food articles.
effects of independent causes or a common cause.
One of these statements may be the effect of the II. Now, people have become more
other statement. Read both the statements and conscious about their health condition
decide which of the following answer choices and food habits.
correctly depicts the relationship between these Answers with Explanation
two statements. 1. (D) Both the statements are effects of
Mark Answer independent causes they do not have any
(A) If statement I is the cause and statement relation between them.
II is its effect. 2. (A) In the I statement, the cause is given and
(B) If statement II is the cause and statement in II statement, the effect it is given.
I is effect. 3. (E) There may be a common cause for both
(C) If both the statements I and II are the statements.
independent causes. 4. (A) II is effect of the I statement. It is only
(D) If both the statements I and II are effects reason that many people stayed indoor during
of independent causes. the past three days.
Logical Reasoning & Analytical Ability | 67

5. (B) Statement II is the cause and Statement I 5. Statements


shows the effect. It is clearly mentioned in I. A recent tiger census in the tiger reserve
Statement I that there has been a considerable in the state has reported significant
increase in the sale of fat free food article. reduction over the last census.
II. The Govt. has initiated an enquiry to
Type I ascertain the facts relating the tiger
Exercise population in the state.
Directions(Q. 15) Below in each question Directions(Q. 610) Below in each ques-
are given two statements (I) and (II). These tion are given two statements (I) and (II). These
statements may be either independent causes or statements may be either independent causes or
may be effects of independent causes or a common may be effects of independent causes or a
cause. One of these statements may be the effect common cause. One of these statements may be
of the other statement. Read both the statements the effect of the other statement. Read both the
and decide which of the following answer choice statements and decide which of the following
correctly depicts the relationship between these answer choice correctly depicts the relationship
two statements. Now, mark your answer as between these two statements.
(A) If statement (I) is the cause and statement Mark Answer
(II) is its effect. (A) If statement (I) is the cause and statement
(II) is its effect.
(B) If statement (II) is the cause and (B) If statement (II) is the cause and
statement (I) is its effect. statement (I) is its effect.
(C) If both the statements (I) and (II) are (C) If both the statements (I) and (II) are
independent causes. independent causes.
(D) If both the statements (I) and (II) are (D) If both the statements (I) and (II) are
effect independent causes. effects of independent causes.
(E) If both the statements (I) and (II) are (E) If both the statements (I) and (II) are
effects of some common cause. effects of some common cause.
1. Statements 6. I. Government of state X imposed a
I. Majority of the residents of the housing higher sales tax on petroleum products
society participated in the dinner hosted compared to the neighbouring states.
by one of the members of the society. II. All the petrol pumps in the state
II. Most of the people living in the housing observed bandh in protest.
society invite other members for the 7. I. Attendance for the All India examination
functions at their house. held in July 2006 was poor at all the
2. Statements centres.
I. Many shops in the locality remained II. Western States of the country were
closed throughout the day. affected by heavy floods during July
II. Many offices in the locality closed 2006.
during the day. 8. I. High Court has declared stay on
3. Statements construction of residential buildings on
I. The school authority has decided to the land under dispute.
increase tuition fees by 30 per cent from II. A large number of middle class salaries
the next academic year. people had booked flats in the buildings
II. The Govt. has urged the local public to under dispute.
enroll all their children to schools in the 9. I. Residents in the locality have now
area. decided to launch a cleanlines drive.
4. Statements II. Civic authorities in the city have
recorded many cases of Cholera and
I. The train services in the suburban areas gastro.
of the city were disrupted for four hours. 10. I. Senior citizens of the city have
II. The overhead electrical wire snapped complained about the late night
between two stations in suburban area of disturbance caused due to loudspeakers
the city. used during festivals.
68 | Logical Reasoning & Analytical Ability

II. Though, the Government has issued a


directive banning late night celebrations
Type II
involving use of loudspeakers, it is not Exercise
being strictly followed in some of the 1. CauseAll the major rivers in the state have
areas. been flowing way over the danger level for
Directions(Q. 1115) Below in each ques- the past few weeks.
tion are given two statements I and II. These Which of the following is / are possible
statements may be either independent causes or effect(s) of the above cause ?
may be effects of independent causes or a (1) Many villages situated near the river-
common cause. One of these statements may be banks are sub-merged forcing residents
the effect of the other statement. Read both the to flee.
statements and decide which of the following
(2) Govt. has decided to provide alternate
answer choice correctly depicts the relationship
shelter to all the affected villagers
between these two statements.
residing near the river banks.
Mark Answer
(A) if statement (I) is the cause and statement (3) The entire state has been put on high
(II) is its effect. flood alert.
(B) if statement (II) is the cause and (A) Only (1)
statement (I) is its effect. (B) Only (1) and (2)
(C) if both the statements (I) and (II) are (C) Only (2) and (3)
independent causes. (D) All (1), (2) and (3)
(D) if both the statements (I) and (II) are (E) None of these
effects of independent causes. 2. EffectThis year majority of the final year
(E) if both the statements (I) and (II) are students of the management institute have
effects of some common cause. opted for finance specialization.
11. I. Many people in the area are reported to Which of the following can be a probable
be suffering from Malaria. cause of the above effect ?
II. Private Medical Practitioners in the area (A) Last year most of the students with HR
have decided to close their clinics for specialization got better job offers than
few days. other specializations.
12. I. The State Govt. has announced special
(B) The management instiute offers only
tax package for the new industries to be
finance specialization to its final year
set up in the state.
students.
II. Last year the State Govt. had hiked the
taxes for all industrial activities in the (C) Last year the students with finance
State. specialization bagged most of the
13. I. The vegatable prices in the local market lucrative offers vis-a-vis students with
have increased manifold during the past other specializations.
few days. (D) The management institute has recently
II. Incessant rains have created flood like started its finance specialization in
situation in most rural parts of the state. addition to Marketing and HR being
14. I. Police authority has recently increased offered earlir.
vigil during the evening hours in the (E) None of these
locality. 3. EffectGovt. has allowed all the airlines to
II. There has been considerable reduction in charge additional amount as peak time
the incidents of petty crimes in the congestion charges for the flights landing
locality. between 6.00 a.m. to 10.00 a.m.
15. I. There was a huge rush of people to the Which of the following is a probable cause of
temple last Sunday the 15th of the
month. the above effect ?
II. The temple authority had decided to (A) All the airline companies had threatened
close down the temple for repairs from to suspend their services during peak
17th of the month. hours.
Logical Reasoning & Analytical Ability | 69

(B) The Govt. has increased its tax for peak 2. (D) Both the statements (A) and (B) are
time flights. effects of some independent causes because
(C) The aircrafts are routinely put on hold the shops and offices in the locality were
over the airports while landing during closed due to other causes.
peak time causing extra fuel consump- 3. (E) Both the statements (A) and (B) are
tion. effects of some common causes because the
(D) The airlines companies can now charge urge of government for the enroll and the
unlimited additional charge for peak time decision to increase tuition fee, together are
flights. the effects of common causes.
(E) None of these 4. (B) Statement (B) is cause and statement (A)
4. CauseThe cement manufacturing com- is its effects because on account of snapping
panies have increased the price of cement by of overhead electrical wire there was a result
about fifteen per cent with immediate effect. of disruption of train services.
Which of the following is/are possible 5. (C) These both statements are independent
effects(s) of the above cause ? causes.
(1) Govt. will direct the cement manufac- 6. (C) 7. (A) 8. (D) 9. (E) 10. (B)
turing companies to reduce the price 11. (D) Both the statement I and II are effects of
increase to five per cent. independent causes. Because spreading
(2) The prices of residential flats and malaria or to be suffering from malaria may
commercial companies will see an because mosquitoes and dirtiness. But the
upward trend. decision of private medical practitioners to
(3) The construction companies may stop all close the clinics may be other causes.
on going construction projects with 12. (B) Because due to hike of taxes last year the
immediate effect. State Government has announced special tax
(A) Only (2) (B) Only (1) package for the new industries to be set up in
(C) Only (3) (D) Only (2) and (3) the state.
(E) None of these 13. (B) The transportation get effect due to the
5. EffectThere has been unprecedented flood created by incessant rains in the most
increase in the number of institutions training rural part of the state, therefore, on account of
spoken phonetic English in all the major cities this, it is possible to increase the vegetable
of India during the last few years. prices in the local market.
Which of the following can be a probable 14. (A) Statement I is the cause and statement II
cause of the above effect ? is its effect.
(A) Many parents want their children to 15. (B)
speak fluent English. Answers with Explanation
(B) Various activities being outsourced to Type II
India by many European and North
American countries. 1. (C) Only III follows. After finding the cause
(C) English is no longer being taught in the of the sickness, necessary action should be
schools and colleges in India. taken.
(D) India has highest number of English 2. (E) To improve the standard in English
speaking educated youth compared to Language, all the courses of action are
any other country. necessary.
(E) None of these 3. (D) In order to save the people from getting
drowned all the courses of action are
Answers with Explanation necessary.
Type I 4. (A) Such institutes should immediately be
1. (B) Statement (B) is the cause and statement derecognised by the university. Hence, I
(A) is its effect because the people course of action follows.
participated in the dinner on account of the 5. (B) Only II follows because providing relief
invitation. supplies to the affected areas is necessary.

11 Data Sufficiency
In this type of a question followed by two or 1. Among M, N, T, Q and D, who is the
more information is given. Questions are designed youngest ?
to test candidates ability to relate given I. T and D are younger than M.
information with that of required one based on the II. Q is older than T, but younger than D
question asked. The questions consist of any topic and N.
such Coding-Decoding, Puzzle Test, Blood 2. How is steel written in a code language ?
Relations, Mathematical calculation etc. The I. steel container more costly is written as
candidates is required to find out which of the ho na pa da in that code language.
given statements is/are sufficient to answer the II. buy more steel vessels is written as na
given question. Sometimes single information ka ta da in that code language.
is sufficient to answer the question though
3. How many sons does K have ?
other information may also include relevant
information. In such case that triangle information I. M and T are brothers of D.
is our answer. Sometimes single information is not II. D is the only daughter of N and K.
sufficient to answer the question a combination of 4. How is M related to P ?
information, which can help to answer the I. Ms sister is married to R.
question, will be one answer. II. Rs brother is married to Ps sister.
Example 5. Who among Naveen, Mohan, Prakash and
Directions(Q. 15) Each of the questions Kishore reached office first ?
below consists of a question and two statements I. Mohan reached office before Naveen and
numbered I and II given below it. You have to Kishore, but was not the first to each
decide whether the data provided in the statements office.
are sufficient to answer the question. Read both II. Kishore reached office after Mohan, but
the statements and give the answer before Naveen.
(A) If the data in statement I alone are sufficient Answers with Explanation
to answer the question, while the data in
1. (C) According to the question,
statement II alone are not sufficient to answer
From statement I.
the question.
M > T/D (i)
(B) If the data in statement II alone are sufficient and from statement II
to answer the question, while the data in
D/N > Q> T (ii)
statement I alone are not sufficient to answer
the question. Data given in both the staements
(C) If the data either in statement I alone or in I and II together are not sufficient to answer
statement II alone are sufficient to answer the the question.
question. 2. (D) According to the question,
(D) If the data given in both the statements I and From statement I.
II together are not sufficient to answer the steel container more costly ho na pa da.
question, and From II.
(E) If the data in both the statements I and II Buy more steel vessels na ka ta da.
together are necessary to answer the From both I and II.
question. steel more na da
Logical Reasoning & Analytical Ability | 71

Therefore, data in both the statement I and II (C) If the data either in statement I alone or in
together are not sufficient to answer the statement II alone is sufficient to answer the
question. question.
(D) If the data in both the statement I and II
3. (E) According to question, together are not sufficient to answer the
rother T question.
Brother M B (E) If the data in both the statements I and II
D
together are necessary to answer the
ter
Da
gh

question.
ug
u

ht e
Da

1. How is M related to F ?
r

K N Statements
Husband I. F is sister of N who is mother of R.
Wife II. M has brothers of which son is R.
K has two sons M and T. Therefore, to 2. On which date in March was Pravins fathers
answer the question data given in both the birthday ?
statements I and II together are necessary. Statements
4. (E) According to the question, I. Pravin correctly remembers that his
Husband fathers birthday is after 14th but before
Sister RB 19th March.
M ro
th II. Pravins sister correctly remembers that
er
their fathers birthday is after 17th but
before 21st March.
d 3. Among M, N, T, R and D each having
P ban
Sister Hus different age who is the yongest ?
Statements
Therefore, M is the brother of P, Hence, I. N is younger than only D among them.
data in both the statements, I and II are II. T is older than R and younger than M.
necessary to answer the question. 4. Village D is which direction of village H ?
5. (A) According to the question, Statements
From statement I. I. Village H is to the South of village A
which is to the South-East of village D.
Prakash > Mohan > Naveen > Kishore
II. Village M is to the East of village D and
Therefore, data in statement I alone are to the North-East of village H.
sufficient to answer the question.
5. How is food written in a code language ?
While the data in statement II alone are not
Statements
sufficient to answer the question.
I. Always eat good food is written as ha
Exercise na pa ta in that code language.
Directions(Q. 12) In the following II. Enjoy eating good food is written as ni
questions each of the questions below consists of a ha ja pa in that code language.
question and two statements numbered I and II 6. F is in which direction with respect to M ?
given below it. You have to decide whether the Statements
data provided in the statements are sufficient to I. F is to the West of Q which is to the
answer the question. Read both the statements and West of M.
give answer II. M is to the East of Q, F, M and Q are on
(A) If the data in statement I alone is sufficient to the straight line.
answer the question, while the data in 7. What is Samirs rank from the top in the class
statement II alone is not sufficient to answer of 30 students ?
the question. Statements
(B) If the data in statement II alone is sufficient I. Sudhir, who is four ranks above Samir, is
to answer the question, while the data in fifteenth in rank from the bottom.
statement I alone is not sufficient to answer II. Samir is three ranks below Neeta who is
the question. eighteenth from the bottom.
72 | Logical Reasoning & Analytical Ability

8. In the code language what is the code for II. come home later is written as pa ta ho
fat ? in that code language.
Statements 17. What is Samirs rank from the top in class of
I. In the code language she is fat is 40 students ?
written as he ra ca. Statements
II. In the same code language fat boy is I. Nikhil who is five ranks below Samir is
written as ra ka. tenth from the bottom.
9. How many children are there in the group ? II. Suresh who is six ranks above Samir is
Statements twentieth from the top.
I. Sangita has scored more marks than 12 18. Who among P, Q, R, T and W is the oldest ?
children in the group. Statements
II. Reena has scored less than Sangita. I. R and T are older than P and W.
10. Who among L, N, F, G and Q was the first to II. Q is older than P but younger than R.
reach the college ? 19. What is Ts position with respect to W when
Statements T, W, R, J and M are sitting around a circle
facing at the center ?
I. F reached before L and G but not before
Q who was not the first to reach. Statements
I. R is second to the left of M and second
II. N reached before F and G and L reached
to the right of W.
after F.
II. J is not an immediate neighbour of M.
11. How is Nikhil related to Rama ?
20. How is come written in a code language ?
Statements
Statements
I. Nikhil is the only grandson of Ramas I. come and go is written as pit ka ja in
father-in-law. that code language.
II. Rama has no siblings. II. go and tell is written as ja ma ka in
12. What is the value of 36$48? that code language.
Statements 21. How is P related to N ?
I. P$Q means divide P by Q. Statements
II. AB means multiply A by B. I. Ps mother is sister of Ns father.
13. On which day of the week was Joseph born ? II. Ns uncle is grandfather of Ps brother.
Statements 22. On which day of the week did Mohan visit
I. Josephs birthday was one day after his Bangalore ?
sisters birthday. Statements
II. Joseph was born on the third day of the I. Mohan took leave on Wednesday.
week. II. Mohan visited his brother on Friday the
14. M, P, D, K and R are sitting around a circle previous day of his visit to Bangalore.
facing at the centre. Who is to the immediate 23. What is the code for health in that code
right of P ? language.
Statements Statements
I. D is sitting between M and R. I. In the code language go for walk is
II. K is not to the immediate left of R. written as he ta ja.
15. How is D related to M ? II. In the same code language morning
Statements walk improves health is written as pa ra
I. D has two sisters K and N. ta ko.
II. Ks father is brother of M. 24. How is Shubhada related to Ravindra ?
16. How is go written in a certain code lan- Statements
guage ? I. Shubhadas mother is the only daughter
Statements of Ravindras father.
I. go home now is written as ja ho na in II. Shubhada is the only grand-daughter of
that code langauge. Ravindras father.
Logical Reasoning & Analytical Ability | 73

25. H is in which direction with respect to L ? 6. (A) From I,


Statements N
I. L is to the East of M which is to the W E
North of H. F Q M
II. L is to the North of J which is to the S
East of H. F is in west of M.
26. Who was the first person to board the train From II,
among P, Q, R, S and T ? Q M
Statements
The answer is from only I.
I. Q boarded after P and T but not after S.
7. (C) From I, + 14
II. R was not the last person to board.
27. How many children are there in the group ?
Statements Sudhir Samir
I. In descending order of height, Radha is Samirs rank from the top
tenth from the top in the group. = 30 15 + 1
II. In ascending order of height Radha is = 16th
20th from the top in the group. From II, + 17
Answers with Explanation Neeta Samir
Sister N Mother R Neetas rank from the top
1. (E) F
= (30 17) = 13th
Brother

Samirs rank from the top


= 13 + 3 = 16th
M The answer of the question can be obtained
F is the maternal aunt of M. from only I or from only II.
2. (E) From I, His fathers birthday is on 15 or 8. (E) From I,
16 or 17 or 18th March. she is fat he ra ca
From II, His fathers birthday is on 18th or From II,
19th or 20th March. fat boy ra ka
Fathers birthday is on 18th March. From I and II,
3. (E) From I, D > N > T, R, M Fat ra
From II, M>T>R 9. (D)
The youngest is R.
10. (A) From I, N > Q > F > L and G
4. (C) From I, D From I, N reached the college first.
A From II, N > F and G
and F > L
H Hence, answer is obtained only from I.
11. (A) From I,
D is in N-W of H.
Only grandson of father-in-law
From II, N
D
M W E
S
Nikhil Rama
H Hence, Nikhil is the son of Rama From II, the
D is in N-W of H. answer of question is not obtained.
5. (D) From I, Always eat good food ha na pa 12. (E) From I,
ta 3 6 $ 4 36 4 = 9
From II, Enjoy eating good food ni ha ja From II, 6 8 9 8 = 72
pa From I and II together,
Code for food cannot be known. 3 6 $ 4 8 = 72
74 | Logical Reasoning & Analytical Ability

13. (B) From II Joseph was born on Tuesday 21. (C) From I,
because third day of the week is P Mother
Tuesday.
14. (D) Sister
15. (D) From I and II, Father
D
r
ste
Si

Si N
st e
r

K Father N P is cousin of N.
Fa

From II,
th
er

M
Brother Uncle
N
As D may be female or male.
Grandfather
Hence, the relation between D and M cannot
be known.
16. (D) From I, P Brother
go home now ja ho na P is niece or nephew of N.
From II, 22. (B) From II, Mohan visited his brother on
come home later pa ta ho Friday. Previous day of his visit to Bangalore.
go ja or na Thus, Mohan visited Bangalore as Saturday.
17. (C) +9 23. (D) From I and II
Samir Nihil ta walk
From I, rank of Samir from the top The data in both I and II are not sufficient
= 40 14 = 26th to answer the question.
From II, Samir rank from the top 24. (C) From I Shubhada is the niece of Ravindra.
= 6 + 19 + 1 From II Shubhada is the daughter or niece of
= 26th Ravindra.
19 + 25. (C) From I M L N
Suresh Samir
W E
18. (D) From I, (R, T) > (P, W)
From II, R > Q>P S
H
19. (E)
R From II L
T J
H J
H is in South-West with respect to L.
Right

Left

26. (D) From I : (P, T), Q, S


M W From II : R was not the last person to board.
Hence, the data in both the statements I and II
together are not sufficient to answer the
Hence, T is second to the left of W. question.
20. (E) From I, 27. (E) From I : In descending order of height.
come and go pit ka ja Radha is 10th from the top in the group.
From II, From II : In descending order of height Radha
go and tell ja ma ka is 20th from the top.
From I and II, Total No. of Children = 9 + 1 + 19
come pit = 29

Analytical Reasoning or
12 Problem Solving
Question based upon analytical reasoning are (C) F and C (D) B and D
undoubtedly the longest and most time consuming (E) None of these
of all types of question that can be asked in
Some important steps to solve such type of
various examinations. However, their importance
questions.
in exams cannot be underestimated as these
questions decide merit of a Test. Read the question carefully by keeping in
ExampleRead the following information mind every point or detail. It will take a few
carefully and answer the questions that follow. seconds. After reading the question, you will get
the general idea as to what the general theme of
There are six cities A, B, C, D, E and F. the problem is. Determine the usefulness of each
A is not a hill station. of the information and classify all the information
B and E are not historical places. into (A) Actual information (B) Useful secondary
D is not an industrial city. information and finally (C) Negative information.
A and D are not historical places. Refer to example given below :
A and B are not alike. Detailed Explanation
1. Which two cities are industrial centres ? Step IThe informations can be analysed in
(A) A and B (B) E and F the following manner.
(C) C and D (D) B and F A B C D E F G
(E) A and D Historical
Industrial
2. Which two cities are historical places ? Hill Station
(A) A and C (B) B and F
Step IIIn above mentioned example, we
(C) C and F (D) B and E see that the first and last sentences have useful
(E) A and D secondary information whereas all the remaining
3. Which two cities are hill stations ? have negative information. By studying the
second, third, fourth and fifth sentences we put
(A) A and B (B) C and A mark.
(C) B and D (D) A and F A B C D E F
(E) None of these Historical
4. Which city is a hill station and an industrial Industrial
centre but not a historical place ?
Hill Station
(A) E (B) F
(C) A (D) B We can observe that above table gives some
(E) C definite information about A and D. When A is
neither a historical city nor a hill station. It is,
5. Which two cities are neither historical places therefore, an industrial city. In the same way D is
nor industrial centres ? neither historical nor industrial so it must be a hill
(A) A and B (B) D and E station. We can mark at the appropriate places.
76 | Logical Reasoning & Analytical Ability

Step IIIFinally we come to useful Now, our final Table is as below :


secondary information and negative information. A B C D E F
For instance, A and B are not alike. This means Historical
that B is not an industrial city. As B is not a Industrial

historical place. So, it must be hill station. Now,
Hill Station
all the remaining information are negative. A lack
of negative information about a city would mean On the basis of above table, the answers can
that the city does have the quality whose absence be drawn.
is not mentioned. 1. (B) 2. (C) 3. (C) 4. (A) 5. (D)
The problems can be classified into the
Here in the example, there are no negative following (1) Classification or categorization (2)
information about C and F and partly about E. We Question based on blood relation (3) Seating and
can assure that C and F are hill station, industrial placing arrangements (4) Comparision type test
and historical cities and E is industrial as well as (5) Sequential order of things (6) Selection based
hill station. on certain given pre-conditions.

1. Classification or Categorization
In this type of test, the questions are asked Example 2. Six students P, Q, R, S, T and U
based upon the given data belonging to different are studying different subjects for their Master
groups or possessing different qualities. You Degree. The subjects are English, History,
would be supposed to analyse these data by Philosophy, Physics, Statistics and Mathematics.
placing the different items or concepts in different 1. In these students, two live in hostel, two are
categories with the help of given information. paying guests and remaining two are living in
These problems can be easily solved by their home.
constructing a proper table or chart.
2. R is not a paying guest and study
Example 1. Six friends P, Q, R, S, T and U Philosophy.
are the members of a club. They play Football,
Cricket, Tennis, Basketball, Badmintion and 3. The students who are studying History and
Volleyball. Statistics are not paying guests.
1. T, who is taller than P and S, plays football. 4. T and S study Mathematics and Statistics
2. The tallest member plays basketball. respectively.
3. The shortest member plays volleyball. 5. U and S live in the hostel.
4. Q and S neither play volleyball nor play 6. T is a paying guest and Q lives in his home.
basketball. Find out who is in study of English ?
5. R plays volleyball. (A) S (B) T
6. According to height T is in between Q, who
(C) U (D) R
plays football, and P is shorter than T.
Who plays Basketball ? (E) None of these
(A) Q (B) R Answer with ExplanationWriting the
(C) S (D) U information on a chart.
(E) None of these Name of Subjects Residence
Answer with ExplanationWriting these the students
information on a chart. P English Paying guest
Member Name of the game Height Q History or Statistics Home
P Shorter than T R Philosophy Home
Q Football Taller than T S Physics Hostel
R Volleyball Shortest T Mathematics Paying guest
S Shorter than T U Statistics or History Hostel
T Tennis Shorter than U From the above chart it is clear that P studies
U Basketball Tallest English which is not given in options.
Logical Reasoning & Analytical Ability | 77

Exercise 8. Who is Poonams boy friend ?


Directions(Q. 1-5) Study the following (A) Ashok (B) Ved
information carefully and answer the questions (C) Prem (D) Raj
that follow. 9. Who does not like Shushma and Vimla ?
(i) Madhu and Shobha are good in (A) Poonam (B) Raj
Dramatics and Computer Science. (C) Ashok (D) Ved
(ii) Anjali and Madhu are good in Computer Directions(Q. 1014) Read the following
Science and Physics. information carefully and answer the questions
(iii) Anjali, Poonam and Nisha are good in that follow
Physics and History.
(i) P.Q.R.S.T and U are six students
(iv) Nisha and Anjali are good in Physics and
procuring their Masters Degree in six
Mathematics.
different subjectsEnglish History,
(v) Poonam and Shobha are good in History
Philosophy, Physics, Statistics and
and Dramatics.
Mathematics.
1. Who is good in Computer Science, History
(ii) Two of them stay in hostel, two stay as
and Dramatics ?
paying guest (PG) and the remaining
(A) Anjali (B) Madhu two stay at their home.
(C) Shobha (D) Nisha (iii) R does not stay as PG and studies
2. Who is good in Physics, Dramatics and Philosophy.
Computer Science ? (iv) The students studying Statistics and
(A) Shobha (B) Poonam History do not stay as PG.
(C) Madhu (D) Anjali (v) T studies Mathematics and S studies,
3. Who is good in Physics, History and Drama- Physics. U and S stay in Hostel. T stays
tics ? as PG and Q stays at home.
(A) Poonam (B) Shobha 10. Who studies English ?
(C) Madhu (D) Anjali (A) R (B) S
(C) T (D) U
4. Who is good in History, Physics, Computer (E) None of these
Science and Mathematics ?
(A) Poonam (B) Nisha 11. Which of the following combinaion of
(C) Madhu (D) Anjali subjects and place of stay is not correct ?
(A) English-Hostel
5. Who is good in Physics, History and (B) MathematicsPG
Mathematics but not in Computer Science ? (C) PhilosophyHome
(A) Madhu (B) Poonam (D) PhysicsHostel
(C) Nisha (D) Anjali (E) None of these
Directions(Q. 69) Read the following 12. Which of the following pairs of students stay
information carefully and answer the question that one each at hostel and at home ?
follow : (A) QR (B) SR
Four young men Raj, Prem, Ved and Ashok (C) US (D) Data inadequate
are friendly with four girls Shushma, Kusum, (E) None of these
Vimla and Poonam. Shushma and Vimla are
friends. Veds girl friend does not like Shushma 13. Which subject does Q study ?
and Vimla. Kusum does not care for Ved. Prems (A) History (B) Statistics
girls friend is friendly with Shushma. Shushma (C) History or Statistics
does not like Raj. (D) Date inadequate
6. Who is Rajs girl friend ? (E) None of these
(A) Shushma (B) Kusum 14. Which of the following pairs of students stay
(C) Vimla (D) Poonam at home ?
7. With whom Shushma is friendly ? (A) PQ (B) QR
(A) Raj (B) Prem (C) RS (D) ST
(C) Ved (D) Ashok (E) None of these
78 | Logical Reasoning & Analytical Ability

Answers with Explanation


For Question No. 1 to 5.
From the given information we can make a chart as follows :
Dramatics Compt. Sci. Physics History Mathematics
Madhu
Shobha
Anjali

Poonam
Nisha

1. (C) 2. (C) 3. (A) 4. (D) 5. (C)


For Question No. 6 to 9.
As per informtion Shushma and Vimla are friends and Prems girl firend is friendly with
Shushama. It is clear that Prems girl friend is Vimla. Veds girl friend does not like Shushma and
Vimla. Hence, she may be Kusum or Poonam. But Kusum dislikes Ved. Therefore, Veds girl friend is
Poonam. Sushma does not like Raj. So, Rajs girl friend is Kusum. Clearly, Shushma is Ashoks girl
friend.
6. (B) 7. (D) 8. (B) 9. (A)
For Question No. 1 to 14.
P Q R S T U
Subject English Statistics Philosophy Physics Mathematics Statistics
History History
Accommodation Paying guest Home Home Hostel Paying Guest Hostel

10. (E) P studies English. 11. (A)


12. (B) S stays at hostel and R stays at home. 13. (C) History or Statistics.
14. (B)

2. Blood Relations with their


Relationship and Professions
For solving this type of questions where (ii) There are two reporters, two technicians,
persons engaged in different professions and they one photographer and one writer in the
are also relatives to one another in which some group.
clues are also given regarding relationship among (iii) The photographer P is married to S who
different members of a family and their is a reporter.
professions, qualities, dresses, preferences etc. On (iv) The writer is married to Q who is in the
the basis of given informations, students are same profession as that U.
advised to draw a family tree of the generations
(v) P, R, Q, S are two married couples and
and analyse the whole information and then
nobody in the group has same profession.
answer the given questions accordingly.
(vi) U is brother of R.
Example 1. Read the following information
carefully and answer the questions that are given 1. Which is the following is a pair of techni-
below. cians?
(i) P, Q, R, S, T and U are travelling in a (A) RS (B) SU
bus. (C) PT (D) QU
Logical Reasoning & Analytical Ability | 79

2. Which of the following is a pair of reporters ? (3) Q, the lawyer, is married to P


(A) PQ (B) RT (4) P has one son and one grandson.
(C) ST (D) SU (5) Of the two married ladies one is house-
wife.
3. How is R related to U ?
(6) There is one student and one male
(A) Brother
engineer in the family.
(B) Sister
1. How is P related to R ?
(C) Uncle
(A) Grand father (B) Mother
(D) Cannot be determined
(C) Sister (D) Grand mother
4. Which of the followings is a couple ? (E) None of these
(A) PQ (B) QR
(C) QS (D) PT 2. Who among the following is housewife ?
(A) P (B) Q
5. Which of the following is a pair of husbands ?
(C) S (D) T
(A) PQ
(E) None of these
(B) PR
(C) QS 3. How R related to U ?
(D) Cannot be determiend (A) Brother
Answers with Explanation (B) Sister
(C) Brother or Sister
From the statement (iii), The Photographer is
married to S who is reporter. It means one couple (D) Data inadequate
is PS and another is (RQ) From statement (V) in (E) None of these
six persons Ps profession is photographer and Ss 4. Which of the following represents the group
profession is reporter, it means that in remaining of females in the family ?
four members, there are two technicians one
writer and one reporter. The writer is married to B (A) PSR (B) PSU
who is in same profession as that of U. Another (C) QTR (D) Data inadequate
couple is RQ is which R is writer and Q and U (E) None of these
have same profession becasue there are two
technicians. Now, only T is there and his 5. Which of the following is true about the grand
profession is reporter because there are two daughter in the family ?
reporters and we have known one reporter which (A) She is a lawyer
has been given in the statement (iii). It is clear that (B) She is a student
T must be the second reporter.
(C) She is an engineer
P Q R S T U (D) Data inadequate

(E) None of these
Photo- Tech- Writer Reporter Repor- Tech-
grapher nician ter nician Answers with Explanation
P + S First couple. There are two couples in six members as
R + Q Second couple. mentioned in statement 3. One couple is PQ and
1. (D) 2. (C) 3. (D) 4. (B) 5. (D) another couple is T and S and their two children in
which one is student or another is male engineer.
Example 2. Study the following information They are R and U. When T teacher married to S
carefully and answer the questions given below it. doctor and Q lawyer is married to P who is
(1) P, Q, R, S, T and U are six members in a housewife because one married lady S is house
family in which there are two married wife. P the houswife has one son who is married
couple. to T teacher and a grandson who is a male
(2) T, a teacher, is married to the doctor who engineer from R and U. B between R and U sex
is mother of R and U. cannot be determined.
80 | Logical Reasoning & Analytical Ability

The following chart can well elaborate the 3. How many male members are in the group ?
professions and their relations. (A) More than three
P Q R S T U (B) Two
(C) Three
House Law- Student Doctor Tech- Student/ (D) Cannot be determined
wife yer /Engi- nician Engi-
neer neer
(E) None of these
(1) Couple P + Q 4. Who is the lawyer in the group ?
(2) Couple S + T (A) C
Answers (B) F
(C) B
1. (D) P is the grand mother of R.
(D) Cannot be determined
2. (E) P is the houswife
(E) None of these
3. (C) R may be brother or Sister of U
4. (D) Since, the sex of R and U is not given so 5. Who is married to Doctor ?
the set of three females cannot be determined. (A) B
5. (B) The grand daughter is a student. (B) D
Exercise (C) F
(D) Cannot be determined
Directions(Q. 1-5) Read carefully the
following information for answering the given (E) None of these
questions. Directions(Q. 6-10) P, Q, R, S, T, V and
(i) A, B, C, D, E and F are the six members W are the members of a family. In this family
of a group. In this group, there are two there are three ladies and four gents. In this group
married couples. two members are managers, two lawyers one
(ii) There are two teachers, one doctor, one teacher, one engineer and one doctor. Neither of
lawyer and two engineers. Both the the ladies is a teacher or an engineer. R is a lawyer
teachers are of same sex. who is married with P, the teacher. Engineer V is
(iii) A and C are in the same profession. married to S who is neither lawyer nor doctor.
Now, two ladies have same profession. W, who is
(iv) Doctor is married to teacher. One
a manager and his sister is Q.
engineer is married to female lawyer.
(v) A is an engineer. E is a male teacher. 6. What is the profession of T ?
Both are unmarried. (A) Manager
(vi) F is the husband of B. (B) Lawyer
1. Which of the following is a pair of two (C) Manager or Lawyer
married couples ? (D) Data inadequate
(A) CD, FB (E) None of these
(B) CB, DF
7. What is the profession of Q ?
(C) ED, CB
(A) Doctor
(D) Cannot be determined
(B) Manager
(E) None of these
(C) Lawyer
2. What is the profession of B ?
(D) Data inadequate
(A) Lawyer
(E) None of these
(B) Doctor
(C) Teacher 8. Which of the following is a group of males ?
(D) Cannot be determined (A) PQVW
(E) None of these (B) TVSW
Logical Reasoning & Analytical Ability | 81

(C) PRTV (A) 2


(D) PTVW (B) 7
(C) 4
(E) None of these
(D) Minimum 3
9. Which of the following pairs have same
profession ? 15. If such type of dance is organized, how many
dances are possible that each man dance with
(A) PS (B) RS opposite sex ?
(C) TP (D) WT (A) 8 (B) 16
(E) None of these (C) 25 (D) 20
10. Which of the following has the same Answers with Explanation
profession which D belongs ?
For Question No. 1 to 5.
(A) R
Member Profession Sex and Relation
(B) T
A Engineer Unmarried, Male/Female
(C) S B Doctor Female, Teachers wife
(D) Data inadequate C Engineer Male, Lawyers husband
(E) None of these D Lawyer Female, Engineers wife
E Teacher Male, Unmarried.
Directions(Q. 11-15) All the questions are
based on the following information. Five men A, F Teacher Male, Doctors husband
B, C, D and E and their wives P, Q, R, S and T 1. (A) 2. (B) 3. (D) 4. (E) 5. (C)
join a dance party, but it is not necessary that the For Question No. 6 to 10.
order of husbands, and wives are the same. This is
the information that Members of the
Sex Profession
family
(i) Husbands dance only with their wives.
P Gent Teacher
(ii) C has a sister T and has only a brother A. Q Lady Doctor
(iii) If D has danced with T, It means that he R Lady Lawyer
danced with anothers wife. S Lady Manager
(iv) B does not dance with Q or T or S. T Gent Lawyer
(v) P is wife of As borther. V Gent Engineer
(vi) Q did not dance with D. W Gent Manager
6. (B) T is a lawyer.
11. With whom R danced ?
7. (A) Q is a doctor.
(A) B (B) C 8. (D) The group of gents is PTVW.
(C) D (D) E 9. (E) From the given pairs, no one has the same
12. If B and D interchange their partners who profession.
will dance with B ? 10. (C) W is a manager as well as S is the
(A) P (B) Q manager.
(C) R (D) S For Question No. 11 to 15.
The pair of Husband and Wife.
13. If E is the Ds son, how is B related to Q ?
Husband Wife
(A) Brother-in-law A Q
(B) Son B R
(C) Son-in-law C P
(D) Cannot be determined D S
14. How many members are from the same E T
family since birth ? 11. (A) 12. (D) 13. (D) 14. (D) 15. (C)
82 | Logical Reasoning & Analytical Ability

3. Seating and Placing Arrangements


In this type of questions, some information or Answers with Explanation
facts are given regarding seating or placing From the given information first of all, we see
arrangements, whether the sequence may be linear which one is positive information and we find that
or circular in which either the persons or objects 4th and 5th sentences constitute definite and
are given. The candidate is required to solve these positive information. Ist, 2nd, 6th and 7th are
questions in proper sequence and he/she can use comparative information and remaining 3rd is a
various clues and information given in the negative information. We start from the definite
question. information and draw the following diagram
There are few examples which describe the EC
seating or placing arrangement. and now with the help of comparative
ExampleStudy the following (for question information which tells about E and C from the
1-5) information carefully and answer the given 7th, 2nd and 6th sentences which lead to
question below it. Five friends A, B, C, D and E EBAC and finally with the help of 2nd
are sitting on a bench. sentence
(i) A is sitting next to B. E B A C D
(ii) C is sitting next to D. Answers
(iii) D is not sitting with E.
1. (E) 2. (A) 3. (D) 4. (B) 5. (D)
(iv) E is on the left end of the bench.
(v) C is on the second position from the Exercise
right. 1. There are five different houses A to E in a
(vi) A is on the right of B and E. row. A is to the right of B and E is to the left
(vii) A and C sitting together. of C and right of A. B is to the right of D.
1. Where is A sitting ? Which of the houses is in the middle ?
(A) Between B and D (A) A (B) B
(B) Between D and C (C) D (D) E
(C) Between E and D
Directions(Q. 2-4) Read the following
(D) Between C and E
information carefully and answer the questions
(E) Between B and C
given below it.
2. Who is sitting in the centre ? (A) There are five friends.
(A) A (B) B (B) They are standing in a row facing South.
(C) C (D) D (C) Jayesh is immediate right of Alok.
(E) E (D) Pramod is between Bhagat and Subodh.
(E) Subodh is between Jayesh and Pramod.
3. C is sitting between
(A) B and D (B) A and E 2. Who is at the extreme left end ?
(C) D and E (D) A and D (A) Alok (B) Bhagat
(E) A and B (C) Subodh (D) Data inadequate
4. What is the position of D ? (E) None of these
(A) Extreme left (B) Extreme right 3. Who is in the middle ?
(C) Third from left (D) Second from left (A) Bhagat (B) Jayesh
(E) None of these (C) Pramod (D) Subodh
(E) Alok
5. What is the position of B ?
(A) Second from right 4. To find answers to the above mentioned two
(B) Centre quesions, which of the given statements can
(C) Extreme left be dispensed with ?
(D) Second from left (A) None (B) A only
(E) None of these (C) B only (D) C only
Logical Reasoning & Analytical Ability | 83

Directions(Q. 5-9) Read the following (D) Position of J cannot be determined


information carefully and answer the questions (E) None of these
given below it. Directions(Q. 10-12) Study the given
(i) Eight persons E, F, G, H, I, J, K and L information carefully and answer the questions
are seated around a square tableTwo that follows.
on each side. (i) There are seven books, one each of
(ii) There are three lady members and they Psychology, Hindi, English, Sociology,
are not seated next to each other. Economics, Education and Accountancy
(iii) J is between L and F. lying on a table one above the others.
(iv) G is between I and F. (ii) Sociology is on the top of all books.
(v) H, a lady member, is second to the left (iii) Accountancy is immediately below
of J. Education which is immediate below
(vi) F, a male member, is seated opposite E, Sociology.
a lady member. (iv) Economics is immediately above Psy-
(vii) There is a lady member between F and chology but not in the middle.
I. (v) Hindi is immediately below Psychology.
5. Who among the following is seated between 10. Economics is between which of the following
E and H ? books ?
(A) F (A) Accountancy and Education
(B) I (B) Psychology and Hindi
(C) J (C) English and Psychology
(D) Cannot be determined (D) Psychology and Sociology
(E) None of these (E) None of these
6. How many persons are seated between K and 11. Which three books are between Accountancy
F? and Hindi ?
(A) One (A) English, Economics and Psychology
(B) Two (B) Economics, Psychology and Education
(C) Three (C) Economics, Psychology and Hindi
(D) Cannot be determined (D) Cannot be determined
(E) None of these (E) None of these
7. Who among the following are three lady 12. If Sociology and English, Accountancy and
members ? Hindi and Education and Psychology inter
(A) E, J and G change their position, which book will be
(B) E, H and G between Psychology and Sociology ?
(C) G, H and J (A) Accountancy (B) Psychology
(D) Cannot be determined (C) Hindi (D) Economics
(E) None of these (E) None of these
8. Who among the following is to the immediate Directions(Q. 13-17) Read the following
left of F ? information carefully and answer the questions
(A) G given below it.
(B) I Seven friends Kamala, Manish, Rohit, Amit,
(C) J Gaurav, Pritam and Priya are sitting in a circle.
(D) Cannot be determined Kamla, Manish , Rohit, Amit, Pritam and Priya
(E) None of these are sitting at equal distance from each other. Rohit
is sitting two places right of Pritam, who is sitting
9. Which of the following is true about J ? one place right of Amit. Kamala forms an angle of
(A) J is a male member 90 degree from Gaurav and angle of 120 degree
(B) J is a female member from Manish. Manish is just opposite Priya and is
(C) Sex of J cannot be determined sitting on the left of Gaurav.
84 | Logical Reasoning & Analytical Ability

13. Who is the only person sitting between Rohit 8. (C)


and Manish ? 9. (A) Clearly, J is a male member.
(A) Pritam (B) Amit For Question No. 10 to 12.
(C) Gaurav (D) Kamala The book of Sociology is on the top of all
14. Gaurav is not sitting at equal distance from books. Education is below Sociology. Accoun-
tancy is below Education. Economics is above
(A) Rohit and Pritam Psychlology and Psychology is above Hindi. In
(B) Amit and Kamala the statement that the Economics is not in the
(C) Manish and Pritam middle, It is clear that the remaining book English
(D) All of these must be in middle.
15. Gaurav is sitting of Priya. Thus, the sequence of books from top to
(A) To the left bottom is
(B) To the right Sociology, Education, Accoutancy, English,
(C) Two places right Economics, Psychology, Hindi.
(D) None of these 10. (C) Economics is between English and
Psychology.
16. The angle between Gaurav and Manish in the
clockwise direction is 11. (A) There are three books between Accoun-
tancy and Hindi viz., English, Economics
(A) 150 (B) 180
and Psychology.
(C) 210 (D) None of these
12. (C) The new sequence will be English,
17. Which of the following statements is not Psychology, Hindi, Sociology, Economics,
correct ? Education and Accountancy.
(A) Pritam is between Manish and Kamala Clearly, Hindi is between Psychology and
(B) Manish is two places away from Priya Sociology.
(C) Gaurav is sitting apposite Pritam
For Question No. 13 to 17.
(D) All of the above
From the information the sitting arrangement
Answers with Explanation in a circle is as follows.
1. (A) B is the right of D. A is to the right of E is
to the right of A and left of C. Therefore, the
Priya Kamala
order is
60 60
DBAEC
Pritam
Clearly, AS house is in the middle position. 60 60 Rohit
For Question No. 2 to 4.
On the basis of information we can draw the 60 30 30
Gaurav
following diagram displaying the seating Amit Ma
nis
arrangement on a square table. h

K (M) E (F)
I (M) 13. (C) Gaurav is sitting between Rohit and
H (F)
Manish.
M = Male 14. (A) Gaurav is not at equal distance from
F = Female Rohit and Pritam or Amit and Kamala or
Manish and Pritam.
L (M) G (F)
15. (D) Gaurav is three places left or four places
J (M) F (M) right of Priya.
E is seated opposite F. Since G is between F 16. (D) The angle between Gaurav and Manish in
and I. So G is a female member. clockwise direction is 30.
5. (E) 6. (C) 17. (D) Clearly, all the statements follow the
7. (B) The three lady members are E, H and G. diagram above mentioned.
Logical Reasoning & Analytical Ability | 85

4. Comparison Type Test


In Such type of questions, comparison of Hence, the sequence of height is
different objects or person has to be made with M<P<N<K<R
respect to one or more qualities. The candidates For Age sequence
are required to analyse the information and form
By (ii) we have K < R
either ascending or descending sequence. There
may be comparison of heights, marks and age etc. (iii) we have P < M
These can be illustrated through the given (iv) we have R = M
example : (v) we have R < N
ExampleRead the information given below There are two possibilities
and answer the questions that follow I is N < R = M < K < P
(i) There is a group of five girls. II is N < R = M < P < K
(ii) Kamini is second in height but younger 1. Clearly, in the increasing order of height
than Rupa. Neelam is in third position.
(iii) Pooja is taller than Monika but younger Hence, the answer is (E).
in age.
2. Clearly in the descending order of ages,
(iv) Rupa and Monika are of the same age Neelam will be in fourth position (because
but Rupa is tallest among them. Monika and Rupa both lie at third position).
(v) Neelam is taller than Pooja and elder to
3. Clearly I statement is not necessary. Hence,
Rupa.
the answer is (A).
1. If they are arranged in the ascending order of
height who will be in third positon ? Exercise
(A) Monika (B) Rupa Directions(Q. 1-5) Read the following
(C) Monika or Rupa (D) Data inadequate information and answer the questions given below
(E) None of these it :
2. If they are arranged in the descending order of (i) Seven students P, Q, R, S, T, U and V
their ages who will be in fourth positon ? take a series of tests.
(A) Monika or Rupa (ii) No two students get similar marks.
(B) Kamini (iii) V always scores more than P.
(C) Monika (iv) P always scores more than Q.
(D) Data inadequate (v) Each time either R scores the highest
and T gets the least, or alternatively S
(E) None of these scores the highest and U or Q scores the
3. To answer the question, Who is the youngest least.
person in the group, which of the given 1. If S is ranked sixth and Q is ranked fifth
statements is superfluous ? which of the following can be true ?
(A) Only (i) (B) Only (ii) (A) V is ranked first or fourth
(C) Only (v) (D) Either (i) or (iv) (B) R is ranked second or third
(E) None of these (C) P is ranked second or fifth
Answers with Explanation (D) U is ranked third or fourth
(E) T is ranked fourth or fifth
We have to determine the sequence of height
as well as sequence of age. 2. If R gets the most, V should be ranked not
By (iii), We have M < P lower than
By (v), We have P < N (A) Second (B) Third
It is clear that Rupa is the tallest and Kamini (C) Fourth (D) Fifth
is second in height. (E) Sixth
86 | Logical Reasoning & Analytical Ability

3. If R is ranked second and Q is ranked fifth, (D) P gets more than V


which of the following must be true ? (E) T gets more than Q
(A) S is ranked third 5. If V is ranked fifth, which of the following
(B) T is ranked sixth must be true ?
(C) P is ranked sixth (A) S scores the highest
(D) V is ranked fourth (B) R is ranked second
(E) U is ranked sixth
(C) T is ranked third
4. If S is ranked second, which of the following (D) Q is ranked fourth
can be true ? (E) U scores the least
(A) U gets more than V
(B) V gets more than S Answers
(C) P gets more than R 1. (D) 2. (C) 3. (B) 4. (A) 5. (A)

5. Sequential Order of Things


In this type of question, some hints are given 3. At what time the visit of Doctor T would be
regarding the occurrence of certain events. The over on Sunday ?
candidate is required to go through the given (A) 10 A.M. (B) 11 A.M.
information and make the right sense and then (C) Either 10 A.M. or 11 A.M.
answer the pertaining questions accordingly.
(D) Data inadequate
ExampleRead the following information
carefully and answer the questions given below it. (E) None of these
(i) Eight doctors P, Q, R, S, T, U, V and W 4. If the lunch break and subsequent visiting
visit a charitable dispensary every day hours are reduced by 15 minutes. At what
except on a holiday i.e., Monday. time doctor U is expected to attend the
(ii) Each doctor visits for one hour from dispensary ?
Tuesday to Sunday except Saturday. The (A) 315 P.M. (B) 4 P.M.
timings are 9 A. M. to 1 P. M. and 2
P.M. to 6 P.M., 1 P.M. to 2 P.M. is lunch (C) 415 P.M. (D) 445 P.M.
break. (E) None of these
(iii) On Saturday it is opened only in the Answers with Explanation
morning i.e., 9 A.M. to 1 P.M. and each
doctor visits for only half an hour. We can make the time sequence by their visits
(iv) No other doctor visits the dispensary using (iv), (v) and (vi).
before doctor Q and after U. It is clear that Q visits first and R visits last
(v) Doctor W comes immediately after the from (iv)
lunch break and is followed by R. From (v), It is also clear that W visits first
(vi) S comes in the same order as P in the after break and is followed by R.
afternoon session. From (vi) We know that P visits after break
1. Doctors P visits in between which of the means after W and Rs visits.
following pairs of doctors ? Thus, the sequence of visits after lunch break
(A) S and V (B) U and W will be WRPU.
(C) R and W (D) R and U Also, S has the same position in morning
(E) None of these session as P in afternoon session means third in
2. At what time the visit of doctor R is over morning session.
Sunday ? The morning session sequence will be Q,
(A) 1 P.M. (B) 3 P.M. T/V, S, V/T because the position of V and T is not
(C) 4 P.M. (D) 5 P.M. clear.
(E) None of these 1. (D) P visits between R and U.
Logical Reasoning & Analytical Ability | 87

2. (C) W visits between 2 to 3 P.M. and R visits Directions(Q. 5-9) Read the following
between 3 P.M. to 4 P.M. Hence, the visits of information carefully and answer the questions
R is over at 4 P.M. that follow.
Six lectures A, B, C, D, E and F are to be
3. (C) Clearly T visits either second or fourth.
organised in a span of seven days from Sunday to
The time of visit on Saturday will be either
Saturday. Only one lecture on each day in
930 A.M. or 1030 A.M. It means that Ts
accordance with the following
visit will be over at either 10 A.M. to 11 A.M.
(i) A should not be organised on Thursday.
4. (B) As mentioned the time of lunch break is (ii) C should be organised immediately after
over and doctor W visits at 145 P.M., Doctor F.
R will visit at 230 P.M., Doctor will visit at (iii) There should be a gap of two days
315 P.M. and U will visit at 4 P.M. between E and D.
Exercise (iv) One day there will be no lecture (Friday
is not that day), just before that day D
1. Five boys took part in a race. Raj finished will be organised.
before Mohit but behind Gaurav. Ashish
(v) B should be organised on Tuesday and
finished before Sanchit but behind Mohit.
should not be followed by D.
Who won the race ?
5. On which day there is no lecture ?
(A) Raj (B) Gaurav
(A) Monday
(C) Mohit (D) Ashish
(B) Friday
Directions(Q. 2-4) Read the following (C) Sunday
information carefully and answer the questions (D) Cannot be determined
that follow (E) None of these
Five plays A, B, C, D and E are to be staged
6. How many lectures are organised between C
from Monday to Friday of a week. On each day,
and D ?
only one play will be staged. D or E should not be
either the first or last to be staged. E should be (A) None (B) One
immediately followed by C. B should be staged (C) Two (D) Three
immediately after D. One play is staged between (E) None of these
A and B. 7. Which day will the lecture F be organised ?
2. Which is the first play to be staged ? (A) Thursday (B) Friday
(A) A (C) Saturday (D) Sunday
(B) B (E) None of these
(C) C 8. Which of the following is the last lecture in
(D) Cannot be determined the series ?
(E) None of these (A) A
(B) B
3. Which of the following is the correct (C) C
sequence of staging all the plays ? (D) Cannot be determined
(A) ADBCE (B) AECDB (E) None of these
(C) BDAEC (D) DBECA
9. Which of the following informations is not
(E) None of these required in finding the complete sequence of
4. Which play was staged on Wednesday ? organisation of lectures ?
(A) A (A) (i) only
(B) B (B) (ii) only
(C) Either B or C (C) (i) and (ii) only
(D) Cannot be determined (D) (v) only
(E) None of these (E) All are required
88 | Logical Reasoning & Analytical Ability

Answers with Explanation Sunday. There is a gap of two days between D and
E. So E will be organised on Wednesday because
1. (B) The order is Gaurav, Raj, Mohit, Ashish,
A cannot be organised on Thursday and C should
Sanchit. Hence, Gaurav won the race.
be organised immediately after. Clearly F and C
For Question No. 2 to 4. will be organised on Thursday and Friday
Monday Tuesday Wednesday Thursday Friday respectively. Hence, A will be organised on
A D B E C
Saturday. So, the correct sequence is
Sun- Mon- Tues- Wednes Thurs- Fri- Satur
2. (A) A is the first play to be staged. day day day -day day day -day
3. (E) The proper sequence is ADBEC. D B E F C A
4. (B) B was staged on Wednesday.
5. (A) No lecture on Monday.
For Question No. 5 to 9.
6. (C) There are three lectures between C and D
From the statement, B is organised on i.e., B.E.F.
Tuesday. B can not be followed by D. It means
there are two days before B in which any lecture 7. (A) F is organised on Thursday.
can be possible but D is followed by the day with 8. (A) A is the last lecture.
no lecture. It is clear that D will be organised on 9. (E) All the statements are required.

6. Selection Based on Certain Given Pre-conditions


This type of question is solved by keeping in 3. If two of the members are girls and D is one
mind various given pre-conditions and a few of the members, the members of the team
essential criteria for selection of group items. The other than D are
candidate has to make the required selection as per (A) PQBC (B) PQCE
the directions given in each question.
(C) PSAB (D) PSCE
Example 4. If A and C are the members, the other
Directions(Q. 1-5) Study the following members of the team cannot be
information carefully and answer the questions (A) BES (B) DES
below.
(C) ESP (D) PQE
A team of five is to be selected from amongst
five boys A, B, C, D and E and four girls P, Q, R 5. If including P at least three members are girls,
and S. Some criteria for selection are the members of the team other than P are
A and S have to be together (A) QSAB (B) QSBD
P cannot be put with R (C) QSCE (D) RSAD
D and Q cannot go together
C and E have to be together Answers with Explanation
R cannot be put with B 1. (A) If A is selected naturally S has to be
Unless otherwise stated, these criteria are appli- selected.
cable to all questions below. If B is selected R cannot be selected.
1. If two of the members have to be boys, the If D is selected Q cannot be selected.
team will consist of
So, the options ADSQR and BDSRQ are
(A) ABSPQ (B) ADSQR wrong and CESPQ is not possible because S
(C) BDSRQ (D) CESPQ has to be accompanied with A.
2. If R be one of the members, the other 2. (D) If R is selected P cannot be selected,
members of the team are therefore option PSAD is wrong.
(A) PSAD (B) QSAD D and Q cannot go together. So, QSAD is
(C) QSCE (D) SACE wrong.
Logical Reasoning & Analytical Ability | 89

S and A have to be together. So, QSCE is At an electric Data Processing Unit five out of
wrong. Now, the team combination will be the eight program sets P, Q, R, S, T, U, V and W
SACE are to be operated daily. On any one day except
3. (C) If D is selected Q cannot be selected. for the first day of the month only three of the
therefore PQBC and PQCE are not correct. S program sets must be the ones that were operated
and A have to be always together. on the previous day. The program operating must
Therefore, PSCE is wrong. also satisfy the following conditions.
4. (D) If A and C are the members, S and E have (i) If program P is to be operated on a day,
to be selected. Therefore, option (D) PQE is V cannot be operated on that day.
not the correct combination. (ii) If Q is to be operated on a day, T must
5. (A) be one of the programs to be operated
after Q.
Exercise (iii) If R is to be operated on a day, V must
Directions(Q. 1-5) Read the following be one of the programs to be operated
information carefully and answer the questions after R.
given below it. (iv) The last program to be operated on any
Eight students A, B, C, D, E, F, G and H are day must be either S or U.
planning to enjoy car racing. There are only two 4. Which of the following could be the set of
cars and following are the conditions. program to be operated on the first day of the
(i) One car can accommodate maximum month ?
five and minimum four students. (A) V Q R T S (B) U Q S T W
(ii) A will sit in the same car in which D is (C) T U R V S (D) Q S R V U
sitting but H is not in the same car. (E) P R V S U
(iii) B and C cannot sit in the same car in 5. Which of the following is true of any days a
which D is sitting. valid program set operation ?
(iv) F will sit in the car of four people only (A) P cannot be operated at third place
along with A and E but certainly not (B) Q cannot be operated at third place
with G. (C) R cannot be operated at fourth place
1. If H and G are sitting in the same car, who are (D) T cannot be operated at third place
other two students sitting in the same car ? (E) U cannot be operated at fourth place
(A) B and C (B) C and D
6. If R is operated at third place in a sequence,
(C) B and D (D) E and B which of the following cannot be the second
(E) None of these program in that sequence ?
2. If E and A are sitting in the same car, which (A) Q (B) S
of the following statements is true ? (C) T (D) U
(A) Five students are sitting in the same car (E) W
(B) B is sitting in the same car 7. If the program sets R and W are to be
(C) F is not sitting in the same car operated on the first day which of the
(D) G is not sitting in the same car following could be the other programs on that
day ?
(E) None of these
(A) P, T, V (B) Q, V, S
3. Which of the following statements is super- (C) Q, T, V (D) T, S, U
fluous for the above sitting arrangements ? (E) T, S, V
(A) Only (i) (B) Only (ii) 8. If the program sets operated on a day is P, Q,
(C) Only (iii) (D) Only (iv) W, T, U each of the following could be the
(E) None of these next days program set except.
Directions(Q. 4-8) Study the following (A) W, T, U, V, S (B) W, T, S, P, U
information carefully and answer the questions (C) W, R, V, T, U (D) Q, T, V, W, S
that follow. (E) Q, R, V, T, U
90 | Logical Reasoning & Analytical Ability

Answers with Explanation For Question No. 4 to 9.


For Question No. 1 to 3. 4. (C)
There are two cars I and II.
5. (C) R cannot be operated at fourth place.
A and D sit in the same car, means, in I car H
cannot sit in the same car i.e., H will sit in car II B 6. (A)
and C are not in the same car in which D is sitting 7. (E) Since R is operated, so V must also be
i.e. B and C will sit in car II. operated.
G is also in other car, means car II.
Also, S or U is to be taken at the end.
Now, we have
Therefore, the possible combinations are QSV
Car I ADEF
and TSV.
Car II BCGH
1. (A) B and C are in the same car in which G Now, Q must have also T as one of the
and H are there. program after it which is not possible. Hence,
2. (D) Clearly G is not sitting. Q, S, V is incorrect.
3. (A) State I is superfluous, it is not necessary. 8. (B)

7. Jumbled Problems
Example I 1. What is the colour of Kapils shirt ?
Direction(Q. 1-3) Read the following (A) White (B) Green
information carefully and answer the questions (C) Blue (D) Data inadequate
mentioned below. (E) None of these
(i) Five friends Amar, Kapil, Sarvesh, Rohan
2. Who likes reading ?
and Nagesh put on five shirts of different
colours i.e., Red, Yellow, Blue, White and (A) Rohan (B) Amar
Green, while they were going to attend a (C) Kapil (D) Data inadequate
party. These colours are not in order. (E) None of these
(ii) They have different hobbies as Reading, 3. Which is the following combination of
Playing, Outing, Singing and Writing. Person-Colour-Hobby is correct ?
(iii) Kapil, who likes singing, does not wear (A) Rohan - Blue - Reading
yellow shirt. Sarvesh wears red shirt and he
(B) Nagesh - White - Playing
does not like reading or writing. Nagesh
likes playing and he does not wear blue or (C) Amar - Yellow - Writing
yellow shirt. Amar likes writing and Rohan (D) Nagesh - Green - Playing
does not wear yellow or green shirt. (E) None of these

Answers with Explanation


Hobbies Colour of Shirt

Reading Playing Outing Singing Writing Red Yellow Blue White Green
Amar
Kapil
Sarvesh
Rohan
Nagesh
Logical Reasoning & Analytical Ability | 91

1. (D) The colour of Kapils shirt may be either 1. Who is doctor among these friends ?
Blue or White or Green. Hence, data are (A) Data inadequate
inadequate.
(B) Neha
2. (A) Rohans hobby is reading.
(C) Yogita
3. (C) The combination Amar-yellow-writing is
(D) Smita
correct.
(E) None of these
Example II
2. What is the hobby of Yogita ?
Directions(Q. 1-4) Read the following
(A) Drama (B) Reading
information carefully and answer the questions.
(C) Dancing (D) Singing
Neha, Charulata, Vaidehi, Yogita and Smita
(E) None of these
are five friends. They have five different occupa-
tionsDoctor, Lawyer, Architect, Beautician and 3. What is the occupation of Vaidehi ?
Computer Engineer. All the five friends have (A) Doctor
different hobbiesDrama, Reading, Dancing, (B) Computer Engineer
Cooking and Singing (It is not necessary that the
name of the persons, their hobbies and occupa- (C) Architect
tions are in same order.) (D) Data insufficient
Neha is a lawyer whose hobby is cooking. (E) None of these
Vaidehi is neither a doctor nor a beautician. She 4. Who likes Singing ?
likes dancing. Yogita is neither a architect nor a (A) Charulata (B) Vaidehi
doctor. She does not like drama nor sing a song.
Charulata is a Computer engineer who does not (C) Yogita (D) Neha
like Drama. (E) None of these

Answers with Explanation


Name Occupation Hobby
S.N. of the Archi- Beau- Computer Read- Dan- Cook- Sing-
Docto Lawyer Drama
Person tect tician Engineer ing cing ing ing
r
1. Neha

2. Charu-
lata
3. Vaidehi

4. Yogita

5. Smita

1. (D) 2. (B) 3. (C) 4. (A)


Example III
Directions(For question 1-4) Study the following information carefully and answer the questions
below mentioned.
(i) Seven subjects viz., Sociology / Psychology / English / History / Geography / Economics / Hindi
are taught by persons A, B, C, D and E from Monday to Friday.
(ii) Each person teaches at least one subject. At least one subject is taught each day. No one teaches
two subjects in one day. B teaches Sociology on Wednesday, History is taught by E but not on
Monday or Thursday. English is taught by A on Monday. Geography and Economics are taught on
Monday and Tuesday respectively. D teaches only Psychology on Tuesday. Geography is not
taught by E or B.
92 | Logical Reasoning & Analytical Ability

1. Who teaches Economics ?


(A) E (B) A (C) B
(D) Cannot be determined (E) None of these
2. Who teaches Geography ?
(A) C (B) E (C) B
(D) Cannot be determined (E) None of these
3. Which subject is taught on Thursday ?
(A) History (B) Economics (C) Geography
(D) Cannot be determined (E) None of these
4. Which subject is taught on Friday ?
(A) Hindi (B) Economics (C) History
(D) Cannot be determined (E) None of these
Answers with Explanation
Sociology Psychology English History Geography Economics Hindi
Monday
Tuesday
Wednesday
Thursday
Friday
A
B
C
D
E

1. (D) 2. (A) 3. (E) 4. (C)


Exercise on Friday and the person who avails holiday on
Saturday does not work with T. Z works neither in
Directions(Q. 1-5) Study the following II nor in III shift.
information and answer the questions below 1. On which day is the holiday of Q ?
mentioned. (A) Wednesday (B) Thursday
P, Q, R, S,T, V and Z are seven employees in (C) Tuesday (D) Saturday
a call centre. These people work in three shifts (E) None of these
i.e., I, II and III. Any shift comprises at least one 2. Which of the following three work in the
but not more than three employees. Each gets one shift ?
day holiday in a week from Monday to Sunday. Q (A) I (B) II
works in II shift with T and he gets weekly (C) III (D) I or III
holiday immediate after Ps holiday. S avails (E) Data inadequate
holiday on Sunday but he is not engaged in his
duty with R and Qs shift. P along with R is in 3. Which of the following gets the holiday
shift I. Rs holiday is immediate after Qs holiday immediate after Vs holiday ?
and immediate before Ts holiday. Vs holiday is (A) P (B) Z
immediate after Ts holiday but not on Saturday. (C) T (D) Data inadequate
The employee who works in shift III, gets holiday (E) None of these
Logical Reasoning & Analytical Ability | 93

4. In which shift does V work ? Directions(Q. 11-15) Study the following


(A) II (B) III information carefully and answer the question
(C) II or III (D) Data insufficient given below it.
(E) None of these Of the five boys A, B, C, D and E two are
5. Which of the following combination of good, one is poor and two are average in studies.
Employee-holiday-shift is not correct ? Two of them study in post-graduate classes and
three in under graduate classes. One comes from a
(A) T Thursday II
rich family, two from middle class families and
(B) B Friday III two from poor families. One of them is interested
(C) P Wednesday I in music, two in acting and one in sports. Of those
(D) T Saturday I studying in under graduate classes, two are
(E) All are correct average and one is poor in studies. Of the two
Directions(Q. 6-10) Study the following boys interested in acting, one is a post-graduate
information carefully and answer the questions student. The one who is interested in music comes
given below it. from a middle class family. Both of the boys
There are five friends A, B, C, D and E. Two interested in acting are not industrious, good in
of them are businessmen while the other three studies, come from a middle-class families are
belong to different occupations viz., medical, average in studies and one of them is interested in
engineer and legal. One businessman and the acting. The boy interested in sports comes from a
lawyer stay in the same locality while the other poor family, while the one interested in music is
three stay in three different localities P, Q and R. industrious. E is industrious, good in studies
Two of these five persons are Hindu while comes from a poor family and is not interested in
remaining three come from three different acting, music or sports. C is poor in studies in
communities viz., Muslim, Christian and Sikh. spite of being industrious. A comes from a rich
The lawyer is the oldest in age while one of the family is not industrious but good in studies. B is
businessmen who runs a factory is the youngest. industrious and comes from a middle class family.
The other businessman is a cloth merchant and 11. Name of the boy interested in sports.
agewise lies between the doctor and the lawyer. D (A) A (B) B
is a cloth merchant and stays in locality S while E
is a Muslim and stays in locality R. The doctor is a (C) C (D) D
Christian and stays in locality P, B is a Sikh while 12. Name of the boy interested in music.
A is Hindu and runs a factory.
(A) A (B) B
6. Who stays in locality Q ?
(C) C (D) D
(A) A (B) B
(C) C (D) E 13. Name of middle class family boy interested in
acting.
7. What is Es occupation ?
(A) Business (B) Engineer (A) A (B) B
(C) Lawyer (D) Doctor (C) C (D) D
8. Agewise who among the following lies 14. Name of the boys studying in post-graduate
between A and C ? class.
(A) Lawyer (B) Doctor (A) A, D (B) A, E
(C) Cloth Merchant (D) Engineer (C) B, C (D) D, E
9. What is Bs occupation ? 15. Name of the boy who is not industrious and is
(A) Business (B) Engineer average in studies.
(C) Lawyer (D) Doctor (A) A (B) B
10. What is Cs occupation ? (C) C (D) D
(A) Doctor (B) Lawyer Directions(Q. 16-19) Read the following
(C) Engineer (D) Business information and answer the question given below.
94 | Logical Reasoning & Analytical Ability

(i) Seven friends P, Q, R, S, T, U and W have 18. R has specialization in which of the
gathered at Mumbai airport. Five of them following fields ?
are scheduled to go to five different places (A) Finance
Delhi, Chennai, Lucknow, Banglore and
Calcutta. (B) Marketing
(ii) Five of them are executives, each speciali- (C) Either marketing or Finance
sing in Administration, Human Resource (D) None of these
Management, Marketing, System and (E) Cannot be determined
Finance.
19. Who is going to fly Banglore ?
(iii) T, an executive, is going to Chennai and
neither is from Finance nor Marketing. (A) Data inadequate
(iv) W is a System Specialist and is leaving for (B) R
Delhi. U is an executive but is not going to (C) S
one of the five places. (D) P
(v) Q is an executive from HRM but has come
(E) None of these
to the airport to see his friends.
(vi) P is an executive but not from Marketing Answers with Explanation
and is flying to one of the destinations but For Question No. 1 to 5.
not to Banglore or Calcutta.
16. The one who is going to fly Chennai is Employee Shift Holiday
(A) Not an Executive I II III
(B) From Administration P Monday
(C) From Management
(D) From Finance Q Tuesday
(E) None of these R Wednesday
17. Who among the following specializes in S Sunday
Marketing ?
T Thursday
(A) S
(B) P V Friday
(C) U Z Saturday
(D) Data inadequate
(E) None of these 1. (C) 2. (A) 3. (B) 4. (B) 5. (C)

For Question No. 6 to 10.

A B C D E
Profession Factory owner Lawyer Doctor Cloth Merchant Engineer
Religion Hindu Sikh Christian Hindu Muslim
Locality Q S P S R
Age ranking in descending order 5 1 3 2 4

6. (A) A stays in locality Q.


7. (B) E is an engineer.
8. (D) E lies between A and C. E is an Engineer.
9. (C) B is a lawyer.
10. (A) C is a doctor.
Logical Reasoning & Analytical Ability | 95

For Question No. 11 to 15.


Performance in A B C D E
Studies Good Ave. Poor Ave. Good
Class (Studying) PG UG UG UG PG
Family Status Rich Middle Poor Middle Poor
Interest Acting Music Sports Acting
Industrious/Not Industrious Not Industrious Industrious Industrious Not Industrious Industrious

11. (C) C interested in Sports.


12. (B) B is interested in Music.
13. (D) D is middle class family boy interested in Acting.
14. (B) A & E study in Post Graduate classes.
15. (D) D is not Industrious and average in studies.
For Question No. 16 to 19.
P Q R S T U W

Destination Lucknow Bangalore/ Bangalore/ Chennai Delhi


Calcutta Calcutta

Executives in
Finance HRM Administration Not confirmed System
their specialized

16. (B)
17. (D) Data inadequate.
18. (A) None of these
19. (A) Data inadequate.

You might also like